Download as pdf or txt
Download as pdf or txt
You are on page 1of 97

By Vishal sir

SBI PO, SBI CLERK, IBPS PO, IBPS CLERK, RRB PO, RRB CLERK, NIACL,
- LIC RBI grade B, RBI ASSISTANT, & Other competitive Exams

Facebook Page- https://www.facebook.com/vishalpariharpage Youtube- https://www.youtube.com/vishalparihar


Follow
1 Vishal Sir Telegram Channel- https://t.me/englishbyvishalsirchannel Instagram- https://www.instagram.com/vishalthetrainer
By Vishal sir

SBI PO, SBI CLERK, IBPS PO, IBPS CLERK, RRB PO, RRB CLERK, NIACL,
- LIC RBI grade B, RBI ASSISTANT, & Other competitive Exams

Facebook Page- https://www.facebook.com/vishalpariharpage Youtube- https://www.youtube.com/vishalparihar


Follow
2 Vishal Sir Telegram Channel- https://t.me/englishbyvishalsirchannel Instagram- https://www.instagram.com/vishalthetrainer
By Vishal sir

SBI PO, SBI CLERK, IBPS PO, IBPS CLERK, RRB PO, RRB CLERK, NIACL,
- LIC RBI grade B, RBI ASSISTANT, & Other competitive Exams

Dear Students,

In these times of immense competition, success is not something that’s easier to


achieve. You have to excel your preparation and perform exceptionally well at various
stages. And to achieve this you need the best material present out there.

With an initiative to provide you with the best, Vishal Parihar has come with this new
book ‘500+ Brainy Fillers’ for all competitive exams. The 500+ Brainy Fillers in this series are
the most useful questions in English. They were found by analysis of a collection of different
previous year papers from different competitive examinations like SBI, IBPS, CAT, MAT, and
various insurance examinations. Each question in this book is a high-frequency question.
This means that the effort in solving the filler questions is well repaid by the number of times
learners have a chance to encounter or solve them. You can rely on this book to improve
your solving ability that will help you to achieve better scores in the examinations

We would like to take this opportunity to thank our team (Bhavesh Patni, Aanchal Singh,
Rishika Jain) for their relentless efforts towards the design and the quality content of the
book.

Good Luck!

Vishal Parihar

Facebook Page- https://www.facebook.com/vishalpariharpage Youtube- https://www.youtube.com/vishalparihar


Follow
3 Vishal Sir Telegram Channel- https://t.me/englishbyvishalsirchannel Instagram- https://www.instagram.com/vishalthetrainer
By Vishal sir

SBI PO, SBI CLERK, IBPS PO, IBPS CLERK, RRB PO, RRB CLERK, NIACL,
- LIC RBI grade B, RBI ASSISTANT, & Other competitive Exams

Dedication

I dedicate this book to "my parents

and my family" for their relentless

support. Hope this book will help

students to achieve their goal.

Facebook Page- https://www.facebook.com/vishalpariharpage Youtube- https://www.youtube.com/vishalparihar


Follow
4 Vishal Sir Telegram Channel- https://t.me/englishbyvishalsirchannel Instagram- https://www.instagram.com/vishalthetrainer
By Vishal sir

SBI PO, SBI CLERK, IBPS PO, IBPS CLERK, RRB PO, RRB CLERK, NIACL,
- LIC RBI grade B, RBI ASSISTANT, & Other competitive Exams

The information you will find in this book and any publications or other study materials published by Vishal Parihar

is for information only, which is to be used for learning English grammar.

Tips, strategies, methods of study, lessons and all the resources are only recommendations for the aspirants, and

reading any information in this book is provided ‘as is’.

The whole book is made keeping in mind, all the previous years questions asked in bank & other relevant

exams and expected to be asked in the upcoming exams. The author has made reasonable efforts to provide current

and accurate information to his readers. The author will not be held liable for any unintentional errors or omissions

that may be found.

Facebook Page- https://www.facebook.com/vishalpariharpage Youtube- https://www.youtube.com/vishalparihar


Follow
5 Vishal Sir Telegram Channel- https://t.me/englishbyvishalsirchannel Instagram- https://www.instagram.com/vishalthetrainer
( NEW)

JOIN OUR TELEGRAM CHANNEL


BANKING VIDYARTHI
BANKING VIDYARTHI FOR EXAM
PD
EXAM NEWS
THE HINDU NEWSPAPAR DAILY
AT 4-5 AM
SEARCH FOR CLICK HERE
( TELEGRAM CHANNEL )
BANKING VIDYARTHI4
( YOUTUBE CHANNEL )
BANKING VIDYARTHI
By Vishal sir

SBI PO, SBI CLERK, IBPS PO, IBPS CLERK, RRB PO, RRB CLERK, NIACL,
- LIC RBI grade B, RBI ASSISTANT, & Other competitive Exams

Directions (1-5): (c) contradictory, attention, intrude


In each of the following sentence there are three blank spaces. (d) inflicted, abstention, ostracized
Below ach sentence there are five options and each option (e) enforced, extrapolation, snubbed
Q5. The best place to explore the _________ of a people is in their art,
consists of three words which can be filled up in the blanks in
especially _________, for creative artists are _________ to the
the sentence to make the sentence grammatically correct.
implications of the culture on life.
(a) the, deemed, affable
Q1. Altogether, a good portion of the factory workers and others caught (b) spirit, literature, sensitive
up in the downsizing adjustments to a new, _______ competitive (c) this, contemplated, gruesome
economy have grown _________ and suspicious of their leaders (d) recent, reckoned, bland
and their government, regarding them as ________ or even (e) while, estimated, grim
destructive
(a) fiercely, contemptuous, irrelevant Answers And Solutions
(b) Courageously, acute, mien
(c) Resolutely, subtle, demeanor Q1Ans. (a)
(d) Boldly, keen, plan Fiercely- to a high degree
(e) Securely, judicious, point Contemptuous- manifesting, feeling, or expressing deep hatred or
disapproval
Q2. It is unwise to let our _______ for universal ________ overcome Acute- very serious
our capability for progress because in the majority of cases this Mien- the appearance, behaviour or typical expression of a person
________ on the opinions of others makes us lose the focus on our that shows their real feelings
own cases apirations and work output. Demeanor- the way a person behaves, dresses, speaks, looks, etc.
(a) Straight, amity, fiercely
that show what their character is like
(b) Completely, civility, riotously
Q2Ans. (d)
(c) Whole, committee, mildly
(d) yearning, approbation, concentration Fiercely- to a high degree
(e) Entirely, concord, tamely Civility- a polite act or expression
Riotously- in a very loud and uncontrolled way that is full of energy
Q3. The slowdown is visible, it is fairly _______, besides our own Yearning- to have an earnest or strong desire
Approbation- approval or praise
concerns that internally we have also global headwinds. There are
Concord- peace and agreement
also _______on account of the global geopolitical _______and also
Tamely- Naturally unafraid
climate change — that is important to us given that agriculture is
Q3Ans. (d)
very important for India.
In the given sentence, only ‘widespread, vulnerabilities, situation’
A.widespread, vulnerabilities, idiocy
make it grammatically as well as contextually correct.
B.judgement, lunacy, idiocy
Option A): is incorrect because idiocy does not fit here.
C.judgement, lunacy, situation
Option B): is incorrect as ‘judgement’ does not make any sense
D.widespread, vulnerabilities, situation
here.
E.None of these
Option C): is incorrect as ‘lunacy’ means the state of being a
lunatic; insanity (not in technical use) which does not fit here.
Q4. He combined many ________ skills like broad sweeps of the range
Option D): is the correct alternative among the following as
with _______ to details. Accessibility to all and yet not letting
‘widespread, vulnerabilities, situation’ fit here both grammatically
anyone ________ on his time.
and contextually.
(a) nibbled, notion, sanctioned
Lunacy means very stupid behaviour
(b) revoked, surmise, comprehended

Facebook Page- https://www.facebook.com/vishalpariharpage Youtube- https://www.youtube.com/vishalparihar


Follow
6 Vishal Sir Telegram Channel- https://t.me/englishbyvishalsirchannel Instagram- https://www.instagram.com/vishalthetrainer
By Vishal sir

SBI PO, SBI CLERK, IBPS PO, IBPS CLERK, RRB PO, RRB CLERK, NIACL,
- LIC RBI grade B, RBI ASSISTANT, & Other competitive Exams

Vulnerability means the quality of being easily hurt or attacked. (c) measure, external (d) destination, magnify
Idiocy means extremely stupid behaviour. (e) appreciation, percolate
Q4Ans. (c)
Nibbled- to eat something by taking small bites Q9. In contrast to the India-Pakistan matches in the past, the current
Surmise- to guess or suppose that something is true without series ________ bonhomie but there is, of course, the tension of the
definitely knowing game and _______ are no doubt around.
Comprehended - grasp mentally; understand. (a) facility, evolves
Abstention- the act or practice of choosing not to do or have (b) development, injects
something (c) exuded, passions
Ostracized- exclude from a society or group. (d) input, develops
Extrapolation- to predict by projecting past experience or known (e) element, exhibits
data
Q5Ans. (b) Q10. Occassional short-lived stress usually is not a problem but when
Affable - friendly, good-natured, or easy to talk to. stress becomes ________, the immune system can begin to become
Gruesome - causing repulsion or horror; grisly. ________ in a major way.
While- in spite of the fact that; although (a) refutable, disturbed (b) questionable, formidable
Estimated- roughly calculate or judge the value, number, quantity, (c) creditable, forbidden (d) viable, enlarged
or extent of (e) chronic, impaired
Grim- very serious or gloomy Q11. The U.S. chamber which is prepared to _______ public debates to
Directions (6-10): Each question below has two blanks. There are five demolish the myths about outsourcing, says that to create jobs it is
pairs of words below the sentence. Each pair is lettered. Choose _______ that America remains open to the world economy where
the pair of words which can be filled up in the blanks in the 95 percent of its potential customer live.
sentence in the same order so as to complete the sentences (a) management, supreme
meaningfully. (b) improving, low
(c) higher, vulnerable
Q6. For all that the term _________, the people seem pretty displeased
(d) organize, critical
with modern democracy and they hold politicians in varying
(e) better, supreme
degrees of _______.
(a) interfere, refused (b) revealed, classified Answers And Solutions
(c) signifies, contempt (d) impeded, excluding
(e) withdrawn, leaked Q6Ans. (c)
Contempt- the feeling that somebody/something does not deserve
Q7. The revolutionaries of recent years cry out for freedom and any respect or is without value
Impeded- to make it difficult for somebody/something to move or
________ as did our forefathers, but the concept of _______ has
go forward
undergone a radical transformation along with the concept of Q7Ans. (d)
morality. Stipulated- to say exactly and officially what must be done
(a) cooperate, stipulated (b) declared, pressurized Inculcate- to make somebody learn or remember ideas, habits, etc.
(c) inculcate, ordered (d) individualism, liberty especially by repeating them often
(e) encourage, enforced
Q8Ans. (c)
Q8. However, moving further away from training inputs makes it more Percolate- to ooze or trickle through a permeable substance
Q9Ans. (c)
cumbersome to ________ the effects of training as these effects
Exuded- if you exude a particular attitude, feeling or quality, people
become perplexing by an increasing number of ________
can easily see that you have a lot of it.
influences.
Q10Ans. (e)
(a) achievement, crumble (b) realization, damage
Facebook Page- https://www.facebook.com/vishalpariharpage Youtube- https://www.youtube.com/vishalparihar
Follow
7 Vishal Sir Telegram Channel- https://t.me/englishbyvishalsirchannel Instagram- https://www.instagram.com/vishalthetrainer
By Vishal sir

SBI PO, SBI CLERK, IBPS PO, IBPS CLERK, RRB PO, RRB CLERK, NIACL,
- LIC RBI grade B, RBI ASSISTANT, & Other competitive Exams

Refutable- To deny the accuracy or truth of (e) disheartning………explain


Chronic- that continues for a long time
Impaired- diminished or damaged Answers And Solutions
11Ans. (d)
Vulnerable- weak and easy to hurt physically or emotionally Q12Ans. (d)
In the first blank the word enough makes a sensible statement and
Directions (12-16): Each sentence below has one or two blanks, each burdened completes the second black. Hence option (d) makes a
blank indicating that something has been omitted. Beneath the meaningful sentence.
sentence are four sets of words. Choose the word or set of words fascination means the quality or power of fascinating
for each blank that best fits the meaning of the sentence as a oppressed means to crush or burden by abuse of power or authority
whole. restitute means to restore to a former state or position
Q12. The problem is that not ________ faculty members are hired, and 13Ans. (b)
that those hired are _________ with additional tasks such as The answer is option (b), as the first statement talks about new
running the canteen, etc. technologies, therefore the appropriate word for first blank should
(a) fascination with…….corrupted be invent. In the second statement, the word demolished makes it a
(b) admission of……..oppressed meaningful sentence.
(c) flight from……restitute Detriment means the state of being harmed or damaged.
(d) enough………………burdened Demolish means to destroy something
(e) rejection of……………made 14Ans. (c)
The statement talks about the location of the University which is
situated in the corner of its territory. Here the word remote makes
Q13. A waning fantasy is that only large companies and organisations an appropriate sentence. Hence the correct answer is option (c).
can _______ new technologies. This belief has been _________ by 15Ans. (c)
so many start-ups in the past few decades The sentence talks about how a final year project is important for
(a) detriment…………..founded completing an academic degree. The word fulfilling indicates that
(b) invent………demolished the first word should be positive. Therefore, the correct answer is
(c) folly……secured option (c).
(d) responsibility…. started Whining- to utter a high-pitched plaintive or distressed cry.
(e) crucial………completed Q16Ans. (b)
The answer is option (b), as the sentence talks about the
Q14. Japan’s newest world class University, the Okinawa Institute of contribution of women for White Revolution and the reason to
Science and Technology Graduate University, is in the most celebrate it in the month of March. Therefore, the corret answer is
________ corner within its territory, Okinawa island. option (b).
(a) foul (b) anticipated
(c) remote (d) huge
(e) repudiate Directions (17-21): Each question below has one or more blanks, each
Q15. The substantial final year project can be _________ and fulfilling blank indicating that something has been omitted. Choose the
for anyone completing an academic degree words for each blank that best fits the meaning of the sentence
(a) burdening (b) spoiling as a whole.
(c) nourishing (d) whining Q17. The creators of Huga include two architects and two engineers,
(e) attending along with several university professors, who were ________ by
their students' aspirations for _________ and location-independent
Q16. The achievements of women dairy farmers in _________ to India’s housing.
‘White Revolution’ are perhaps the greatest ________ for (a) disgusted, irritable
celebrating the Women’s History Month in March (b) renewed, foreseeable
(a) revealing…….reveal (b) contributing…….cause (c) Inspired, affordable
(c) abrupting……..precede (d) mesmerizing …….confirm (d) Modernized, perishable

Facebook Page- https://www.facebook.com/vishalpariharpage Youtube- https://www.youtube.com/vishalparihar


Follow
8 Vishal Sir Telegram Channel- https://t.me/englishbyvishalsirchannel Instagram- https://www.instagram.com/vishalthetrainer
By Vishal sir

SBI PO, SBI CLERK, IBPS PO, IBPS CLERK, RRB PO, RRB CLERK, NIACL,
- LIC RBI grade B, RBI ASSISTANT, & Other competitive Exams

(e) Modified, permeable


Directions (22 - 26): Pick out the most effective pair of words form the
Q18. Creative thinking requires our brains to make ________ between given pair of words to make sentence/sentences meaningfully
seemingly _______ ideas complete.
(a) Compliance, shifted (b) Accord, pardoned
Q22. Three-degree warming is a __________ for short-term disaster:
(c) distance, excused (d) Dissent, rearranged
(e) Connections, unrelated forests in the Arctic and the _______ of most coastal cities.
(a) prescription, loss (b) affection, force
Q19. As long as you meet a ________ of intelligence, then brilliant (c) construction, maturity (d) introduction, spin
creative work is well within your ________. (e) option, ground
(a) backbone, indifference (b) framework, thinking
(c) threshold, reach (d) class, thought Q23. The world’s major powers came within several signatures of
(e) Infrastructure, attention
_______ a binding, global framework to __________ carbon
Q20. Some experts believe the feathers of birds evolved from reptilian emissions.
scales. Through the forces of ________, scales gradually became (a) assuming, increase (b) endorsing, reduce
small feathers, which were used for warmth and ________ at first. (c) challenging, move (d) permitting, manufacture
(a) disproof, imagination (e) experiencing, make
(b) indication, pollution
(c) information, correction Q24. One of the most ________ things about the growth of the internet
(d) data, fiction is this: to a select group of technological ________, the surprise
(e) evolution, insulation
Q21. When you are dealing with a ________ problem, it is usually better wasn't how quickly it spread across the world
(a) intriguing, thinkers (b) attempting, outcome
to build upon what already ________.
(c) discriminating, factors (d) commissioning, prohibiters
(a) figure out, confidence (b) describe, distrust
(e) negative, favours
(c) complex, works (d) estimate, credit
(e) evaluate, credence
Q25. The _______ of the internet in the shape we know it today was
never a matter of _________.
Answers And Solutions (a) Innocence; time (b) arrival, inevitability
Q17Ans. (c) (c) matter, pace (d) Complain, hardwork
Foreseeable- that can be expected; that you can guess will happen (e) message, century
Perishable- to become destroyed or ruined
Permeable- having pores or openings that permit liquids or gases to Q26. Americans are in the _________ of a raging epidemic of mental
pass through
illness, at least as judged by the _________ in the numbers treated
Q18Ans. (e)
for it
pardoned- the excusing of an offense without exacting a penalty
(a) extension, plan
Dissent- to withhold assent or approval
(b) middle, experience
Accord- to grant or give especially as appropriate, due, or earned
(c) midst, increase
Q19Ans. (b)
(d) period, project
Infrastructure- the basic physical and organizational structures and
(e) profit, aim
facilities”
attention -notice taken of someone or something Answers And Solutions
Q20Ans. (e) 22Ans. (a)
fiction- something invented by the imagination or feigned ‘prescription, loss’ is the correct use.
21Ans. (c) 23Ans. (b)
credence- mental acceptance as true or real
Facebook Page- https://www.facebook.com/vishalpariharpage Youtube- https://www.youtube.com/vishalparihar
Follow
9 Vishal Sir Telegram Channel- https://t.me/englishbyvishalsirchannel Instagram- https://www.instagram.com/vishalthetrainer
By Vishal sir

SBI PO, SBI CLERK, IBPS PO, IBPS CLERK, RRB PO, RRB CLERK, NIACL,
- LIC RBI grade B, RBI ASSISTANT, & Other competitive Exams

‘endorsing, reduce’ is the correct use. (a) intended, profound (b) performed, replete
Endorsing- to inscribe (one's signature) on a check, bill, or note (c)forced, profuse (d)discovered, beneath
24Ans. (a) (e)produced, spirited
The correct use is ‘intriguing, thinkers’ as the sentence talks about
Q31. The stuff that catches fire around stove burners is known as natural
the growth of the internet and the blank should be filled with a gas, ________ to methane, a colorless, _______ gas that has the
positive word. Therefore, the correct answer is option (a). same chemical makeup no matter what the source.
25Ans. (b) (a)referring, odorless (b)interfacing, regardless
Here the most appropriate word for first blank is arrival as it talks (c)fundraising, meaningless (d)challenging, patternless
about the state of internet. Inevitability means incapable of being (e)scattering, horizonless
avoided. Hence the correct answer is option (b).
26Ans. (c)
Answers And Solutions
Midst- a period of time about the middle of a continuing act or
27Ans. (b)
condition
Corrigibilities- capable of being set right
Affectabilities- to influence
Directions (27-31): In each of the following paragraph, there are two 28Ans. (d
blank spaces. Below each paragraph, there are five options and Authoritative- possessing recognized or evident authority
each option consists of two words which can be filled up in the 29Ans. (a)
provided blanks respectively to make the paragraph Obscure- not discovered or known about; uncertain.
grammatically and coherently correct. Find the most 30Ans. (d)
appropriate set of words that fit into the blanks contextually. Profound- (of a state, quality, or emotion) very great or intense.
Replete- filled or well-supplied with something.
Profuse- (especially of something offered or discharged) very
Q27. For thousands of years, the struggle between rich and poor has
plentiful; abundant.
largely taken the form of _______ between creditors and debtors—
31Ans. (a)
of _________ about the rights and wrongs of interest payments. Odorless- something which has no smell.
(a) experiences, growths (b)conflicts, arguments
(c) corrigibility, alleges (d)affectabilities, technicals Directions (32-36): In the following questions a sentence is given with
(e) strangles, nations a blank and bold word. Against each question five alternatives
are provided; reflecting the word that should fill the blank and
Q28. The __________ in the near-empty stadium are amazing–you can conveying whether the word in bold is contextually correct or
hear every breath, every sneaker's squeak, the __________ pang of incorrect. Choose the best option as your answer.
the ball against very tight strings.. Q32. If the Indian economy is in the _____________, imagine the state
(a) animals, talkative (b) power, transformative of the economy in Kashmir, forcibly shutdown along with
(c) noises, productive (d) acoustics, authoritative communications and all other signs of life on August 5.
(e) efforts, expensive (a) Redamancy, incorrect (b) Doldrums, correct
(c) Rapacity, incorrect (d) Yelloch, correct
Q29. The realities of the men's professional-tennis tour bear about as `(e) None of the above
much ________ to the lush finals you see on TV as a
slaughterhouse does to a well-__________ cut of restaurant sirloin Q33. The _________ of bolted joints was morally sound and
(a) resemblance, presented (b) mainstay, obscured professionally correct
(c) guard, agreed (d) brace, rose (a)start, incorrect (b) match, correct
(e) reinforcements, constant (c) choice, incorrect (d) Feel, correct
(e) eliminate, correct
Q30. In the 1970s, geologists _________ crystalline natural gas—
methane hydrate, in the jargon- ___________ the seafloor.
Facebook Page- https://www.facebook.com/vishalpariharpage Youtube- https://www.youtube.com/vishalparihar
Follow
10 Vishal Sir Telegram Channel- https://t.me/englishbyvishalsirchannel Instagram- https://www.instagram.com/vishalthetrainer
By Vishal sir

SBI PO, SBI CLERK, IBPS PO, IBPS CLERK, RRB PO, RRB CLERK, NIACL,
- LIC RBI grade B, RBI ASSISTANT, & Other competitive Exams

Q34. The corroded device still bore faded __________ and it appeared `The most appropriate choice to be filled in the blank. The word
to have the guts of a clock. extrusion is incorrect here. The correct replacement of the word is
(a)Interventions, incorrect (b)prescriptions, incorrect ‘intrusion’ as it means to intervene in some process.
(c)intentions, correct (d)inscriptions, correct
(e)communications, correct
Directions (37-41): In each of the questions given below, a paragraph
Q35. The ________ of the web, building to link one resource to the
is given which has some blanks and those blanks have to be
next, was the beginning.
(a) Inclinations, incorrect (b) Embrace, incorrect filled with the same word out of the five words given below it.
(c) Abomination, incorrect (d) infrastructure, incorrect You have to choose that same word as your answer and fill up
(e) Gratitude, correct the blanks with that appropriate word.
Q37. __________, in navigation or surveying, the primary device for
Q36. Many health experts take the ________ further, contending that any
direction-finding on the surface of the Earth. __________ may
coming plague will come from human extrusion into the natural operate on magnetic or gyroscopic principles or by determining the
world. direction of the Sun or a star. The oldest and most familiar type of
(a) Humility, correct (b) notion, incorrect _________ is the magnetic __________, which is used in different
(c) Nuances, correct (d) Contrition, correct forms in aircraft, ships, and land vehicles and by surveyors.
(e) Dispute, correct (a) Manometer (b) Thermometer
(c) Compass (d) Barometer
Notion- a theory or belief held by a person or group
(e) Lux Meter
Nuances- a subtle difference in or shade of meaning, expression, or
sound Q38. _________ comes from the Greek word for "sweet." _________ is
Answers And Solutions a type of sugar you get from foods you eat, and your body uses
_________ for energy. As _________ travels through your
32Ans. bloodstream to your cells, its called blood _________ or blood
The correct choice is B, doldrums mean unsteady, other options are sugar.
erroneous. redamancy means reciprocated relationship, rapacity (a) Carbon Dioxide (b)Oxygen
means greedy, yelloch means to yell loudly (c)Water (d)Glucose
33Ans. (c)
(e)Sulphur
The most apt choice to be filled in the blank. Here word morally is
wrong the correct word should be technically. glucose for energy as Glucose is the component that gives energy
34Ans. (d) to the body.
Intervention- the act of interfering with the outcome or course
especially of a condition or process Q39. The ________ is a fretted musical instrument that typically has six
Inscriptions- the wording on a metal piece
35Ans.(d) strings. _________ is held flat against the player's body and played
The word building is incorrect, the correct replacement is ‘built’. by strumming or plucking the strings. The sound of the
Embrace means hold (someone) closely in one's arms, especially as __________ is projected either acoustically, by means of a resonant
a sign of affection. chamber on the instrument, or amplified by an electronic pickup
Gratitude means the quality of being thankful; readiness to show and an amplifier. The _________ is classified as a chordophone –
appreciation for and to return kindness. meaning the sound is produced by a vibrating string.
Inclination means a person's natural tendency or urge to act or feel (a)Xylophone (b)Guitar
in a particular way; a disposition. (c)Saxophone (d)trumpet
Abomination means a thing that causes disgust or loathing. (e)Trombone
36Ans. (b)

Facebook Page- https://www.facebook.com/vishalpariharpage Youtube- https://www.youtube.com/vishalparihar


Follow
11 Vishal Sir Telegram Channel- https://t.me/englishbyvishalsirchannel Instagram- https://www.instagram.com/vishalthetrainer
By Vishal sir

SBI PO, SBI CLERK, IBPS PO, IBPS CLERK, RRB PO, RRB CLERK, NIACL,
- LIC RBI grade B, RBI ASSISTANT, & Other competitive Exams

Q40. _________ is a racket sport that can be played individually against


a single opponent (singles) or between two teams of two players Directions (42-46): In following questions, a sentence is given with two
each (doubles). Each player uses a ____________ racket that is blanks. From the given alternatives, choose the most
strung with cord to strike a hollow rubber ball covered with felt appropriate set of words that fills the blank forming a
over or around a net and into the opponent's court. The object of grammatically correct and contextually meaningful sentence.
Q42. Some thinkers hold that mathematics is a kind of language--a
__________ is to maneuver the ball in such a way that the opponent
systematic _________ of signs, the criteria for the authority of
is not able to play a valid return..
which are internal ________, elegance, and depth.
(a) Table tennis (b) cricket (a) impetus, alliance
(c) Tennis (d) Badminton (b) seditious, imprecision
(e) Archery (c) rigorous, nuance
Q41. ___________ involves a complex set of economic, demographic, (d) contrivance, coherence
social, cultural, technological, and environmental processes that (e) radical, congruence
result in an increase in the proportion of the population of a territory
that lives in towns and cities. ___________ is the inevitable Q43. Gmail is very effective at filtering spam emails but _________
phenomenon of social and economic development. __________ seem to have figured out a new way to _________ the spam filters
significantly alters the hydrology of catchments, the transport of and send emails that land right in the user’s inbox.
sediment, nutrients, and pollutants and consequently has a (a) estrangement, perturbate (b) conformity, disdain
degrading impact on urban stream biota. (c) discord, affinity (d) spammers, bypass
(a) Immigration (b) Collaboration (e) perseverance, fervor
(c) Ingenuity (d) Urbanisation
(e) globalisation Q44. Schools need boost air ventilation to create a safe and _________
Answers And Solutions environment for students and increase the amount of _________ air
that circulates through their HVAC systems.
37Ans. (c) A.healthy, outdoor B.sound, ground
The appropriate answer as the choice. Manometer is used to C.health, surface D.perfect, creative
measure pressure. Thermometer is used to measure temperature. E.secure, circulation
Barometer is used to measure atmospheric pressure. Lux meter is
used to measure light intensity. Q45. The policy ________ child labor exists but lack of enforcement of
labor restrictions _________ child labor.
38Ans.(d)
(a) curbing, perpetuates (b) emancipating, untamed
Tthe only relevant choice among all the options. This is evident
(c) generating, tortures (d) reclaiming, assort
from the line your body uses
(e) ambulating, indignant
39Ans. (b)
As guitar is the only musical instrument that has strings from the Q46. He __________ the identities of new visitors and stopped any hawker
given options. who would _____ into the building during the quiet afternoons.
40Ans. (c) (a) verified, wander (b) checked, roaming
The relevant and correct option as tennis is the only sport out of (c) scrutiny, running (d) investigated, wanderer
given options that uses rackets and hollow rubber ball. Hence the (e) None of these
correct answer is option (c). Answers And Solutions
41Ans. (d)
Urbanisation means process in which there is an increase in the 42Ans. (d)
number of people living and working in a city or metropolitan area. ‘contrivance, coherence’ are best fit for the blank.
Hence the correct answer is option (d). Seditious means inciting or causing people to rebel against the
authority of a state or monarch.
Facebook Page- https://www.facebook.com/vishalpariharpage Youtube- https://www.youtube.com/vishalparihar
Follow
12 Vishal Sir Telegram Channel- https://t.me/englishbyvishalsirchannel Instagram- https://www.instagram.com/vishalthetrainer
By Vishal sir

SBI PO, SBI CLERK, IBPS PO, IBPS CLERK, RRB PO, RRB CLERK, NIACL,
- LIC RBI grade B, RBI ASSISTANT, & Other competitive Exams

Rigorous means extremely thorough and careful. ________ word! The original root meant hard, so your _______
Nuance means a subtle difference in or shade of meaning, friendship or your ________ interest in sports is solid enough to
expression, or sound. stand the test of time.
Contrivance means the use of skill to create or bring about (a) enduring (b) permitting
something, especially with a consequent effect of artificiality. (c) disapproving (d) concealing
Congruence means agreement or harmony; compatibility. (e) removing
43Ans. (d)
‘spammers, bypass’ is the correct set of words making the sentence Q48. _________ is the process whereby complex organic molecules
correct and meaningful. such as kerogens or long-chain hydrocarbons are broken down into
Estrangement means the fact of no longer being on friendly terms simpler molecules such as light hydrocarbons, by the breaking of
or part of a social group. carbon-carbon bonds in the precursors. The rate of __________ and
Perturbate means anxiety; mental uneasiness. the end products are strongly dependent on the temperature and
Discord means disagreement between people. presence of catalysts. __________ is the breakdown of a large
Perseverance means persistence in doing something despite alkane into smaller, more useful alkenes. Simply put, hydrocarbon
difficulty or delay in achieving success. __________ is the process of breaking a long chain of
Fervor means intense and passionate feeling. hydrocarbons into short ones.
44Ans. (c) (a) making (b) putting (c) wrapping
‘reared, prevent’, fits in the sentence correctly. After (d) smashing (e) Cracking
‘commercially’ and before ‘chicken’, ‘reared’ gives the exact .
sense. In the 2nd blank, only ‘prevent, gives the exact meaning, as Q49. _________ is generally a questioning attitude or doubt towards
before this blank ‘aggressive’ has been mentioned. grown, convert; one or more putative instances of knowledge. __________ is a
cultivated, reach; poked, make ----- these pairs have no connection topic of interest in philosophy, particularly epistemology.
with the sentence grammatically and contextually. Hence, these do ________ as an expression of questioning or doubt can be applied
to any topic, such as politics, religion, or pseudoscience. ________
not fit in the sentence
has also inspired a number of contemporary social movements.
45Ans. (a)
(a) polytheism (b) skepticism
‘curbing, perpetuates’ best suits the purpose.
Curbing means to limit or control something, especially something bad (c) initialism (d) capabilities
Assort means place in a group; classify. (e) negativism
Ambulating means walk; move about. Polytheism means the belief in or worship of more than one god
Indignant means feeling or showing anger or annoyance at what is Skepticism means an attitude of doubt or a disposition to
perceived as unfair treatment. incredulity either in general or toward a particular object
46Ans. (a)
‘verified, wander’, fits in the sentence correctly. Other options do Q50. ___________ are particles smaller than five millimeters deriving
not fit in the sentence. from the degradation of plastic objects present in the environment.
____________ can move from the environment to living
Directions (47-51): In each of the questions given below, a paragraph
organisms, including mammals. In total, 12 ___________
is given which has some blanks and those blanks have to be
filled with the same word out of five words given below it. You fragments (ranging from 5 to 10 μm in size), with spheric or
have to choose that same word as your answer and fill up the irregular shape were found in 4 placentas
blanks with that appropriate word. (a) plastics (b) microplastics
(c) rubber (d) benzene
Q47. Many people have an _________ love for ice cream, that is, they (e) polymers
have loved it for a long time and will continue to love it into the
future. ________ means long-lasting. _________ has roots that go Q51. _____________ is a constitutional right of every citizen that
back about 1,500 years to the Late Latin period. It is quite an prepares an individual to play their role as a sophisticated member
of society. The importance of __________ can be implied by
Facebook Page- https://www.facebook.com/vishalpariharpage Youtube- https://www.youtube.com/vishalparihar
Follow
13 Vishal Sir Telegram Channel- https://t.me/englishbyvishalsirchannel Instagram- https://www.instagram.com/vishalthetrainer
By Vishal sir

SBI PO, SBI CLERK, IBPS PO, IBPS CLERK, RRB PO, RRB CLERK, NIACL,
- LIC RBI grade B, RBI ASSISTANT, & Other competitive Exams

Rigorous means extremely thorough and careful. ________ word! The original root meant hard, so your _______
Nuance means a subtle difference in or shade of meaning, friendship or your ________ interest in sports is solid enough to
expression, or sound. stand the test of time.
Contrivance means the use of skill to create or bring about (a) enduring (b) permitting
something, especially with a consequent effect of artificiality. (c) disapproving (d) concealing
Congruence means agreement or harmony; compatibility. (e) removing
43Ans. (d)
‘spammers, bypass’ is the correct set of words making the sentence Q48. _________ is the process whereby complex organic molecules
correct and meaningful. such as kerogens or long-chain hydrocarbons are broken down into
Estrangement means the fact of no longer being on friendly terms simpler molecules such as light hydrocarbons, by the breaking of
or part of a social group. carbon-carbon bonds in the precursors. The rate of __________ and
Perturbate means anxiety; mental uneasiness. the end products are strongly dependent on the temperature and
Discord means disagreement between people. presence of catalysts. __________ is the breakdown of a large
Perseverance means persistence in doing something despite alkane into smaller, more useful alkenes. Simply put, hydrocarbon
difficulty or delay in achieving success. __________ is the process of breaking a long chain of
Fervor means intense and passionate feeling. hydrocarbons into short ones.
44Ans. (c) (a) making (b) putting (c) wrapping
‘reared, prevent’, fits in the sentence correctly. After (d) smashing (e) Cracking
‘commercially’ and before ‘chicken’, ‘reared’ gives the exact .
sense. In the 2nd blank, only ‘prevent, gives the exact meaning, as Q49. _________ is generally a questioning attitude or doubt towards
before this blank ‘aggressive’ has been mentioned. grown, convert; one or more putative instances of knowledge. __________ is a
cultivated, reach; poked, make ----- these pairs have no connection topic of interest in philosophy, particularly epistemology.
with the sentence grammatically and contextually. Hence, these do ________ as an expression of questioning or doubt can be applied
to any topic, such as politics, religion, or pseudoscience. ________
not fit in the sentence
has also inspired a number of contemporary social movements.
45Ans. (a)
(a) polytheism (b) skepticism
‘curbing, perpetuates’ best suits the purpose.
Curbing means to limit or control something, especially something bad (c) initialism (d) capabilities
Assort means place in a group; classify. (e) negativism
Ambulating means walk; move about. Polytheism means the belief in or worship of more than one god
Indignant means feeling or showing anger or annoyance at what is Skepticism means an attitude of doubt or a disposition to
perceived as unfair treatment. incredulity either in general or toward a particular object
46Ans. (a)
‘verified, wander’, fits in the sentence correctly. Other options do Q50. ___________ are particles smaller than five millimeters deriving
not fit in the sentence. from the degradation of plastic objects present in the environment.
____________ can move from the environment to living
Directions (47-51): In each of the questions given below, a paragraph
organisms, including mammals. In total, 12 ___________
is given which has some blanks and those blanks have to be
filled with the same word out of five words given below it. You fragments (ranging from 5 to 10 μm in size), with spheric or
have to choose that same word as your answer and fill up the irregular shape were found in 4 placentas
blanks with that appropriate word. (a) plastics (b) microplastics
(c) rubber (d) benzene
Q47. Many people have an _________ love for ice cream, that is, they (e) polymers
have loved it for a long time and will continue to love it into the
future. ________ means long-lasting. _________ has roots that go Q51. _____________ is a constitutional right of every citizen that
back about 1,500 years to the Late Latin period. It is quite an prepares an individual to play their role as a sophisticated member
of society. The importance of __________ can be implied by
Facebook Page- https://www.facebook.com/vishalpariharpage Youtube- https://www.youtube.com/vishalparihar
Follow
13 Vishal Sir Telegram Channel- https://t.me/englishbyvishalsirchannel Instagram- https://www.instagram.com/vishalthetrainer
By Vishal sir

SBI PO, SBI CLERK, IBPS PO, IBPS CLERK, RRB PO, RRB CLERK, NIACL,
- LIC RBI grade B, RBI ASSISTANT, & Other competitive Exams

habituating the lack of its existence. ________ prepares its wings d) CD e) None of these
to fly and explore the surroundings while being confident and
opportunistic. _________ builds individuals, educated individuals Q53. Clare wanted to keep a slightly ____________working-class look
build better societies, and better societies build great nations. to the house.
(a) unity (b) relationship A. sybaritic B. luxurious
(c) education (d) surmise
C. ascetic D. austere
(e) writing
a) AB b) BC
Answers And Solutions c) BD d) CD
e) None of these
47Ans. (a) Ascetic/austere would perfectly fit in; hence, (d) is the right answer.
The most suitable word that will fit in all the blanks of the
paragraph is “enduring”. Since one line in the paragraph mentions Q54. Phil and Dave were raring to get off to the mountains; they packed
that the word means lon lasting. Hence the correct option out of all
up their ski gear and climbed into the van with _________.
given options is ‘enduring’
48Ans. (e) A. alacrity B. apathy
The correct answer is option (e). As it makes the paragraph meaningful C. indifference D. ardour
49Ans. (b) a) AD b) BC
The most appropriate word that should fill all the blanks of the c) BD d) CD
sentence is “skepticism”. All the other words are either e) None of the above
grammatically incorrect or contextually. Hence, option (b) is the Q55. The attorney came up with several far-fetched arguments in a vain
most suitable answer choice. attempt to ________ his weak case.
50Ans. (b) A. buttress B. undermine
The correct answer is option (b), as the first sentence mentions the C. subvert D. underpin
degradation of plastic, also it is smaller then five millimeters. a) AB b) BC
Hence the correct answer is microplastics. c) BD d) AD
51Ans. (c) e) CD
The most appropriate word that should fill all the blanks of the
Q56. It was inevitable that David chose to go to Harvard; he had several
sentence is “education”. All the other words are either
grammatically incorrect or contextually. Hence, option (c) is the ________ reasons for doing so, including a full-tuition scholarship.
most suitable answer choice. A. obscure B. indistinct
Unity means come or bring together for a common purpose or action. C. cogent D. potent
a) AB b) BC
Surmise means suppose that something is true without having
c) BD d) CD
evidence to confirm it.
e) None of the above
Direction (52-56): In the given sentences, a blank is given indicating Answers And Solutions
that a word is missing. Among the four given options, a
combination of words would fit the blank thereby making it 52Ans. (b)
grammatically and contextually correct. Choose that option as Since the sentence talks about two responsibilities/aspects of the
your response. woman, the adjectives which would qualify 'activities'(noun) would
reflect the same. Multifarious and variegated would correctly fit in
Q52. Her life was as strange and adventurous as any of her novels, which the sentence (many or of various types). Homogeneous- of similar
are for the most part idealized versions of the _________ incidents type, which is opposite of what the writer is trying to convey.
of her life. Variety- noun: grammatical misfit since an adjective is required.
A. homogenous B. multifarious Hence, (b) is the right answer.
C. variegated D. variety 53Ans. (d)
a) AB b) BC c) BD

Facebook Page- https://www.facebook.com/vishalpariharpage Youtube- https://www.youtube.com/vishalparihar


Follow
14 Vishal Sir Telegram Channel- https://t.me/englishbyvishalsirchannel Instagram- https://www.instagram.com/vishalthetrainer
By Vishal sir

SBI PO, SBI CLERK, IBPS PO, IBPS CLERK, RRB PO, RRB CLERK, NIACL,
- LIC RBI grade B, RBI ASSISTANT, & Other competitive Exams

Sybaritic- fond of sensuous luxury or pleasure; self-indulgent. Q58. [I]. Prince Andrew touched the head with his hand; even the hair
Ascetic/austere- (of living conditions or a way of life) having no was __________, so profusely had the child perspired.
comforts or luxuries. As per the sentence's context, the adjective [II]. A little ahead of them walked a peasant guide, _________ to
the skin and wearing a grey peasant coat and a white knitted cap.
we are looking for should be on similar lines with 'strict' to maintain
(a) wet (b) caring
coherency. (c) modern (d) hurting
54Ans. (a) (e) noisy
Alacrity- cheerful promptness or willingness; eagerness; speed or
quickness. Ardour- great enthusiasm or passion. Apathy/ Q59. [I]. The entire business of the psychic tipster has faded rapidly
indifference- lack of interest, enthusiasm, concern. As per the without new news to _________the sharks
statement, since Phil and Dave were enthusiastic and eager (raring) [II]. Alex had the shed built so that she could ___________the
to go to the mountains, 'alacrity' and 'ardour' would fit in the blank; buffalo without going into the pen, but today she wanted to check
hence, option (a) is the right answer. on the cow.
55Ans. (d) (a) notice (b) play
As per the context, the attorney is trying to come up with arguments (c) feed (d) research
in order to support his weak case. Buttress- support physically; prop (e) reserve
up; support something or someone by supplying evidence;
undermine- lessen the effectiveness, power, or ability of, especially Q60. [I]. The thought disgusted me, but at the same time, made me
gradually or insidiously; subvert- undermine the power and __________.
authority of (an established system or institution); underpin- [II]. She is an absolute sweetheart and we’re really __________ to
support, justify, or form the basis for. Both 'buttress' and 'underpin' have her.
fit in the blank as per the context; hence, option (d) is the right (a) sick (b) quiet
answer. (c) rocked (d) serious
56Ans. (a) (e) glad
Obscure- not clearly expressed or easily understood; Indistinct- not Q61. [I]. Sandy Cadway laughed, __________ her wavy blonde hair up
clear or sharply defined; Cogent- clear, logical, and convincing; into a messy ponytail.
Potent- having great power, influence, or effect. As per the [II]. I unlocked the dorm door and stepped outside, __________ my
statement, since it was inevitable for David to choose to go to unruly red hair up.
Harvard, the second clause must be on similar lines (giving (a) spending (b) running
compelling/convincing reasons). 'Cogent' and 'potent' would fit in (c) opening (d) tying
the blank perfectly; hence, option (d) is the right answer. (e) meeting
The word ‘tie’ also fits appropriately with the blank of the second
Direction (57-61): There are two different sentences with a blank sentence.
space in each sentence. Choose the word from the given options In each of the questions given below, a part of the sentence is
which fits appropriately in both the blanks adding a proper and missing. Choose the part(s) that fits in the sentence contextually
logical meaning to the sentences. and grammatically.

Q57. [I] Queen Fembon, who was defeated, was forced to


Answers And Solutions
__________ herself before the conquering Romanians, who 57Ans. (b)
made her march in chains before the king, abase [verb] means ‘behave in a way that belittles or degrades
[II]. Thus she enters the dining room ready to __________ (someone)’;
himself because she disdains everyone else. A queen who is defeated would be forced to humiliate herself.
(a) add (b) abase Someone feels humiliated when forced to dine with those whom
(c) allow (d) believe she/he doesn’t like.
So, among the given words, the correct answer is the option (b).
(e) cash
58Ans. (a)

Facebook Page- https://www.facebook.com/vishalpariharpage Youtube- https://www.youtube.com/vishalparihar


Follow
15 Vishal Sir Telegram Channel- https://t.me/englishbyvishalsirchannel Instagram- https://www.instagram.com/vishalthetrainer
By Vishal sir

SBI PO, SBI CLERK, IBPS PO, IBPS CLERK, RRB PO, RRB CLERK, NIACL,
- LIC RBI grade B, RBI ASSISTANT, & Other competitive Exams

On what type of rock, a person could slip? On a wet rock, a person Q65. Human rights are __________________________ in the case of
could slip is an appropriate answer. refugees which exhibit a range of meanings - from the provision of
The word ‘wet’ also fits appropriately in the blank of the first protection to the fulfillment of basic necessities of livelihood.
sentence. 1. vulnerable to dubiety 2. prone to ambiguity
Among the given options, the word ‘wet’ appropriately fits the 3. open to criticisms
blank of both sentences. Hence, the option (a) is the correct answer. a) Only 1 b) Only 1 and 2
59Ans. (c) c) Only 2 and 3 d) Only 1 and 3
Among the given options, the word ‘feed’ correctly fits the context e) All 1, 2 and 3
of both the sentences.
Hence, the option (c) is the correct answer. Q66. __________________________ the jungles of Mesoamerica,
60Ans. (e) soaring pyramids, hieroglyphs, and long-hidden murals attest to the
The first sentence shows a contrast. In the initial phrase, it talks magnificence and mystery of the ancient Maya.
about being disgusted, so, in the next phrase, the word to be used 1. Nestled with 2. Huddled inside
should be happy or glad. 3. Nestled within
The word glad appropriately fits the blank of the second sentence. a) Only 1 b) Only 1 and 2
61Ans. (d) c) Only 2 and 3 d) All 1, 2 and 3
e) None of the above
What could one do with hair up so to have a messy ponytail? She/he
could tie her/his hairs. Answers And Solutions
Direction (62-66): In the given sentences, a blank is given indicating 62Ans. (b)
that a word is missing. Among the four given options, a The option B provides a grammatically and contextually correct
combination of words would fit the blank thereby making it filler. The word 'all-out' means 'using all resources' and makes
grammatically and contextually correct. Choose that option as sense in the sentence with the word 'hunt', which means 'search
your response determinedly'. The phrases/ words in A and C are grammatically
Q62. Most people may assume that scientists and drug companies have inappropriate. (b) is the right answer.
always been on _____________ for a cure for Aids. 63Ans. (a)
A. with great effort B. an all-out hunt All the three options A, B and C provide grammatically and
C. extremes contextually correct fillers. While the first two phrases have a
a) Only A b) Only B similar meaning, and are apt in the blank, 'can be easily said' too
c) Only A and B d) Only A and C fits easily to provide a meaningful sentence. (a) is the right answer
e) None of the above 64Ans. (a)
Q63. Behind closed doors, it _____________ that politics in China are Only A provides a grammatically and contextually correct filler.
no less vicious than in the Rome of Julius Caesar. The phrasal verb 'show up' means 'get exposed (generally of a
A. is reasonable to suppose B. is fair to assume fault)' and is correct in the blank with 'may not'. B and C are
C. can be reasonably said meaningless phrases in the context of the sentence. (a) is the right
(a) All A, B and C (b) Only A and B answer.
(c) Only B and C (d) Only A and C 65Ans. (b)
(e) None of the above 'prone' -- 'likely to suffer from, vulnerable'; 'ambiguity -- open to
. individual interpretations'; 'dubiety' -- 'doubt'. Both (1) and (2)
Q64. These chemicals boost harvests, but they also have unpredictable contain phrases that can fit in the blank. However, 'open to
side effects that _____________ for years. criticisms' in (3) does not make contextual sense. Hence (b) is
A. may not show up
correct.
B. is unlikely to be given
66Ans. (c)
C. is noticed
a) Only A b) Only A and B 'Nestled' -- 'settled comfortably within (something)'; 'huddled' --
c) Only B and C d) Only A and C 'crowded together'. (1) is incorrect since 'Nestled with' is incorrect
e) None of the above syntax -- 'Nestled in' or 'Nestled within' are correct. 'Huddled inside'
is also correct. The correct option is (c).

Facebook Page- https://www.facebook.com/vishalpariharpage Youtube- https://www.youtube.com/vishalparihar


Follow
16 Vishal Sir Telegram Channel- https://t.me/englishbyvishalsirchannel Instagram- https://www.instagram.com/vishalthetrainer
By Vishal sir

SBI PO, SBI CLERK, IBPS PO, IBPS CLERK, RRB PO, RRB CLERK, NIACL,
- LIC RBI grade B, RBI ASSISTANT, & Other competitive Exams

Stick to means When something is attached to another by some


Directions (67-70): In each of the following questions, a sentence is form of adhesive.
given with a blank. There are five phrases given below each sentence, Stick out means to extend something outward
one out of which can be used in the blank to form a meaningful Stick with means to continue to use or do something.
sentence. Choose the most appropriate phrase among the five options Stick in means insert casually
that makes the sentence contextually meaningful.
69Ans. (c)
Q.67. The house they had offered us __________ to be a tiny apartment. The most suitable phrase that provides the sentence a meaningful
1. turned out 2. turned in context is ‘take it out on’. ‘take it out on’ means to treat someone
3. turned over 4. turned into badly because you are upset or angry, even if they have done
5. turned off nothing wrong. All the other phrases fail to make the sentence
grammatically correct or contextually meaningful. Hence, option
Q.68. She taught her children to _______ for themselves at school. (c) is the most suitable answer choice.
(a) stick to (b) stick out Take back means to reclaim possession of something.
(c) stick with (d) stick up for Take apart means to disconnect or separate the parts of an object.
(e) stick in Take in means include or encompass something.
Take out means escort someone to a social event or place of
Q.69. I know you've had a bad day, but you don't have to_________me! entertainment.
(a) take back (b) take apart 70Ans. (b)
(c) take it out on (d) take in The sentence is in the form of a fact and it is in simple present tense
(e) take out form. A base form of verb is needed in the blank. This eliminates
all the options except option B as all of them are in past or past
Q70. What you believe about the world shapes how you perceive and
participle form. Option B is the correct answer.
_____ with the world.
In the given sentences, a blank is given indicating that something
(a) Mingled (b) Interact
is missing. From the given four options, choose the one that
(c) Mixed (d) Analysed
provides the correct word to be filled in the blank, thereby making
(e) None of the above
the sentence grammatically and contextually correct. In case none
Answers And Solutions of the options fit, choose "none of the above" as the answer.

67Ans. (a) 7: 1 Directions (71-93): In each of the following questions, a sentence is


The most suitable phrase that provides the sentence a meaningful given with a blank/blanks. There are five options given below each
context is ‘turned out’. ‘turned out’ means “prove to be the case”. sentence, one out of which can be used in the blank/blanks to form a
All the other phrases fail to make the sentence grammatically meaningful sentence. Choose the most appropriate phrase/word/words
correct or contextually meaningful. Hence, option (a) is the most among the five options that makes the sentence contextually
suitable answer choice. meaningful.
Turned in means go to bed in the evening.
Turned over means (of an engine) start or continue to run properly. Q71. A living wage should make ______ that people earn enough to
Turned into means become (a particular kind of thing or person); make ends meet.
be transformed into. (a) Definitely (b) Define
Turned off means to repulse, disgust, or discourage. (c) Sure (d) Certainly
68Ans. (d)
(e) None of the above
The most suitable phrase that provides the sentence a meaningful
context is ‘stick up for’. ‘stick up for’ means support or defend. All
the other phrases fail to make the sentence grammatically correct Q72. One study found that while most _____ entrepreneurs call
or contextually meaningful. Hence, option (d) is the most suitable themselves optimists, optimistic entrepreneurs earn 30% less than
answer choice. pessimistic ones on average.
Facebook Page- https://www.facebook.com/vishalpariharpage Youtube- https://www.youtube.com/vishalparihar
Follow
17 Vishal Sir Telegram Channel- https://t.me/englishbyvishalsirchannel Instagram- https://www.instagram.com/vishalthetrainer
By Vishal sir

SBI PO, SBI CLERK, IBPS PO, IBPS CLERK, RRB PO, RRB CLERK, NIACL,
- LIC RBI grade B, RBI ASSISTANT, & Other competitive Exams

(a) Conduct There can only be a lack of enthusiasm about the subjects.
(b) Conducted
(c) Successful Q80. For something no one has ever seen, black holes seem ______
familiar and science-fiction authors and film directors love using
(d) Concern
their ______ to warp space and time as a plot device.
(e) None of the above (a) Awfully, ability (b) Awful, able
(c) Apparent, damage (d) Consistently, relevant
Q73. Whether you're trying to switch roles at work or launch a new idea, (e) None of the above
waiting for _____ can be your greatest enemy.
a) Circulating b) Circulation Q81. Tropical rainforests play ______ roles in carbon sequestration and
c) Misleading d) Perfection are hot spots for biodiversity, but tropical forests are being replaced
e) None of the above by rubber plantations, causing widespread ______ of a crash in
biodiversity.
Q74. It might seem strange that _____ struggling airlines are the most (a) Important, concern (b) significant, concerned
willing to spend more on safety. (c) Significantly, concerned (d) Significantly, concern
a) Monetary b) Financially (e) None of the above
c) Economical d) Supply
e) None of the above Q82. Global assessments of food security under ______ conditions face
not only the challenge of data scarcity, but also limited process
Q75. The humanitarian reasons for _____________ this provision
knowledge with respect to low-input ______ systems and their
should be sufficient in themselves. management.
(a) Downfall (b) Economy (a) Future, agricultural
(c) Safeguarding (d) Liability (b) Emotion, underscored
(e) Betraying (c) Emotional, underscore
Q76. As traffic grew, the _______________ wisdom was that roads
(d) Disputing, cultural
should be widened to make room.
(e) None of the above
(a)Free-spirit (b)Provided
(c)Conventional (d)Groundbreaking Q83. The crusade against corruption should continue but the ________
(e)Freakish should not kill businesses.
(a) Communities (b) Productivity
Q77. It is difficult to expect warmth of feeling when people are (c) Connectivity (d) Zealousness
excessively _____________ in behavior. (e) None of the above
(a) Exhausting (b) Polite
(c) Supposing (d) Commencing Q84.. _____________ a campaign to bring military history out of its
(e) Passing cloistered preserves and make it more accessible, is India's oldest
and most venerable think-tank, the United Service Institution.
Q78. A fall in interest rates is a strong ____________ in the present (a) Spearheading (b) Connecting
economic climate. (c) Contradicting (d) Resigning
(a) Inquiry (b) Unity (e) None of the above
(c) Probability (d) Security
(e) Restriction Q85. Mental illness is now the ________ cause of disability in children.
(a)Deactivating, correct (b) Coinciding, incorrect
Q79. University teachers generally have admitted a lack of _________ (c) Leading, correct (d) Professing, correct
about their subjects. (e) Professing , incorrect
(a) Neutrality (b) Opposition
(c) Memory (d) Enthusiasm
(e) Frequency
Facebook Page- https://www.facebook.com/vishalpariharpage Youtube- https://www.youtube.com/vishalparihar
Follow
18 Vishal Sir Telegram Channel- https://t.me/englishbyvishalsirchannel Instagram- https://www.instagram.com/vishalthetrainer
( NEW)

JOIN OUR TELEGRAM CHANNEL


BANKING VIDYARTHI
BANKING VIDYARTHI FOR EXAM
PD
EXAM NEWS
THE HINDU NEWSPAPAR DAILY
AT 4-5 AM
SEARCH FOR CLICK HERE
( TELEGRAM CHANNEL )
BANKING VIDYARTHI4
( YOUTUBE CHANNEL )
BANKING VIDYARTHI
( NEW)

JOIN OUR TELEGRAM CHANNEL


BANKING VIDYARTHI
BANKING VIDYARTHI FOR EXAM
PD
EXAM NEWS
THE HINDU NEWSPAPAR DAILY
AT 4-5 AM
SEARCH FOR CLICK HERE
( TELEGRAM CHANNEL )
BANKING VIDYARTHI4
( YOUTUBE CHANNEL )
BANKING VIDYARTHI
By Vishal sir

SBI PO, SBI CLERK, IBPS PO, IBPS CLERK, RRB PO, RRB CLERK, NIACL,
- LIC RBI grade B, RBI ASSISTANT, & Other competitive Exams

expected from them. Exhausting, supposing and passing are Deactivate means make (something) inactive by disconnecting or
irrelevant and cannot be used here. Commencing means beginning. destroying it.
78Ans. (c) In the question below, a sentence is given with two blanks, that
The appropriate choice to be filled in blank is option (c). Probability indicate that some parts are missing. Identify the correct pair of
means possibility. Here the statement means a fall in interest rates is words that fit in the sentence to make it grammatically and
a strong likelihood in the present economic climate.
contextually correct.
Inquiry is a question which you ask in order to get some
85Ans. (c)
information.
Unity means integration, togetherness. Profess means claim, pretend.
Security means the state of being free from danger or threat. Deactivate means make (something) inactive by disconnecting or
Restriction means limitation. destroying it.
79Ans. (d) In the question below, a sentence is given with two blanks, that
The appropriate choice to be filled in blank is option (d). indicate that some parts are missing. Identify the correct pair of
Enthusiasm is the most suitable word which means intense and words that fit in the sentence to make it grammatically and
eager enjoyment, interest, or approval. contextually correct
Neutrality means absence of decided views, expression, or strong
feeling. 86Ans. (c)
Frequency means the rate at which something occurs over a 1st and 2nd blank- both the blanks require verb in its present form
particular period of time or in a given sample. (since the statement is in present tense). Confess, qualify- contextual
There can only be a lack of enthusiasm about the subjects. misfit (we don't confess or qualify our ideas at work). Option (b):
Produce- contextual misfit. As per the above-mentioned criteria,
80Ans. (a) option (c) fits in the blanks grammatically and contextually; hence,
An adverb is needed in the first blank to modify the adjective (c) is the right answer.
'familiar' and a noun is needed in the 2nd blank to represent what 87Ans. (d)
the directors love. For the first blank - Awful(adjective) and 1st blank- it requires a verb in plural form since the subject is plural
Apparent(adjective); this eliminates option B and C. For the second (emissions); 'need' would be appropriate here; needful- adjective;
blank - relevant(adjective); option D is eliminated. Option A is the needed- past tense form; needs- singular verb form; 2nd blank- it
correct answer as it fits grammatically and contextually. requires an adjective which would qualify the noun 'level';
81Ans. (a) affliction, adversity, disaster- noun; disastrous- adjective; hence,
An adjective is needed in the first blank to modify the noun 'roles' (d) is the right answer.
87Ans. (a)
and a noun is needed in the second blank to be modified by the
adjective 'widespread'. For the 1st blank - 'significantly' is an In the given sentence, only austerity and substantial make it
adverb hence option C and D are eliminated. For the 2nd blank - grammatically as well as contextually correct. Austerity means a
'concerned' is an adjective or a verb in past participle form; option difficult economic situation caused by a government reducing the
B is eliminated. Option A is the correct answer as it fits amount of money it spends while substantial means concerning the
grammatically and contextually. essentials of something.
82Ans. (d)
88Ans. (a)
83Ans. (d)
1st blank- an adverb is required here, which would qualify the
The correct option is D, zealousness which means intense passion
adjective 'dire'; 'progressively'- adverb, progression- noun,
for something, Other options are contextually wrong. This
progress- verb, progressive- adjective. 2nd blank- it requires a
statement talks about putting a lot of effort to end corruption.
83Ans. (a) noun; only 'hope' fits in here. Hence, (a) is the right answer.
89Ans. (a)
The correct option is A, spearheading means leading something,
Other options are erroneous, contradicting means opposite of In the given sentence, only austerity and substantial make it
something. grammatically as well as contextually correct. Austerity means a
84Ans. (c) difficult economic situation caused by a government reducing the
Profess means claim, pretend.

Facebook Page- https://www.facebook.com/vishalpariharpage Youtube- https://www.youtube.com/vishalparihar


Follow
20 Vishal Sir Telegram Channel- https://t.me/englishbyvishalsirchannel Instagram- https://www.instagram.com/vishalthetrainer
By Vishal sir

SBI PO, SBI CLERK, IBPS PO, IBPS CLERK, RRB PO, RRB CLERK, NIACL,
- LIC RBI grade B, RBI ASSISTANT, & Other competitive Exams

amount of money it spends while substantial means concerning the (a) notice (b) play
essentials of something. (c) feed (d) research
(e) reserve
90Ans. (e) Q95. [I]. Hugh Jackman is ________ that The Greatest Showman
In the first blank, due to the presence of 'an' prior to the word, brought a smile to a fan's face.
'plenitude' and 'myriad' are ruled out. 'opulence', meaning wealth, [II]. Gray dropped his hands and looked up with a wild ______ cry.
is unsuitable too. In the second, from 'ferocious' and 'rapacious' we (a) sick (b) quiet
choose the latter, since the context relates to plunder. (c) rocked (d) serious
rapacious means plundering (e) glad
plenitude - plenty, larcenous - thievish
opulence - wealth, voracious - greedy (from hunger) Q96. [I]. You can cover them with __________ towels or sheets and fan
affluence - abundance, ferocious - violent them until it is possible to get them to t a hospital or doctor.
[II]. She followed, slipping on the __________ rock.
myriad - innumerable, aggressive – hostile (a) wet (b) caring
91Ans. (c) (c) modern (d) hurting
As is evident from the sentence, the first blank can be filled up (e) noisy
either by 'majority' (meaning superiority, most in number) or by Among the given options, the word ‘wet’ appropriately fits the
'primary', and the second blank will be filled up either by 'avoid', blank of both sentences. Hence, the option (a) is the correct answer.
'avert' or 'prevent', which are synonyms. Taking both into account,
we get (c) as the answer. 'Maximum' wouldn't be an appropriate
adjective for the noun 'owners'. 'Only' would have been correct had
Answers And Solutions
there been the noun 'owner'(singular)
void -- empty 94Ans. (c)
avert – avoid Among the given options, the word ‘feed’ correctly fits the context
92Ans. (d) of both the sentences.
95Ans. (e)
In the first blank, 'aptitude', 'capability' and 'potential' fit. 'Ineptness'
The first sentence shows a contrast. In the initial phrase, it talks
and 'incompetence' mean the opposite. In the second blank, about being overwhelmed or glad, so, in the next phrase, the word
'insurgent', 'subversive' and 'riotous' fit. Hence, combining the two, to be used should be happy or glad.
the correct answer will be (d). The word glad appropriately fits the blank of the second sentence.
96Ans. (a)
93Ans. (b)
On what type of rock, a person could slip? On a wet rock, a person
Look for key words that create a link in the sentence. "Was not could slip is an appropriate answer.
ruled..but" is an important set of words. Because of the word The word ‘wet’ also fits appropriately in the blank of the first
"citizenry," it would seem that 'societies', Option 2, would be better sentence.
than 'populations' and certainly better than 'cities', Option 3.
Options 4 and 5, also fit in the first blank. Directions (97-98): In the question given below, a sentence is given
The second blank indicates some sort of working with others or with one blank. Below each sentence, FOUR words are given
'participation. out of which two can fit into the sentence. Five options are
Convalesced - recuperate or recover. provided with various combinations of these words. You have
As per the analogy, only option 2 fits perfectly in the blank. to choose the combination with the correct set of words which
Direction (94-96): There are two different sentences with a blank can coherently fit into the given sentence.
space in each sentence. Choose the word from the given options
which fits appropriately in both the blanks adding a proper and Q97. The proofs are anecdotal or based on _____reasoning such as
logical meaning to the sentences.
thinking that a correlation proves a causal connection.
(I) Virtuous (II) Scrupulous
Q94. [I]. Remember to ___________ the dog and don’t burn the house
(III) Flawed (IV) Fallacious
down. (a) (I) – (II) (b) (I) – (IV)
[II]. Did you remember to __________ the cat? (c) (II) – (III) (d) (III) - (IV)

Facebook Page- https://www.facebook.com/vishalpariharpage Youtube- https://www.youtube.com/vishalparihar


Follow
21 Vishal Sir Telegram Channel- https://t.me/englishbyvishalsirchannel Instagram- https://www.instagram.com/vishalthetrainer
By Vishal sir

SBI PO, SBI CLERK, IBPS PO, IBPS CLERK, RRB PO, RRB CLERK, NIACL,
- LIC RBI grade B, RBI ASSISTANT, & Other competitive Exams

(e) All of these Q101. They ______________the necessity for the Federal Republic of
Yugoslavia to accept the international community 's demands
Q98. Studies show that European subscriptions to high-speed Internet (a) underscored (b)hackneyed
service are on track to _____________________ those in the (c)neutralized (d)spotted
United States. (e)repeated
(I) Surpass (II) Authorize
(III) exceed (IV) Sanction hackneyed means (of a phrase or idea) having been overused;
(a) (I) – (II) (b) (I) – (IV) unoriginal and trite.
(c) (I) – (III) (d) (III) - (IV) Word ‘underscored’ completes the sentence in a meaningful way.
(e) None of these
irrelevant in context of the sentence. Hence, option (c) is the most Q102. Extreme weather conditions continued in several parts of the
suitable answer choice. country on Sunday, _____ 41 more lives in the worst-hit Andhra
Surpass means exceed; be greater than. Pradesh.
Sanction means give official permission or approval for (an action). (a), killing (b), claiming
Answers And Solutions (c), mitigating
(e), absolving
(d), warranted

97Ans. (d) Q103. On the world tobacco day, PM Modi ______ the harms associated
The most appropriate combination of words is “Flawed and with tobacco consumption and pressed for reducing the menace.
Fallacious”. All the other words are irrelevant in context of the (a) Repeated (b) accentuated
sentence. Hence, option (d) is the most suitable answer choice.
Virtuous means having or showing high moral standards. (c) told (d) highlighted
Scrupulous means (of a person or process) careful, thorough, and (e) asked
extremely attentive to details.
Flawed means having or characterized by a fundamental weakness
or imperfection. Answers And Solutions
Fallacious means based on a mistaken belief.
98Ans. (c ) 99Ans.(d)
The most appropriate combination of words is “surpass and Stagnating - ceasing to develop; becoming inactive or dull.
exceed”. All the other words are Word ‘shifting’ completes the sentence in a meaningful way.
100Ans.(a)
Directions (99-103): Each question below has one blank, indicating prevention - the action of stopping something from happening or
that something has been omitted. Choose the word from the arising;
options that best fits the meaning of the sentence as a whole. detention - the action of detaining someone or the state of being
detained in official custody.
Q99. Just because momentum is______ away coal in china does not Aberration means a departure from what is normal, usual, or
mean that the country is no longer part of the global coal boom. expected, typically an unwelcome one.
(a) altering (b) changing Word ‘prevention’ completes the sentence in a meaningful way.
(c) stagnating (d) shifting 101Ans.(a)
(e) staying underscored- emphasize
102Ans.(b)
Q100. The national disaster management authority should come up with claiming- formally request or demand.
national guidelines for the _____ of such deaths for immediate Warranted= justify or necessitate (a course of action).
implementation by the states. Mitigating means having the effect of making something bad less
(a)prevention (b)aid severe, serious, or painful.
(c)detention (d)aberration Absolving means declare (someone) free from guilt, obligation, or
(e)obstruction punishment.
Word ‘claiming’ completes the sentence in a meaningful way.

Facebook Page- https://www.facebook.com/vishalpariharpage Youtube- https://www.youtube.com/vishalparihar


Follow
22 Vishal Sir Telegram Channel- https://t.me/englishbyvishalsirchannel Instagram- https://www.instagram.com/vishalthetrainer
By Vishal sir

SBI PO, SBI CLERK, IBPS PO, IBPS CLERK, RRB PO, RRB CLERK, NIACL,
- LIC RBI grade B, RBI ASSISTANT, & Other competitive Exams

Hence, option (b) becomes the most suitable answer choice. (e) shut-borrowing
103Ans.(d)
highlighted- draw special attention to; menace- a person or thing
that is likely to cause harm; a threat or danger.
Answers And Solutions
Accentuated means make more noticeable or prominent.
Word ‘highlighted’ completes the sentence in a meaningful way. 104Ans.(a)
Hence, option (d) becomes the most suitable answer choice. ‘relentlessly-solved’ is the correct use where relentlessly means
continuing in a severe or extreme way. Doggedly means
Directions (Q104-108): Each question below has two blanks, each stubbornly, persistently means continuously
blank indicating that something has been omitted. Choose the 105Ans.(b)
set of words for each blank which best fits the meaning of the ‘groovy-exhorts’is the correct use where groovy means fashionable
sentence as a whole. and exciting and exhorts means strongly encourage or urge
Q104. After ……… pursuing it for nearly twenty-one months, the city (someone) to do something.
106Ans.(a)
police ………. the case of the triple murder of a professor and his
‘rushed-condition’ is the correct use where rushed means to move
family. or to do something very quickly.
(a) relentlessly-solved (b) doggedly-cracked 107Ans.(a) crashed-ago’is the correct use
(c) constantly-handed (d) persistently-found where crashed means collide violently with an obstacle or another
(e) tirelessly-generated vehicle.
108Ans.(a)
Q105. It’s not just the ………… reggae and dance hall music that sets ‘swindled-availing’ is the correct use where ‘swindled’ means
their party experience apart, the group has a Disc Jockey who obtain (money) fraudulently.
………… the crowd to dance, interacts with them and also raps to
the music. Directions (109-113): Below are two sentences with one blank each.
(a) regular-extols (b) groovy-exhorts Choose the correct word which would fill both blanks while
(c) morose-compels (d) upbeat-beats keeping the sentences grammatically correct and contextually
(e) remarkable-makes meaningful.

Q106. The injured security guard was first ………… to a small private Q109. I. The company walked off with the coveted ______ at the SC
hospital from where he was shifted to a bigger civil hospital as his Magazine awards 2013 Europe
………… deteriorated. II. She is set to star in six films in the next few years and given her
(a) rushed-condition (b) hurried-status role choices in the past, her future films will likely receive awards
(c) taken-life (d) admitted-form and ___________
(e) hastened-health (1) Accolades (2) Chasm
(3) Encomiums (4) Eulogies
Q107. The French government has said it will conduct a fourth search for (a) (3) and (4) (b) Only (2)
the flight recorders of an Air France jetliner that …………. into the (c) (1), (3) and (4) (d) (1) and (2)
Atlantic Ocean while flying from Rio de Janeiro to Paris 18 months. (e) None of (1), (2), (3) and (4)
(a) crashed-ago (b) plunged-later The word which satisfy the contextual requirement, of the above
given sentences, is praises.
(c) dove-before (d) fell-prior
So, (1), (3) and (4) are the correct answer and (c) is the correct
(e) drowned-back answer.
Q108. Four officials of a rural cooperative bank ……….. the institution of Q110. I. We waited until after our parents went to sleep to have our
Rs. 1.56 crore, by ……….. of loans through fake Self Help Groups _____________ meeting about their surprise party.
(SHGs). II. After hitting the lottery, the private family hoped to keep their
(a) swindled-availing (b) arrested-seeking _________________ winnings to themselves.
(c) cheated-taking (d) duped-recommending (1) Secret (2) Clandestine

Facebook Page- https://www.facebook.com/vishalpariharpage Youtube- https://www.youtube.com/vishalparihar


Follow
23 Vishal Sir Telegram Channel- https://t.me/englishbyvishalsirchannel Instagram- https://www.instagram.com/vishalthetrainer
By Vishal sir

SBI PO, SBI CLERK, IBPS PO, IBPS CLERK, RRB PO, RRB CLERK, NIACL,
- LIC RBI grade B, RBI ASSISTANT, & Other competitive Exams

(3) Furtive (4) Surreptitious 111Ans.(b)


(a) Only (1) (b) Both (1) and (2) Dichotomy [noun] and chasm [noun] are synonyms which mean ‘a
(c) Only (3) (d) All of (1), (2), (3) and (4) division or contrast between two things that are or are represented
(e) None of (1), (2), (3) and (4) as being opposed or entirely different’.
Paean [noun] and Testimonial [noun] are synonyms which mean ‘a
Q111. I. His ___________ of heaven and hell became an excellent essay song of praise or triumph’ or ‘a creative work expressing
on the contrast between paradise and eternal suffering enthusiastic praise’.
II. When Rajat visited U.S. he recognized a huge ___________ Among the alternatives, (1) and (4) have a meaning closer to the
between the western culture and his own way of life. meaning of ‘difference’ and appropriately fit the blanks.
(1) Dichotomy (2) Paean 112Ans.(a)
(3) Testimonial (4) Chasm Panegyric [noun] means ‘a public speech or published text in praise
(a) Both (1) and (3) (b) Both (1) and (4) of someone or something’.
(c) Only (2) (d) All of (1), (2), (3) and (4) Gulf [noun] and Split [noun] are synonyms which mean ‘a
(e) None of (1), (2), (3) and (4)
profound difference between people, viewpoints, feelings, etc.’.
Q112. I. The children began to ____________ over a toy, and the Tussle [noun] means a vigorous struggle or scuffle, typically to
preschool teacher had to step in to break up the fight. obtain or achieve something.
II. There ensued one of those friendly ___________, not quite The likely word to fill the blank of the statement I is ‘struggle’
fights, that kids find such fun. The presence of the phrase ‘not quite fights,’ after the blank
(1) Panegyric (2) Gulf provides a hint for the word which would fill the blank. The word
(3) Split (4) Tussle should have a meaning closer to ‘fight’ but not exactly fight.
(a) Only (4) (b) Both (2) and (3) Among the options, the word ‘tussle’ has a meaning which satisfy
(c) Only (1) (d) All of (1), (2), (3) and (4) the above-mentioned requirement of the word which would fill the
(e) None of (1), (2), (3) and (4)
blank of the statement II.
Also, the meaning of the word ‘tussle’ is also closer to the meaning
Q113. I. She sobbed and ___________ them to let her out until Carmen of the word ‘struggle’.
felt like her heart was being brutally wrenched from her body. So, the alternative (4) is the correct alternative which would fill the
II. You promised to return in a day's time, and I ____________ you blank, and option (a) is the correct answer.
not to send me back to the catacombs
113Ans.(e)
(1) Spawned (2) Flayed
(3) Berated (4) Belaboured Spawn [verb] means (of a fish, frog, mollusk, crustacean, etc.)
(a) Only (1) (b) Only (4) release or deposit eggs;
(c) Both (2) and (3) (d) All of (1), (2), (3) and (4) Flayed (flay), berated (berate), belabored (belabor) are synonyms
(e) None of (1), (2), (3) and (4) which mean to criticize severely and brutally;
None of the given alternatives fit the context of the sentences I and II.
The word which could have satisfied the context of the sentences I
Answers And Solutions and II are ‘requested’, ‘begged’, ‘cadged’ (means to ask for or
obtain (something to which one is not strictly entitled);
109Ans.(c)
Chasm [noun] means a deep fissure in the earth’s surface or a Directions (114-118): Each question below has one blank, indicating
profound difference between people, viewpoints, feelings etc. that something has been omitted. Choose the word from the
Accolades, Encomiums and Eulogies are plural nouns and options that best fits the meaning of the sentence as a whole.
synonyms which mean a speech or a piece of writing that praises
someone or something highly.
110Ans.(d) Q114. The floors were pale stone, the walls something called latte, the
Clandestine, Furtive, Surreptitious are synonyms of the word furniture in light woods and cream, __________by teal and lemon
‘secret’ which mean ‘keeping something secret; kept secret or done pillows and tasteful throws.
secretly, especially because illicit. (a) Repeated (b) accentuated
So, all of the given alternatives (1), (2), (3) and (4) could fill the (c) told (d) highlighted
blank of above given sentences. (e) asked
Facebook Page- https://www.facebook.com/vishalpariharpage Youtube- https://www.youtube.com/vishalparihar
Follow
24 Vishal Sir Telegram Channel- https://t.me/englishbyvishalsirchannel Instagram- https://www.instagram.com/vishalthetrainer
By Vishal sir

SBI PO, SBI CLERK, IBPS PO, IBPS CLERK, RRB PO, RRB CLERK, NIACL,
- LIC RBI grade B, RBI ASSISTANT, & Other competitive Exams

Word ‘underscored’ completes the sentence in a meaningful way.


Q115. It's the oldest bond there is, old enough to keep the mating bond 118Ans.(b)
from ________ when she wins her deal. claiming- formally request or demand.
(a) altering (b) changing Warranted= justify or necessitate (a course of action).
(c) stagnating (d) shifting Mitigating means having the effect of making something bad less
(e) staying severe, serious, or painful.
Word ‘shifting’ completes the sentence in a meaningful way. Absolving means declare (someone) free from guilt, obligation, or
punishment.
Q116. You must put appropriate systems and reporting arrangements in Word ‘claiming’ completes the sentence in a meaningful way.
place to allow effective _______, control and recovery of arrears.
(a)prevention (b)aid, Directions (119-123): Below are two sentences with one blank each.
(c)detention (d)aberration Choose the correct word which would fill both blanks while
(e)obstruction keeping the sentences grammatically correct and contextually
meaningful.
Q117. The dramatic elements of the show are _________ by both the
comedy and singing in the show. Q119. I. Little was said of our ____________ activities of the past year as
(a) underscored (b)hackneyed we simply enjoyed each other's company.
(c) neutralized (d)spotted II. Very severe punishments were suggested for
(e) repeated the ____________lover, but the emperor rewarded the devotion of
the pair by consenting to their marriage.
Q118. Gabe approached and stopped behind her, ______________the (1) Secret (2) Clandestine
alcohol with one hand.
(3) Furtive (4) Surreptitious
(a)killing (b)claiming
(c)mitigating (d)warranted (a) Only (1) (b) Both (1) and (2)
(e)absolving (c) Only 2 (d) All of (1), (2), (3) and (4)
(e) None of (1), (2), (3) and (4)
Answers And Solutions Q120. I. My account of the historical problem within the religion-
secular ___________can be found at the end of this paper.
114Ans.(d) II. There was a clear _____________ between what Ashley said
highlighted- draw special attention to; menace- a person or thing she would do and what she actually did.
that is likely to cause harm; a threat or danger. (1) Dichotomy (2) Paean
Accentuated means make more noticeable or prominent. (3) Testimonial (4) Chasm
Word ‘highlighted’ completes the sentence in a meaningful way. (a) Both (1) and (3) (b) Both (1) and (4)
115Ans.(d) (c) Only (2) (d) All of (1), (2), (3) and (4)
Stagnating - ceasing to develop; becoming inactive or dull. (e) None of (1), (2), (3) and (4)
116Ans.(a)
prevention - the action of stopping something from happening or Q121. I. First John White took out David Evans in a
arising; titanic ___________ that brought the full house regularly to its feet.
detention - the action of detaining someone or the state of being II. He won a __________ with Everitt outside the box, dragged the
detained in official custody. ball back to Davidson who made aright hash of the shot.
Aberration means a departure from what is normal, usual, or (1) Panegyric (2) Gulf
expected, typically an unwelcome one. (3) Split (4) Tussle
Word ‘prevention’ completes the sentence in a meaningful way. (a) Only (4) (b) Both (2) and (3)
117Ans.(a) (c) Only (1) (d) All of (1), (2), (3) and (4)
underscored- emphasize (e) None of (1), (2), (3) and (4)
hackneyed means (of a phrase or idea) having been overused;
unoriginal and trite.
Facebook Page- https://www.facebook.com/vishalpariharpage Youtube- https://www.youtube.com/vishalparihar
Follow
25 Vishal Sir Telegram Channel- https://t.me/englishbyvishalsirchannel Instagram- https://www.instagram.com/vishalthetrainer
By Vishal sir

SBI PO, SBI CLERK, IBPS PO, IBPS CLERK, RRB PO, RRB CLERK, NIACL,
- LIC RBI grade B, RBI ASSISTANT, & Other competitive Exams

Q122. I. I __________ a lift back to the station in Roger’s car, which was The presence of the phrase ‘not quite fights,’ after the blank
kind of him. provides a hint for the word which would fill the blank. The word
II. This is a preamble to confessing that, like Jackie, I should have a meaning closer to ‘fight’ but not exactly fight.
____________ a few puffs of a fat Cuban on Christmas Day. Among the options, the word ‘tussle’ has a meaning which satisfy
(1) Spawned (2) Flayed the above-mentioned requirement of the word which would fill the
(3) Berated (4) Belaboured blank of the statement II.
(a) Only (1) (b) Only (4) Also, the meaning of the word ‘tussle’ is also closer to the meaning
(c) Both (2) and (3) (d) All of (1), (2), (3) and (4) of the word ‘struggle’.
(e) None of (1), (2), (3) and (4) So, the alternative (4) is the correct alternative which would fill the
Q123. I. The attempt of Des Murs was praiseworthy, but in effect it has blank, and option (a) is the correct answer.
utterly failed, notwithstanding the ___________ passed upon it by 122Ans.(e)
friendly critics Spawn [verb] means (of a fish, frog, mollusk, crustacean, etc.)
II. His character and career have been made the subject release or deposit eggs;
of ____________ much beyond their merits. Flayed (flay), berated (berate), belabored (belabor) are synonyms
(1) Accolades (2) Chasm which mean to criticize severely and brutally;
(3) Encomiums (4) Eulogies None of the given alternatives fit the context of the sentences I and II.
(a) (3) and (4) (b) Only (2) The word which could have satisfied the context of the sentences I
(c) (1), (3) and (4) (d) (1) and (2) and II are ‘requested’, ‘begged’, ‘cadged’ (means to ask for or
(e) None of (1), (2), (3) and (4) obtain (something to which one is not strictly entitled);
123Ans.(c)
Answers And Solutions Chasm [noun] means a deep fissure in the earth’s surface or a
profound difference between people, viewpoints, feelings etc.
Accolades, Encomiums and Eulogies are plural nouns and
119Ans.(d) synonyms which mean a speech or a piece of writing that praises
Clandestine, Furtive, Surreptitious are synonyms of the word someone or something highly.
‘secret’ which mean ‘keeping something secret; kept secret or done The word which satisfy the contextual requirement, of the above
secretly, especially because illicit. given sentences, is praises.
So, all of the given alternatives (1), (2), (3) and (4) could fill the
So, (1), (3) and (4) are the correct answer and (c) is the correct answer.
blank of above given sentences.
120Ans.(b)
Dichotomy [noun] and chasm [noun] are synonyms which mean ‘a Directions (124-125): In each of the following sentences there are two
division or contrast between two things that are or are represented blank spaces. Below each sentence there are five pairs of words
as being opposed or entirely different’. denoted by numbers (a), (b), (c),(d) and (e). Find out which pair
Paean [noun] and Testimonial [noun] are synonyms which mean ‘a of words can be filled up in the blanks in the sentence in the
song of praise or triumph’ or ‘a creative work expressing same sequence to make the sentence grammatically correct and
enthusiastic praise’.
meaningfully complete.
Among the alternatives, (1) and (4) have a meaning closer to the
meaning of ‘difference’ and appropriately fit the blanks.
Q124. He was able to __________ Greek at the __________ of eight.
121Ans.(a) (a) climb, height (b) read, age
Panegyric [noun] means ‘a public speech or published text in praise (c) watch, time (d) see, distance
of someone or something’. (e) invite, table
Gulf [noun] and Split [noun] are synonyms which mean ‘a
profound difference between people, viewpoints, feelings, etc.’.
Q125. ___________ three eggs into a __________ and mix them together.
Tussle [noun] means a vigorous struggle or scuffle, typically to
(a) store, box (b) cook, box
obtain or achieve something.
(c) crack, bowl (d) throw, dustbin
The likely word to fill the blank of the statement I is ‘struggle’
(e) cook, dustbin

Facebook Page- https://www.facebook.com/vishalpariharpage Youtube- https://www.youtube.com/vishalparihar


Follow
26 Vishal Sir Telegram Channel- https://t.me/englishbyvishalsirchannel Instagram- https://www.instagram.com/vishalthetrainer
By Vishal sir

SBI PO, SBI CLERK, IBPS PO, IBPS CLERK, RRB PO, RRB CLERK, NIACL,
- LIC RBI grade B, RBI ASSISTANT, & Other competitive Exams

be cooked into a dustbin or cooked into a box. So, the options (b) Q128. (I) While some public servants are found to be corrupt, it is
and (e) are ruled out. Now, among the options (a), (c) and (d), impossible to ....................... the integrity of the majority of law
selection of the correct word will be made through analyzing the enforcement officers by offering them a bribe.
phrase ‘mix them together’. Storing (or throwing) three eggs into a (II) To examine all the causes which may .....................
box (or a dustbin) and mixing them together don’t make sense. So,
the options (a) and (d) are also ruled out. emendations would mean writing a treatise upon human frailty.
(a) accoutrement (b) scabbard
Answers And Solutions (c) parley (d) vitiate
(e) exodus
124Ans.(b)
Q129. (I) A .................. had broken out between two groups of students,
What is ‘Greek’? Is it something on which someone could climb?
who wanted to approach the VC with their demands.
Is it something which could be watched? Is it something which
(II) The battle of Shiloh was a savage ....................... between two
could be seen? Is it which could be invited? Or Is it something half-disciplined hosts, contested with a fury rare even in this war.
which could be read? (a) bungling (b) superfluity
Greek has two meanings: one denotes the name of a Language, and (c) accusation (d) nibble
the other denotes the name for the citizens of the Greece. (e) scuffle
So, among the options, the two contenders for the correct answer accusation- a charge or claim that someone has done something
are the options (b) and (e). Now, if ‘Greek’ is a common name illegal or wrong.
nibble- take small bites out of.
attributed to a singular person, then it should be preceded by an
scuffle- a short, confused fight or struggle at close quarters.
article which is missing. The word ‘Greek’ can’t be plural because Here, SCUFFLE fits perfectly in both the blanks. Hence, option
the word itself is a singular. Hence, the option (e) is also ruled out.
(e) is the correct choice.
125Ans.(c)
Three eggs can be cracked into a bowl, thrown into a dustbin, stored
into a box, but they can’t Q130. (I) John's go-to move at rallies is to rail against the injustices
..................... by the Mexican establishment.
Direction (126-130): There are two different sentences with a blank (II) Bennett’s attorney portrayed her as a victim of a scheme
space in each sentence. Choose the word from the given options ............................ by another official from her company.
which fits appropriately in both the blanks adding a proper (a) entangled
and logical meaning to the sentences.
(b) slackened
Q126. (I) If the power of the mind could be harnessed, investigators (c) perpetrated
speculated that it might be possible to .................... a foreign leader (d) meddled
or even an enemy army.
(II) As the second-in-command under a nearly .......................
Answers And Solutions
President, many observers had expected her to take center stage.
(a) expropriate (b) incapacitate
126Ans.(b)
(c) quash (d) scramble
(e) anticipate expropriate- take (property) from its owner for public use or
Q127. (I) Sheen has since been released, but still has go in front of the benefit.
judge in February to face the two felonies and one incapacitate- prevent from functioning in a normal way.
………………….. for which he is currently being charged. quash- reject as invalid, especially by legal procedure.
(II) Search, MSN search and Google all have systems in place so scramble- make (something) jumbled or muddled.
that such ................... can be reported. anticipate- regard as probable; expect or predict. Though both the
(a) deportation (b) deprivation
(c) misdemeanor (d) placate option (c) and (d) might be considered as filler for (I) sentence but
(e) stall grammatically they won’t make a sense for the (II) blank hence can
be invalidated as answer choice.

Facebook Page- https://www.facebook.com/vishalpariharpage Youtube- https://www.youtube.com/vishalparihar


Follow
27 Vishal Sir Telegram Channel- https://t.me/englishbyvishalsirchannel Instagram- https://www.instagram.com/vishalthetrainer
By Vishal sir

SBI PO, SBI CLERK, IBPS PO, IBPS CLERK, RRB PO, RRB CLERK, NIACL,
- LIC RBI grade B, RBI ASSISTANT, & Other competitive Exams

127Ans.(c) Ans.(d) In the given sentence, only ‘worshippers, abruptly,


deportation- the action of deporting a foreigner from a country. grabbing’ make it grammatically as well as contextually correct.
deprivation- the damaging lack of material benefits considered to Option A): is incorrect because spark does not fit here.
be basic necessities in a society. Option B): is incorrect as ‘vanish’ does not make any sense here.
misdemeanor- a minor wrongdoing.
placate- make (someone) less angry or hostile. Option C): is incorrect as douse means pour a liquid over; drench
Here, MISDEMEANOUR fits perfectly in both the blanks. which does not fit here.
Hence, option (c) is the correct choice. Option D): is the correct alternative among the following as
128Ans.(d) ‘worshippers, abruptly, grabbing’ fit here both grammatically and
accoutrement- an additional item of dress or equipment. contextually
scabbard- a sheath for the blade of a sword or dagger, typically Douse means to stop a fire from burning by pouring liquid over it
made of leather or metal. Abruptly means in a sudden and unexpected way
parley- a conference between opposing sides in a dispute,
especially a discussion of terms for an armistice. Q132. For the poorest countries, _______actually leads to greater FDI
vitiate- spoil or impair the quality or efficiency of. inflows, but from a smaller pool of countries. Because of this,
Exodus- a mass departure of people authors argue that fears of technological advancement
Here, VITIATE fits perfectly in the both the blanks. Hence, option _______labour may be _______, at least for the time being.
(d) is the correct choice. A.reinforce, enhance, overstated
129Ans.(e) B.automation, displacing, impair
bungling- the action or fact of carrying out a task clumsily or C.reinforce, enhance, impair
incompetently. D.automation, displacing, overstated
superfluity- an unnecessarily or excessively large amount or E.None of these
number of something.
(e) whirled Q133. The Bhopal gas tragedy is the result of ________ negligence on the
130Ans.(c) part of a company that should have _______________ to the
entangled- cause to become twisted together with or caught in. standards it _____________ it followed elsewhere in the world.
slackened- reduce or decrease in speed or intensity. (a) at, question, in
perpetrated- carry out or commit (a harmful, illegal, or immoral (b) deliberate, adhered, claimed
action). (c) with, issue, on
meddled- interfere in something that is not one's concern (d) from, conundrum, amongst
Here, PERPETRATED fits perfectly in both the blanks. Hence, (e) in, palate, among
option (c) is the correct choice Answers And Solutions
Directions (131-133): In each of the following sentence there are three 131 Ans.(d)
blank spaces. Below ach sentence there are five options and In the given sentence, only ‘worshippers, abruptly, grabbing’ make
each option consists of three words which can be filled up in the it grammatically as well as contextually correct.
blanks in the sentence to make the sentence grammatically Option A): is incorrect because spark does not fit here.
correct. Option B): is incorrect as ‘vanish’ does not make any sense here.
Option C): is incorrect as douse means pour a liquid over; drench
Q131. It was the pontiff’s first walkabout among _______after he angrily which does not fit here.
slapped the hand of a woman in response to her _______ Option D): is the correct alternative among the following as
_______his arm on New Year’s Eve. ‘worshippers, abruptly, grabbing’ fit here both grammatically and
A.worshippers, abruptly, spark contextually
B.vanish, douse, spark Douse means to stop a fire from burning by pouring liquid over it
C.vanish, douse, grabbing Abruptly means in a sudden and unexpected way
D.worshippers, abruptly, grabbing 132Ans.(d)
E.None of these In the given sentence, only ‘automation, displacing, overstated’
make it grammatically as well as contextually correct.

Facebook Page- https://www.facebook.com/vishalpariharpage Youtube- https://www.youtube.com/vishalparihar


Follow
28 Vishal Sir Telegram Channel- https://t.me/englishbyvishalsirchannel Instagram- https://www.instagram.com/vishalthetrainer
By Vishal sir

SBI PO, SBI CLERK, IBPS PO, IBPS CLERK, RRB PO, RRB CLERK, NIACL,
- LIC RBI grade B, RBI ASSISTANT, & Other competitive Exams

Option A): is incorrect because reinforce does not fit here. (c) Both (2) and (3) (d) All of (1), (2), (3) and (4)
Option B): is incorrect as ‘impair’ does not make any sense here. (e) None of (1), (2), (3) and (4)
Option C): is incorrect as ‘enhance’ does not fit here.
Option D): is the correct alternative among the following as Answers And Solutions
‘automation, displacing, overstated’ fit here both grammatically
and contextually.
134Ans.(b)
Reinforce means to make something stronger
Dichotomy [noun] and chasm [noun] are synonyms which mean ‘a
Automation means the use of machines instead of people to do
division or contrast between two things that are or are represented
work
as being opposed or entirely different’.
Overstated means to say something in a way that makes it seem
Paean [noun] and Testimonial [noun] are synonyms which mean ‘a
more important than it really is
133Ans.(b) song of praise or triumph’ or ‘a creative work expressing
enthusiastic praise’.
Sol. Deliberate- to think about or discuss issues and decisions
Among the alternatives, (1) and (4) have a meaning closer to the
carefully
meaning of ‘difference’ and appropriately fit the blanks.
Adhered- to stick firmly to something
Hence, option (b) is the correct answer.
Conundrum- a confusing and difficult problem or question.
Directions (134-136): Below are two sentences with one blank each.
135Ans.(a)
Choose the correct word which would fill both blanks while
Panegyric [noun] means ‘a public speech or published text in praise
keeping the sentences grammatically correct and contextually
of someone or something’.
meaningful. Gulf [noun] and Split [noun] are synonyms which mean ‘a
profound difference between people, viewpoints, feelings, etc.’.
Q134. I. It is not the fortune of many more brilliant statesmen to earn Tussle [noun] means a vigorous struggle or scuffle, typically to
this _________ to character. obtain or achieve something.
II. Do you have any contact people for references The likely word to fill the blank of the statement I is ‘struggle’
or ___________ purposes? The presence of the phrase ‘not quite fights,’ after the blank
(1) Dichotomy (2) Paean provides a hint for the word which would fill the blank. The word
(3) Testimonial (4) Chasm should have a meaning closer to ‘fight’ but not exactly fight.
(a) Both (1) and (3) (b) Both (1) and (4) Among the options, the word ‘tussle’ has a meaning which satisfy
(c) Only (2) (d) All of (1), (2), (3) and (4) the above-mentioned requirement of the word which would fill the
(e) None of (1), (2), (3) and (4) blank of the statement II.
Q135. I. But Gladstone risked the reproach, accepted the office and had a Also, the meaning of the word ‘tussle’ is also closer to the meaning
sharp _______ for his seat. of the word ‘struggle’.
II. First John White took out David Evans in a So, the alternative (4) is the correct alternative which would fill the
titanic _________ that brought the full house regularly to its feet. blank, and option (a) is the correct answer.
(1) Panegyric (2) Gulf
(3) Split (4) Tussle 136Ans.(e)
(a) Only (4) (b) Both (2) and (3) Spawn [verb] means (of a fish, frog, mollusk, crustacean, etc.)
(c) Only (1) (d) All of (1), (2), (3) and (4) release or deposit eggs;
(e) None of (1), (2), (3) and (4) Flayed (flay), berated (berate), belabored (belabor) are synonyms
which mean to criticize severely and brutally;
Q136. I. Next day the Emperor arrived in Moscow, and several of the None of the given alternatives fit the context of the sentences I and
Rostovs' domestic serfs ___________ permission to go to have a II. Hence, option (e) is the correct answer.
look at him The word which could have satisfied the context of the sentences I
II. They applauded all his tricks and at the end of the and II are ‘requested’, ‘begged’, ‘cadged’ (means to ask for or
performance ___________ him earnestly not to go away again and obtain (something to which one is not strictly entitled);
leave them.
(1) Spawned (2) Flayed Directions (137-141): In following questions, a sentence is given with
(3) Berated (4) Belaboured two blanks. From the given alternatives, choose the most
(a) Only (1) (b) Only (4)
Facebook Page- https://www.facebook.com/vishalpariharpage Youtube- https://www.youtube.com/vishalparihar
Follow
29 Vishal Sir Telegram Channel- https://t.me/englishbyvishalsirchannel Instagram- https://www.instagram.com/vishalthetrainer
By Vishal sir

SBI PO, SBI CLERK, IBPS PO, IBPS CLERK, RRB PO, RRB CLERK, NIACL,
- LIC RBI grade B, RBI ASSISTANT, & Other competitive Exams

Q86. If we are not thoughtful about how we _______ our ideas at work, (a) aptitude, oppressive (b) ineptness, insurgent
we _____ not being heard or, worse, missing out on the credit we (c) capability, turbulence (d) potential, subversive
are due. (e) incompetence, riotous
(a) qualify, jeopardise Q93. Egypt was ruled not by kings and emperors as was common among
(b) project, produce other ........... at the time, but by a citizenry, which .............. fully in
(c) present, risk the affairs of the city.
(d) confess, endanger (a) populations, cooperated (b) societies, participated
(e) None of the above (c) cities, engaged (d) civilizations, supplemented
(e) monarchies, convalesced
Q87. Global carbon emissions ______ to be reduced to net zero by 2050
to have a good chance of holding global average temperature rises
to no more than 1.5oC, a level that would be __________, but not
Answers And Solutions
catastrophic for human civilization.
(a) needful, affliction (b) needs, adversity 71Ans. (e)
(c) needed, disaster (d) need, disastrous An adjective is needed in the blank. This eliminates option
(e) None of the above A(adverb), B(verb) and D(adverb). Option C is the correct answer.
72Ans. (c)
Q88. In a crisis that has become _________ dire over the past decade, An adjective is needed in the blank to qualify the noun
digital solutions - many with artificial intelligence (AI) at their core 'entrepreneurs'. This eliminates option A(verb/noun), B(verb in the
- offer ______ for reversing the decline in our mental wellness. past form) and D(verb/noun). Option C is the correct answer.
a) progressively, hope b) progression, hopeful 73Ans. (d)
c) progress, hopingly d) progressive, hope A noun is needed in the blank. This eliminates option A (gerund or
e) None of the above present participle) and C(adjective). 'Circulation' means the public
1st blank- an adverb is required here, which would qualify the availability or knowledge of something; it is contextually incorrect.
adjective 'dire'; 'progressively'- adverb, progression- noun, Option D is the correct answer. The sentence says that if you wait
progress- verb, progressive- adjective. 2nd blank- it requires a for perfection before switching roles or launching a new idea, it can
noun; only 'hope' fits in here. Hence, (a) is the right answer. be your greatest enemy.
Q89. Evidence from __________policies a decade ago suggests a 74Ans. (b)
_____________ multiplier effect, as spending cuts lead to falling An adverb is needed in the blank to qualify the adjective
incomes, leading to further spending cuts.
'struggling'. This eliminates option A(adjective), C(adjective) and
(a) austerity, substantial (b) extravagance, generous
(c) acerbity, slight (d) indulgence, consequential D(verb/noun). Option B is the correct answer.
(d) obduracy, trivial 75Ans.(c)
Q90. Politics is funded because political power provides an __________ The most appropriate choice. Safeguarding means protecting. The
of opportunity for ________________ aggrandizement of wealth, provision can only be protected or safeguarded.
in a way no regular business does. Downfall means a loss of power, prosperity, or status.
a) Plenitude, larcenous b) Opulence, voracious Liability means the state of being legally responsible for
c) Affluence, ferocious d) Myriad, aggressive
something.
e) Abundance, rapacious
Betraying means deceiving.
Q91. With the government being the __________ owners of the public 76Ans. (c)
sector banks, the finance In order to manage the traffic an approach of widening the roads is
Q92. The __________ for the ideas spread through print media to used that is the conventional or regular wisdom is used. It cannot
challenge official power structures led some political and religious be free-spirit or provided or groundbreaking or freakish.
authorities to actively suppress publications that they deemed 77Ans. (b)
__________. The appropriate word to be used here is ‘polite’. If people are
excessively polite in their behavior, feeling of warmth cannot be
Facebook Page- https://www.facebook.com/vishalpariharpage Youtube- https://www.youtube.com/vishalparihar
Follow
19 Vishal Sir Telegram Channel- https://t.me/englishbyvishalsirchannel Instagram- https://www.instagram.com/vishalthetrainer
By Vishal sir

SBI PO, SBI CLERK, IBPS PO, IBPS CLERK, RRB PO, RRB CLERK, NIACL,
- LIC RBI grade B, RBI ASSISTANT, & Other competitive Exams

Q86. If we are not thoughtful about how we _______ our ideas at work, (a) aptitude, oppressive (b) ineptness, insurgent
we _____ not being heard or, worse, missing out on the credit we (c) capability, turbulence (d) potential, subversive
are due. (e) incompetence, riotous
(a) qualify, jeopardise Q93. Egypt was ruled not by kings and emperors as was common among
(b) project, produce other ........... at the time, but by a citizenry, which .............. fully in
(c) present, risk the affairs of the city.
(d) confess, endanger (a) populations, cooperated (b) societies, participated
(e) None of the above (c) cities, engaged (d) civilizations, supplemented
(e) monarchies, convalesced
Q87. Global carbon emissions ______ to be reduced to net zero by 2050
to have a good chance of holding global average temperature rises
to no more than 1.5oC, a level that would be __________, but not
Answers And Solutions
catastrophic for human civilization.
(a) needful, affliction (b) needs, adversity 71Ans. (e)
(c) needed, disaster (d) need, disastrous An adjective is needed in the blank. This eliminates option
(e) None of the above A(adverb), B(verb) and D(adverb). Option C is the correct answer.
72Ans. (c)
Q88. In a crisis that has become _________ dire over the past decade, An adjective is needed in the blank to qualify the noun
digital solutions - many with artificial intelligence (AI) at their core 'entrepreneurs'. This eliminates option A(verb/noun), B(verb in the
- offer ______ for reversing the decline in our mental wellness. past form) and D(verb/noun). Option C is the correct answer.
a) progressively, hope b) progression, hopeful 73Ans. (d)
c) progress, hopingly d) progressive, hope A noun is needed in the blank. This eliminates option A (gerund or
e) None of the above present participle) and C(adjective). 'Circulation' means the public
1st blank- an adverb is required here, which would qualify the availability or knowledge of something; it is contextually incorrect.
adjective 'dire'; 'progressively'- adverb, progression- noun, Option D is the correct answer. The sentence says that if you wait
progress- verb, progressive- adjective. 2nd blank- it requires a for perfection before switching roles or launching a new idea, it can
noun; only 'hope' fits in here. Hence, (a) is the right answer. be your greatest enemy.
Q89. Evidence from __________policies a decade ago suggests a 74Ans. (b)
_____________ multiplier effect, as spending cuts lead to falling An adverb is needed in the blank to qualify the adjective
incomes, leading to further spending cuts.
'struggling'. This eliminates option A(adjective), C(adjective) and
(a) austerity, substantial (b) extravagance, generous
(c) acerbity, slight (d) indulgence, consequential D(verb/noun). Option B is the correct answer.
(d) obduracy, trivial 75Ans.(c)
Q90. Politics is funded because political power provides an __________ The most appropriate choice. Safeguarding means protecting. The
of opportunity for ________________ aggrandizement of wealth, provision can only be protected or safeguarded.
in a way no regular business does. Downfall means a loss of power, prosperity, or status.
a) Plenitude, larcenous b) Opulence, voracious Liability means the state of being legally responsible for
c) Affluence, ferocious d) Myriad, aggressive
something.
e) Abundance, rapacious
Betraying means deceiving.
Q91. With the government being the __________ owners of the public 76Ans. (c)
sector banks, the finance In order to manage the traffic an approach of widening the roads is
Q92. The __________ for the ideas spread through print media to used that is the conventional or regular wisdom is used. It cannot
challenge official power structures led some political and religious be free-spirit or provided or groundbreaking or freakish.
authorities to actively suppress publications that they deemed 77Ans. (b)
__________. The appropriate word to be used here is ‘polite’. If people are
excessively polite in their behavior, feeling of warmth cannot be
Facebook Page- https://www.facebook.com/vishalpariharpage Youtube- https://www.youtube.com/vishalparihar
Follow
19 Vishal Sir Telegram Channel- https://t.me/englishbyvishalsirchannel Instagram- https://www.instagram.com/vishalthetrainer
By Vishal sir

SBI PO, SBI CLERK, IBPS PO, IBPS CLERK, RRB PO, RRB CLERK, NIACL,
- LIC RBI grade B, RBI ASSISTANT, & Other competitive Exams

appropriate set of words that fills the blank forming a 139Ans.(c)


grammatically correct and contextually meaningful sentence. ‘reared, prevent’, fits in the sentence correctly. After
‘commercially’ and before ‘chicken’, ‘reared’ gives the exact
Q137. Gmail is very effective at filtering spam emails but _________ sense. In the 2nd blank, only ‘prevent, gives the exact meaning, as
seem to have figured out a new way to _________ the spam filters before this blank ‘aggressive’ has been mentioned. grown, convert;
and send emails that land right in the user’s inbox. cultivated, reach; poked, make ----- these pairs have no connection
(a) estrangement, perturbate (b) conformity, disdain with the sentence grammatically and contextually. Hence, these do
(c) discord, affinity (d) spammers, bypass not fit in the sentence.
(e) perseverance, fervor 140Ans.(e)
‘indigenous, harvesting’ is the correct set of words which are
Q138. Small-scale and community-based studies have found estimated delivering a meaningful sentence.
_________ of 12.6 million children engaged in ___________ Deleterious means causing harm or damage
occupations. Floriferous means (of a plant) producing many flowers.
(a) inbred, confute (b) prevalence, hazardous Bouldering means climbing on large boulders, either for practice or
(c) mitigate, obtund (d) prolong, amplifying as a sport in its own right.
(e) verbose, blunt 141Ans.(d)
Q139. Commercially ________chicken can be unusually aggressive, and ‘exploit, causes’, fits in the sentence correctly. In the 1st blank, we
are often kept in darkened sheds to _______them pecking at each need V1(‘exploit’) [RULE: PLURAL SUBJECT(‘links’) + V1]; all
other. the other options are V5.
A.grown, convert B.cultivated, reach After the 2nd blank, we have “changing colours”, so, ‘causes, fits
C.reared, prevent D.poked, make here. decelerates, persist; accelerates, insist; prevents, makes -- ---
E.None of these these pairs have no connection with the sentence grammatically and
contextually. Hence, these do not fit in the sentence.
Q140. The ________ people follow forestry management practices such Direction (142-146): There are two different sentences with a blank
as assisted forest regeneration, selective _________ and reforesting space in each sentence. Choose the word from the given options
and assisted growth of trees within existing forests which fits appropriately in both the blanks adding a proper and
(a) deleterious, endearing (b) expeditious, galloping logical meaning to the sentences.
(c) floriferous, presuming (d) prosperous, bouldering
(e) indigenous, harvesting Q142. [I] Your argument is being ignored because it is basically nothing
but ___________
Q141. Security inks ________ the same principle that ________the vivid [II] Let us disprove this scientific ________________ by
and constantly changing colours of a film of oil on water. conducting a laboratory experiment.
A.decelerates, persist B.accelerates, insist (a) conjectures (b) feint (c) homage
C.prevents, makes D.exploit, causes (d) case (e) evidence
E.None of these Homage means special honour or respect shown publicly.
Answers And Solutions Q143. [I] Because he was not ready to be tied down, Mark felt a great deal
of _______________ about caring for his sick mother on a full-
137Ans.(d) time basis
‘spammers, bypass’ is the correct set of words making the sentence [II] Even though I am a great swimmer, I have a great deal of
correct and meaningful. _____________ about swimming because my uncle drowned.
138Ans.(b) (a) illumination (b) glee
‘prevalence, hazardous are the words that can be filled in the blanks (c) triumph (d) significance
giving the sentence its correct meaning. (e) ambivalence
Inbred means existing from birth.
Confute means prove (a person or an assertion or accusation) to be Q144. [I] Before I make an offer on the diamond, I need to know its
wrong
Obtund means dull the sensitivity of; blunt; deaden _________ worth aside from the band which contains it.
Verbose means using or expressed in more words than are needed.
Blunt means weaken or reduce the force of (something).

Facebook Page- https://www.facebook.com/vishalpariharpage Youtube- https://www.youtube.com/vishalparihar


Follow
30 Vishal Sir Telegram Channel- https://t.me/englishbyvishalsirchannel Instagram- https://www.instagram.com/vishalthetrainer
By Vishal sir

SBI PO, SBI CLERK, IBPS PO, IBPS CLERK, RRB PO, RRB CLERK, NIACL,
- LIC RBI grade B, RBI ASSISTANT, & Other competitive Exams

[II] Although going on a cruise sounds like a great vacation plan, irrelevant and do not fit in the context of the sentence here. Hence
the option does have the _____________ drawback of being quite option (b) is the correct choice.
costly for a family of eight Illusory means based on illusion; not real.
(a) illusory (b) inherent Imminent means about to happen.
(c) imminent (d) impassive Impassive means not feeling or showing emotion.
(e) ingenious Ingenious means (of a person) clever, original, and inventive.
145Ans.(a)
Q145. [I] The interviewer asked me to _________ my potential The blank can be filled with the word ‘articulate’ making the
contributions to the company. sentence meaningful. Articulate means having or showing the
[II] When I get nervous, it is very hard for me to ___________ my ability to speak fluently and coherently. Other words are irrelevant
and do not fit in the context of the sentence here. Hence option (a)
thoughts.
is the correct choice.
(a) articulate (b) languish Languish means be forced to remain in an unpleasant place or
(c) linguistic situation.
(d) perceive Linguistic means relating to language or linguistics.
(e) perforate Perceive means become aware or conscious of (something); come
to realize or understand.
Q146. [I] Older people who are unfamiliar with computer technology Perforate means pierce and make a hole or holes in.
often have an/a ________to it. 146Ans.(c)
[II] Vegetarians have a strong ____________ to eating meat. The blank can be filled with the word ‘aversion’ making the
(a) delight (b) elation (c) aversion sentence meaningful. Aversion means a strong dislike or
(d) function (e) cognizance disinclination. Other words are irrelevant and do not fit in the
Answers And Solutions context of the sentence here. Hence option (c) is the correct choice.
Delight means take great pleasure in.
142Ans.(a) Elation means great happiness and exhilaration.
The blank can be filled with the word ‘conjectures’ making the Cognizance means knowledge or awareness
sentence meaningful. Conjectures means an opinion or conclusion
formed on the basis of incomplete information. Other words are
irrelevant and do not fit in the context of the sentence here. Hence Direction (147-150): There is a sentence with a blank space in it.
option (a) is the correct choice. Choose the word from the given options which fits
Feint means make a deceptive or distracting movement, especially appropriately in the blank adding a proper and logical meaning
during a fight. to the sentence.
143Ans.(e)
The blank can be filled with the word ‘ambivalence’ making the Q147. He was remarkable for ugliness, and was an object of
sentence meaningful. Ambivalence means the state of having _______________ to his parents.
mixed feelings or contradictory ideas about something or someone. (a) delight (b) elation
Other words are irrelevant and do not fit in the context of the (c) aversion (d) function
sentence here. Hence option (e) is the correct choice. (e) cognizance
Illumination means lighting or light. Q148. I have never counted the number of posts, but I
Glee means great delight, especially from one's own good fortune ______________________ that there are less than five.
or another's misfortune. (a) conjecture (b) feint
Triumph means a great victory or achievement. (c) homage (d) case
144Ans.(b) (e) evidence
The blank can be filled with the word ‘inherent’ making the
sentence meaningful. Inherent means existing in something as a Q149. Respondents expressed_________________________ to their
permanent, essential or characteristic attribute. Other words are drugs in various ways.
(a) illumination (b) glee
Facebook Page- https://www.facebook.com/vishalpariharpage Youtube- https://www.youtube.com/vishalparihar
Follow
31 Vishal Sir Telegram Channel- https://t.me/englishbyvishalsirchannel Instagram- https://www.instagram.com/vishalthetrainer
By Vishal sir

SBI PO, SBI CLERK, IBPS PO, IBPS CLERK, RRB PO, RRB CLERK, NIACL,
- LIC RBI grade B, RBI ASSISTANT, & Other competitive Exams

(c) triumph (d) significance Triumph means a great victory or achievement.


(e) ambivalence
The blank can be filled with the word ‘ambivalence’ making the 150Ans.(b)
sentence meaningful. Ambivalence means the state of having The blank can be filled with the word ‘inherent’ making the
mixed feelings or contradictory ideas about something or someone. sentence meaningful. Inherent means existing in something as a
Other words are irrelevant and do not fit in the context of the permanent, essential or characteristic attribute. Other words are
sentence here. Hence option (e) is the correct choice. irrelevant and do not fit in the context of the sentence here. Hence
Illumination means lighting or light. option (b) is the correct choice.
Glee means great delight, especially from one's own good fortune Illusory means based on illusion; not real.
or another's misfortune. Imminent means about to happen.
Triumph means a great victory or achievement. Impassive means not feeling or showing emotion.
Ingenious means (of a person) clever, original, and inventive.
Q150. These are the great classic preachers whose discourses continue to
be read, and to form an _______________________ pare of the Direction (151-155): There are two different sentences with a blank
body of French literature. space in each sentence. Choose the word from the given options
(a) illusory (b) inherent which fits appropriately in both the blanks adding a proper and
(c) imminent (d) impassive logical meaning to the sentences.
(e) ingenious
Answers And Solutions Q151. [I] When I run into my ex-husband, I experience an ____________
so powerful that it makes me sick to my stomach.
147Ans.(c) [II] Even though Sarah has an ____________ to snakes, she is still
The blank can be filled with the word ‘aversion’ making the brave enough to walk through the forest with us on our hiking trips.
sentence meaningful. Aversion means a strong dislike or (a) delight (b) elation
(c) aversion (d) function
disinclination. Other words are irrelevant and do not fit in the
(e) cognizance
context of the sentence here. Hence option (c) is the correct choice.
Delight means take great pleasure in.
Elation means great happiness and exhilaration. Q152. [I] For the most part, you are merely stating ____________ when
Cognizance means knowledge or awareness you insist your team is going to win the football championship.
148Ans.(a) [II] After doing additional research, the scientist admitted his
The blank can be filled with the word ‘conjecture’ making the ____________ about the chemical reaction was incorrect.
sentence meaningful. Conjecture means an opinion or conclusion (a) conjecture (b) feint
formed on the basis of incomplete information. Other words are (c) homage (d) case
irrelevant and do not fit in the context of the sentence here. Hence (e) evidence
option (a) is the correct choice.
Feint means make a deceptive or distracting movement, especially Q153.
during a fight. [I] Clearly, the historical turn of events from non-violence to
Homage means special honour or respect shown publicly. nuclear armament, suggest a deep __________ about Mahatma
149Ans.(b) Gandhi's legacy.
The blank can be filled with the word ‘ambivalence’ making the [II] It led to a certain ____________ in their attitudes toward the
sentence meaningful. Ambivalence means the state of having organized labor movement, despite their commitment to greater
mixed feelings or contradictory ideas about something or someone. economic equality.
Other words are irrelevant and do not fit in the context of the (a) illumination (b) glee
sentence here. Hence option (e) is the correct choice. (c) triumph (d) significance
Illumination means lighting or light. (e) ambivalence
Glee means great delight, especially from one's own good fortune
or another's misfortune. Q154.

Facebook Page- https://www.facebook.com/vishalpariharpage Youtube- https://www.youtube.com/vishalparihar


Follow
32 Vishal Sir Telegram Channel- https://t.me/englishbyvishalsirchannel Instagram- https://www.instagram.com/vishalthetrainer
By Vishal sir

SBI PO, SBI CLERK, IBPS PO, IBPS CLERK, RRB PO, RRB CLERK, NIACL,
- LIC RBI grade B, RBI ASSISTANT, & Other competitive Exams

[I] She tried to be friendly with the newcomer, but she found his The blank can be filled with the word ‘impassive’ making the
____________ stare more than a little disturbing and excused sentence meaningful.
herself to join her friends. Impassive means not feeling or showing emotion. Other words are
[II] Although robots are well-known for being completely irrelevant and do not fit in the context of the sentence here. Hence
____________, there are many popular movies and TV shows that option (d) is the correct choice.
depict them as adorable creatures that actually do have feelings. Inherent means existing in something as a permanent, essential or
(a) illusory (b) inherent characteristic attribute.
(c) imminent (d) impassive Illusory means based on illusion; not real.
(e) ingenious Imminent means about to happen.
Ingenious means (of a person) clever, original, and inventive.
Q155. 155Ans.(e)
[I] If she would _________ the paper, she would be able to place it The blank can be filled with the word ‘perforate’ making the
in her notebook rings. sentence meaningful.
[II] A swift kick caused the ball to _________ the net and the game Perforate means pierce and make a hole or holes in. Other words
was paused until they found a new one. are irrelevant and do not fit in the context of the sentence here.
(a) articulate (b) languish Hence option (e) is the correct choice.
(c) linguistic (d) perceive Articulate means having or showing the ability to speak fluently
(e) perforate and coherently. Languish means be forced to remain in an
Answers And Solutions unpleasant place or situation.
Linguistic means relating to language or linguistics.
Perceive means become aware or conscious of (something); come
151Ans.(c) to realize or understand.
The blank can be filled with the word ‘aversion’ making the
sentence meaningful. Aversion means a strong dislike or Directions (156-160): In following questions, a sentence is given with
disinclination. Other words are irrelevant and do not fit in the two blanks. From the given alternatives, choose the most
context of the sentence here. Hence option (c) is the correct choice. appropriate set of words that fills the blank forming a
Delight means take great pleasure in. grammatically correct and contextually meaningful sentence.
Elation means great happiness and exhilaration.
152Ans.(a) Q156. Personal Growth is on the off chance that you need to satisfy the --
The blank can be filled with the word ‘conjecture’ making the ---(A)------- among you and your objectives and become the ------
sentence meaningful. Conjecture means an opinion or conclusion (B)--------- you plan to be, however, you’re stuck on the how.
formed on the basis of incomplete information. Other words are (a) goal, person (b) ambition, independent
irrelevant and do not fit in the context of the sentence here. Hence (c) aspiration, single (d) hole, individual
option (a) is the correct choice. (e) All except D
Feint means make a deceptive or distracting movement, especially
during a fight. Q157.
Homage means special honour or respect shown publicly. Devices and objects with built in sensors are connected to an
153Ans.(e) Internet of Things platform, which --- --(A)------- data from the
The blank can be filled with the word ‘ambivalence’ making the different devices and applies analytics to share the most valuable
sentence meaningful. Ambivalence means the state of having information with applications built to -----(B)------- specific needs.
mixed feelings or contradictory ideas about something or someone. A.receives, reach B.consolidates, set
Other words are irrelevant and do not fit in the context of the C.integrates, address D.gather, meet
sentence here. Hence option (e) is the correct choice. E.All of the above
Illumination means lighting or light.
Glee means great delight, especially from one's own good fortune Q158. The problem, so far, has been a --- --(A)------- mismatch between
or another's misfortune. China’s statements and the actions of its troops. Its consistent
Triumph means a great victory or achievement. labelling of India as the aggressor this summer --- --(B)------- the
154Ans.(d)

Facebook Page- https://www.facebook.com/vishalpariharpage Youtube- https://www.youtube.com/vishalparihar


Follow
33 Vishal Sir Telegram Channel- https://t.me/englishbyvishalsirchannel Instagram- https://www.instagram.com/vishalthetrainer
By Vishal sir

SBI PO, SBI CLERK, IBPS PO, IBPS CLERK, RRB PO, RRB CLERK, NIACL,
- LIC RBI grade B, RBI ASSISTANT, & Other competitive Exams

reality that India has, since May, ceded about 1,000 square here both grammatically and contextually Option D): is incorrect
kilometres in Ladakh to Chinese control. as fuzzy means difficult to perceive;
A.genesis, ratchet B.discard, inevitable 159Ans.(a)
C.stark, contradicts D.fuzzy, unravel Staying hydrated has a direct impact on our mental and physical
E.None of these well-being, and when your our bodies start to get dehydrated, it
takes a toll. First part of the sentence discusses positive side of
Q159. Staying ------ (A) -------- has a direct impact on our mental and staying hydrated. Second part discusses the negative side why your
physical well-being, and when your our bodies start to get----- (B) body get dehydrated. Other options are not contextually correct.
-----, it takes a toll. 160Ans.(c)Explanation-In the given sentence, only rattled and prospect
A.hydrated, dehydrated B.dehydrated, hydrated make it grammatically as well as contextually correct. Rattled
C.safe, unhealthy D.unhealthy, healthy means to make someone nervous or worried while prospect means
E.None of the above the possibility or likelihood of some future event.
Flustered means to make somebody feel nervous and confused
Q160. The syndrome, which can be deadly has --- --(A)------- parents and Perturbed means disturb, fluster, and upset
education officials as schools across the United States struggle with Implausible means not easy to believe
the --- --(B)------- of reopening in the fall and the coronavirus Rattled means to make somebody suddenly become worried
continues its spread.
A.flustered, implausibility B.perturbed, failure Directions (161-165): Each question below has one blank, indicating
C.rattled, prospect D.fazed, rarity that something has been omitted. Choose the word from the
E.unnerved, lookout options that best fits the meaning of the sentence as a whole.
Answers And Solutions Q161. Summer time comes with holidays and an opportunity for you to
unwind. It's that time of the month to head over to the Angsana Spa
156Ans.(d) & Resort, situated on the outskirts of the city to experience a few
Personal Growth is on the off chance that you need to satisfy the days of peace and __________
hole among you and your objectives and become the individual you (a) Punitive (b) Antagonism
plan to be, however, you’re stuck on the how. on the off chance - (c) Kindred (d) Tranquillity
If you do something on the off-chance , you do it because you hope (e) None of the above
that it will succeed. The sentences meant to say -> you can see your
Personal growth when you satisfy(fill) the hole(gap) among you Q162. The health minister _____ the need to make the laparoscopic
and your objectives. surgery available to the common people and widen it’s reach.
157Ans.(c) (a) underscored (b) hackneyed
Devices and objects with built in sensors are connected to an (c) neutralized (d) spotted
Internet of Things platform, which integrates data from the (e) repeated
different devices and applies analytics to share the most valuable
Q163. Dignitaries appreciated the __________________ participants for
information with applications built to address specific needs.
their exceptional performance and shared words of wisdom with
Option D “gather” is grammatically incorrect. Verb in “S” form is the prospective lawyers.
correct in the blank A. Options “reach” and “set” are contextually (a) Zealous (b) Displaced
unfit (c) Boisterous (d) Eschewed
158Ans.(c) (e) None of the above
In the given sentence, only ‘stark, contradicts’ make it
grammatically as well as contextually correct. Option A): is Q164. The banking sector, which has applied brakes on loans to risky
incorrect because ratchet means to cause something to rise (or fall) sectors, continues to face a credibility crisis in thewake of the Nirav
as a step in what is perceived as an irreversible process which does Modi and Mehul Choksi scams, the situation has only
____________ following the liquidity crisis and allegations of
not fit here. Option B): is incorrect as ‘discard’ does not make any
financial irregularities at Dewan Housing Finance Corp. Ltd, yet
sense here. Option C): is the correct alternative among the another NBFC.
following as ‘stark’ means sharp and ‘contradicts’ both of which fit (a) Exacerbated (b) Uplifted
(c) Educated (d) Attended
Facebook Page- https://www.facebook.com/vishalpariharpage Youtube- https://www.youtube.com/vishalparihar
Follow
34 Vishal Sir Telegram Channel- https://t.me/englishbyvishalsirchannel Instagram- https://www.instagram.com/vishalthetrainer
By Vishal sir

SBI PO, SBI CLERK, IBPS PO, IBPS CLERK, RRB PO, RRB CLERK, NIACL,
- LIC RBI grade B, RBI ASSISTANT, & Other competitive Exams

(e) None of the above None of the given alternatives fit the context of the sentences I and
Q165. Businessmen have been concerned that the Modi government's II.
policies were invasive to the point of being destructive, and the Hence, option (e) is the correct answer.
promise of lower tax rates never ________________. The word which could have satisfied the context of the sentences I
(a) Materialised (b) Analyzed and II are ‘requested’, ‘begged’, ‘cadged’ (means to ask for or
(c) Unauthorized (d) Compulsory obtain (something to which one is not strictly entitled);
(e) None of the above Answers And Solutions
Answers And Solutions
166Ans.(e)
161Ans.(d) Spawn [verb] means (of a fish, frog, mollusk, crustacean, etc.)
tranquillity means calmness, Other options are erroneous, punitive release or deposit eggs;
means punishment, antagonism means unfriendly, kindred mean Flayed (flay), berated (berate), belabored (belabor) are synonyms
blood relation. which mean to criticize severely and brutally;
162Ans.(a)
None of the given alternatives fit the context of the sentences I and
underscored- emphasize
hackneyed means (of a phrase or idea) having been overused; II.
unoriginal and trite. Hence, option (e) is the correct answer.
Word ‘underscored’ completes the sentence in a meaningful way. The word which could have satisfied the context of the sentences I
Hence, option (a) becomes the most suitable answer choice. and II are ‘requested’, ‘begged’, ‘cadged’ (means to ask for or
163Ans.(a) obtain (something to which one is not strictly entitled);
The correct option is A, zealous means passionate. Boisterous
means noisy, eschewed means avoid. Direction (167-170): In the given sentences, a blank is given indicating
164Ans.(a) that a word is missing. Among the four given options, a
exacerbated means to worsen, Other options are erroneous, uplift is combination of words would fit the blank thereby making it
a positive word which means being hopeful grammatically and contextually correct. Choose that option as
165Ans.(a) your response.
materialized means to happen or become active, Other options are Q167. Coming from a small town of only four hundred residents, Jonas
erroneous, and doesn’t fit contextually and grammatically. was shocked by the millions of people who made up the
______________ population of the big city.
Direction (166): There are two different sentences with a blank space A. homogenous
in each sentence. Choose the word from the given options which B. multifarious
fits appropriately in both the blanks adding a proper and C. variegated
logical meaning to the sentences. D. variety
(a) AB (b) BC (c) BD
Q166. (d) CD (e) None of these
I. A soldier came in and ____________ a pack of cigarettes today,
but as quartermaster I couldn’t provide him with something like Q168. Because Connie loved sugary foods, she found it very hard to stick
that. to her ______ diet, which did not include sweets and fatty foods.
II. The village leader has ____________ an audience with the king, A. sybaritic B. luxurious
but who knows if a man of his stature will spend his valuable time C. ascetic D. austere
on our plight. (a) AB (b) BC
(1) Spawned (2) Flayed (c) BD (d) CD
(3) Berated (4) Belaboured (e) None of these
(a) Only (1) (b) Only (4)
(c) Both (2) and (3) (d) All of (1), (2), (3) and (4) Q169. Waiting all her life for this moment, Susan Boyle sung 'I dream of
(e) None of (1), (2), (3) and (4) dream' with ________________, despite having an unwelcoming
audience.
A. alacrity B. apathy

Facebook Page- https://www.facebook.com/vishalpariharpage Youtube- https://www.youtube.com/vishalparihar


Follow
35 Vishal Sir Telegram Channel- https://t.me/englishbyvishalsirchannel Instagram- https://www.instagram.com/vishalthetrainer
By Vishal sir

SBI PO, SBI CLERK, IBPS PO, IBPS CLERK, RRB PO, RRB CLERK, NIACL,
- LIC RBI grade B, RBI ASSISTANT, & Other competitive Exams

C. indifference D. ardour
(a)AD (b) BC Q171.
(c) BD (d) CD I. Rather than breaking her limbs like rag doll back flipping over
(e) None of the above Niagara she doggy _____________ back to safety in 20 seconds
flat.
Q170. After the humiliating way his girlfriend dumped him, Roger’s
II. There were two occupants of the craft, and, though but mere
friends rallied to his side to act as a ___________ to his deflated
ego. youths, none could have handled the _______________ more
A. buttress B. undermine skilfully.
C. subvert D. stanchion (a) Work (b) Strike
(a) AB (b) BC (c) Gun (d) Paddles
(c) BD (d) AD (e) All can fit
(e) CD
Answers And solutions
Q172. I. Only when she reached the other side of the bridge did she
167Ans.(b) _________ herself to breathe.
Multifarious and variegated would correctly fit in the sentence II. The _____________ is withdrawn if the clearance is not
(many or of various types). Homogeneous- of similar type, which
obtained within three years.
is opposite of what the writer is trying to convey. Variety- noun:
grammatical misfit since an adjective is required. Hence, (b) is the (a) Prevail (b) Help
right answer. (c) Permit (d) Proof
168Ans.(d) (e) All can fit
Sybaritic- fond of sensuous luxury or pleasure; self-indulgent. Q173.
Ascetic/austere- (of living conditions or a way of life) having no I. To be honest, John is more than happy to stay at home and
comforts or luxuries. As per the sentence's context, the adjective _________ dad.
we are looking for should be on similar lines with 'strict' to II. Since the last service there's been a lot of __________ in the
maintain coherency. Ascetic/austere would perfectly fit in; hence, steering.
(d) is the right answer. (a)Play (b)Improvement
(c) Light (d) Start
(e) All can fit
169Ans.(a)
Alacrity- cheerful promptness or willingness; eagerness; speed or Q174.
quickness. Ardour- great enthusiasm or passion. Apathy/ I. they handed him her ticket, which he _____________ and
indifference- lack of interest, enthusiasm, concern. hence, option returned with a grin.
(a) is the right answer. II. She __________ a few buttons on her keyboard and proceeded
170Ans.(d) to interrogate the suspect.
Buttress- support physically; prop up; support something or (a)Clicked (b) Punched
someone by supplying evidence; undermine- lessen the (c) Proved (d) Cleaned
effectiveness, power, or ability of, especially gradually or (e) None can fit
insidiously; subvert- undermine the power and authority of (an
established system or institution); stanchion- an upright bar, post, Q175.
or frame forming a support or barrier. Both 'buttress' and I. The floor seemed to be pitching, like a boat ____________ by
‘stanchion’ fit in the blank as per the context; hence, option (d) is waves.
the right answer. II. Situated in a sensitive seismic zone, the state has been
Direction (171-175): In each of the questions given below, two ____________ by many tremors.
sentences are given with one blank each. From the given options, (a) Smiled (b) Powered
choose the word that correctly fits both the sentences. c) Clocked (d) Rocked

Facebook Page- https://www.facebook.com/vishalpariharpage Youtube- https://www.youtube.com/vishalparihar


Follow
36 Vishal Sir Telegram Channel- https://t.me/englishbyvishalsirchannel Instagram- https://www.instagram.com/vishalthetrainer
By Vishal sir

SBI PO, SBI CLERK, IBPS PO, IBPS CLERK, RRB PO, RRB CLERK, NIACL,
- LIC RBI grade B, RBI ASSISTANT, & Other competitive Exams

e) None can fit grammatically and contextually correct. Choose that option as
your response.
Answers And Solutions Q176. As a career woman and mother, she was constantly busy with the
__________ activities of her daily life.
171Ans.(d) A. homogenous B. multifarious
Paddle - Move through the water in a boat using a paddle or paddle. C. variegated D. variety
Paddle - A short pole with a broad blade at one or both ends, used a) AB b) BC
c) BD d) CD
without a rowlock to move a small boat or canoe through the water.
e) None of these
The dog paddle or doggy paddle is a simple swimming style. The
first meaning is suitable for the first blank and the second for the Q177. The wealthy, self-indulgent young man felt oddly drawn to the
second blank. None of the other options can fit both the blanks to strict, _______ life led by members of some monastic orders.
make them grammatically or contextually correct. D is the right A. sybaritic B. luxurious
answer. C. ascetic D. austere
172Ans.(c) a) AB b) BC
c) BD d) CD
Permit - Provide an opportunity or scope for (something) to take e) None of these
place; make possible. Permit - An official document giving
someone authorization to do something. The first meaning can fit Q178. While he once had the desire to learn Chinese, his _________ to
in the first blank and the second in the second blank. 'Prevail' and master the language faded after realizing how difficult it really is.
'help' do not fit in the second sentence. 'Proof' does not fit in the A. alacrity B. apathy
first sentence as we need a verb here. C is the right answer. C. indifference D. ardour
(a) AD (b) BC (c) BD
173Ans.(a)
(d) CD (e) None of the above
Play - Behave as though one were (a specified type of person). Play
- The space in or through which a mechanism can or does move.
The first meaning suits the first blank and the second meaning can Q179. Today's board members need to discuss these issues, the
suit the second blank. 'Improvement' does not fit grammatically in technologies that __________ them, the role their company
the first sentence. 'light' and 'start' do not fit in either of the wishes to play in their evolution and the potential impacts of
sentences. A is the right answer. investing or not.
174Ans.(b) A. buttress B. undermine
Punch - Pierce a hole in (paper, leather, metal, etc.) with or as C. subvert D. underpin
though with a punch. Punch - Press (a button or key on a machine). a) AB b) BC
The first meaning can fit in the first blank and the second can fit in c) BD d) AD
the second blank. 'Clicked' cannot fit in the first blank. The other e) CD
two do not fit the context of either sentences. B is the right answer. Q180. The extent and glamour of his empire exercised a ________ spell
175Ans.(d) on western Europe.
Rock - Move gently to and fro or from side to side. Rock - (with A. obscure B. indistinct
reference to a building or region) shake or cause to shake or vibrate, C. cogent D. potent
especially because of an impact, earthquake, or explosion. The first a) AB b) BC
c) BD d) CD
meaning can fit in the first blank and the second can fit in the
e) None of the above
second blank. None of the other words can fit in both the sentences
in their respective contexts. D is the right answer
Answers And Solutions
176Ans.(b)
Direction (176-180): In the given sentences, a blank is given indicating
that a word is missing. Among the four given options, a Since the sentence talks about two responsibilities/aspects of the
combination of words would fit the blank thereby making it woman, the adjectives which would qualify 'activities'(noun) would

Facebook Page- https://www.facebook.com/vishalpariharpage Youtube- https://www.youtube.com/vishalparihar


Follow
37 Vishal Sir Telegram Channel- https://t.me/englishbyvishalsirchannel Instagram- https://www.instagram.com/vishalthetrainer
By Vishal sir

SBI PO, SBI CLERK, IBPS PO, IBPS CLERK, RRB PO, RRB CLERK, NIACL,
- LIC RBI grade B, RBI ASSISTANT, & Other competitive Exams

reflect the same. Multifarious and variegated would correctly fit in (e) None of these
the sentence (many or of various types). Homogeneous- of similar Q183. By-----(A)--------- in corrupt practices, company managers expose
type, which is opposite of what the writer is trying to convey. themselves to blackmail. -----(B)----- ---- the security of staff, plant
Variety- noun: grammatical misfit since an adjective is required. and other assets are put at risk.
Hence, (b) is the right answer. (a) conspiring, However (b) indulging, At the same time
177Ans.(d) (c) involving, Due to (d) engaging, Consequently
Sybaritic- fond of sensuous luxury or pleasure; self-indulgent. (e) All except D
Ascetic/austere- (of living conditions or a way of life) having no
comforts or luxuries. As per the sentence's context, the adjective Q184. From purchasing groceries via net banking to -----(A)--------- a cup
we are looking for should be on similar lines with 'strict' to maintain of tea using mobile wallets, Indians are -----(B)--------- turning to
coherency. Ascetic/austere would perfectly fit in; hence, (d) is the digital payments for their daily transactions.
right answer. (a) buy, increasing (b) purchasing, increment
178Ans.(a) (c) buying, increasingly (d) making, swiftly
Alacrity- cheerful promptness or willingness; eagerness; speed or (e) None of the above
quickness. Ardour- great enthusiasm or passion. Apathy/
indifference- lack of interest, enthusiasm, concern. As per the Q185. That is twice as -----(A)--------- as the internal combustion engine,
statement, since the person was enthusiastic and eager (raring) to but only five percentage points -----(B)---------than a diesel hybrid.
learn Chinese language, 'alacrity' and 'ardour' would fit in the blank; (a) good, better (b) best, good
hence, option (a) is the right answer. (c) nice, best (d) super, good
179Ans.(d) (e) None of these
Buttress- support physically; prop up; support something or
someone by supplying evidence; undermine- lessen the
Answers And Solutions
effectiveness, power, or ability of, especially gradually or 181Ans.(d)
insidiously; subvert- undermine the power and authority of (an ‘develop, denied’, fits in the sentence correctly. Other options do
established system or institution); underpin- support, justify, or not fit in the sentence. Indulged means to allow yourself to have or
form the basis for. Both 'buttress' and 'underpin' fit in the blank as do something for pleasure
per the context; hence, option (d) is the right answer. Denied means to state that something is not true; to refuse to admit
180Ans.(d) or accept something
Obscure- not clearly expressed or easily understood; Indistinct- not Enhanced means to increase
clear or sharply defined; Cogent- clear, logical, and convincing; 182Ans.(a)
Potent- having great power, influence, or effect. 'Cogent' and ‘purpose, reference’, fits in the sentence correctly. Other options do
'potent' would fit in the blank perfectly; hence, option (d) is the right not fit in the sentence. Reverence means a feeling of great respect
answer.
183Ans.(d)
Direction (181-185): In the question below, a sentence is given with By engaging in corrupt practices, company managers expose
two blanks, that indicate that some parts are missing. Identify themselves to blackmail. Consequently, the security of staff, plant
the correct pair of words that fit in the sentence to make it and other assets are put at risk. The given sentence conveys
grammatically and contextually correct. company managers engage themselves in corrupt practices thereby
putting other staffs plant and other assets at risk. So Only option
Q181. Mental tension is a general phenomenon in people who -----(A)---- that fits blank B is “Consequently”. Consequently - by that means;
----- peculiar behavioral patterns when their urge is -----(B)--------- as a result of that.
(a) built, stop (b) enhanced, banned 184Ans.(c)
(c) indulge, stopping (d) develop, denied In order to follow the parallelism rule, the verb in the first blank
(e) None of these should be in continuous form (because of 'purchasing'). 2nd blank-
Q182. The -----(A)--------- of note-making is usually for one’s own it requires an adverb, which would qualify the verb 'turning'
personal -----(B)--------- ('increasingly' and 'swiftly' would fit). Option (c) satisfies the
(a) needy, identity (b) aims, randomness above-mentioned criteria; hence, (c) is the right answer.
(c) purpose, reference (d) source, reverence 185Ans.(a)

Facebook Page- https://www.facebook.com/vishalpariharpage Youtube- https://www.youtube.com/vishalparihar


Follow
38 Vishal Sir Telegram Channel- https://t.me/englishbyvishalsirchannel Instagram- https://www.instagram.com/vishalthetrainer
By Vishal sir

SBI PO, SBI CLERK, IBPS PO, IBPS CLERK, RRB PO, RRB CLERK, NIACL,
- LIC RBI grade B, RBI ASSISTANT, & Other competitive Exams

‘good, better’, fits in the sentence correctly. best, good;nice, best; jingoism- extreme patriotism, especially in the form of aggressive
super, good--- these pairs does not make sense grammatically and or warlike foreign policy.
contextually with the sentence. Hence these do not fit in the dissenting- holding or expressing opinions that are at variance with
sentence. those commonly or officially held.
racialism- related to racism gape- be or become wide open.
Directions (186-190): Each question below has two blanks. There are nuances- a subtle difference in or shade of meaning, expression, or
five pairs of words below the sentence. Each pair is lettered. sound.
Choose the pair of words which can be filled up in the blanks murky- obscure or morally questionable.
in the sentence in the same given order, to complete the 187Ans.(d)
sentences meaningfully. Sol.
effeminate- (of a man) having characteristics regarded as typical of
Q186. In this movement of drum-beating, where ................... as patriotism a woman; unmanly.
is the order of the day, a/an .................. voice is not welcome.
patchwork- a thing composed of many different elements so as to
(a) modulation, murky (b) racialism, marvelous
(c) jingoism, dissenting (d) gape, erroneous appear variegated.
(e) nuances, applauding profiteering- make or seek to make an excessive or unfair profit,
especially illegally.
Q187. Peace is not an/a ................. challenge to the machismo of the presidential- relating to a president or presidency.
national security state as idol but a civilisational response to the 188Ans.(b)
easy .............. of the nation state disseminating- spread (something, especially information) widely.
(a) presidential, rational (b) profiteering, concession
offset- the amount or distance by which something is out of line.
(c) patchwork, initiative (d) effeminate, brutality
189Ans.(b)
(e) negotiable, regards
incumbent- necessary for (someone) as a duty or responsibility.
Q188. Public health officials must ..................... data quickly and protégé- a person who is guided and supported by an older and
more experienced or influential person.
transparently, so that it can be ..................... by the global scientific rationalisation- the action of attempting to explain or justify
community. behaviour or an attitude with logical reasons, even if these are not
(a) provision, forced (b) disseminate, analysed appropriate
(c) offset, signified (d) diplomat, initiated
(e) adopt, reported 190Ans.(c)
Q189. It is ..................... on the government and the polity to foster a civic tenebrous- dark; shadowy or obscure.
...................... that allows equality of opportunity for all residents barbarities- barbarities
in the State. downplaying- make (something) appear less important than it
(a) barrier, aegis really is.
(b) incumbent, consciousness turbid- cloudy, opaque, or thick with suspended matter.
(c) preventive, accomplishment muddy- covered in or full of mud.
(d) protégé, rationalisation
Direction (191-195): There are two different sentences with a blank
(e) comparative, periodisation
Q190. When the ................. began, there were worrying indications that space in each sentence. Choose the word from the given options
which fits appropriately in both the blanks adding a proper and
Central and State health officials were ..................... the risk to
logical meaning to the sentences.
women.
(a) tenebrous, criticising (b) barbarities, accomplishing
Q191. [I] The mother-in-law would demean her, as a way to _______ the
(c) epidemics, downplaying (d) turbid, formulating
(e) muddy, evolving young wife in the son's eyes.
[II]. Matthew would __________ others in an attempt to elevate
Answers And Solutions himself
186Ans.(c) (a) add (b) abase

Facebook Page- https://www.facebook.com/vishalpariharpage Youtube- https://www.youtube.com/vishalparihar


Follow
39 Vishal Sir Telegram Channel- https://t.me/englishbyvishalsirchannel Instagram- https://www.instagram.com/vishalthetrainer
By Vishal sir

SBI PO, SBI CLERK, IBPS PO, IBPS CLERK, RRB PO, RRB CLERK, NIACL,
- LIC RBI grade B, RBI ASSISTANT, & Other competitive Exams

(c) allow (d) believe 193Ans.(c)


(e) cash Among the given options, the word ‘feed’ correctly fits the context
of both the sentences.
Hence, the option (c) is the correct answer.
Q192. [I]. Napoleon's short hair was _________ and matted on the 194Ans.(e)
forehead, but his face, though puffy and yellow, expressed physical The first sentence shows a contrast. In the initial phrase, it talks
satisfaction. about being disgusted, so, in the next phrase, the word to be used
[II] Prince Andrew touched the head with his hand; even the hair should be happy or glad.
was ___________, so profusely had the child perspired. The word glad appropriately fits the blank of the second sentence.
(a) wet (b) caring Hence, the option (e) is the correct answer.
(c) modern (d) hurting 195Ans.(d)
(e) noisy What could one do with hair up so to have a messy ponytail? She/he
could tie her/his hairs.
Q193. [I]My own brothers want me to break the Code to _______Sasha The word ‘tie’ also fits appropriately with the blank of the second
to the wolves sentence.
[II]You should never _______ a human a rock, even by mistake. Hence, the option (d) is the correct answer.
(a) notice (b) play
(c) feed (d) research Directions (196-200): Each question below has one blank, which is
(e) reserve indicating that something has been omitted. Find out which
option can be used to fill up the blank in the sentence in the
Q194. [I]She was ___________ Howard was there and that her family same sequence to make it meaningfully complete.
wasn't.
[II]How sorry I am that he died, and how _______ I am that he is Q196. ____________ economic growth, rural distress and less disposable
alive again! incomes have left buyers more cautious than they have been in the
(a) sick (b) quiet
recent past.
(c) rocked (d) serious
(e) glad a) Sluggish b) Marshal
c) Epilogue d) Panned
Q195. [I]The only decision he'd made was to do nothing until there was e) None of the above
clear evidence ________ Byrne to the money.
[II]He catches them by means of a rod smeared with bird-lime, and Q197 We have consistently been overdrafting on the services provided
then _________ a fine string under their wings, he flies them at its by Nature and have invited its ____________ in the form of
end.
draughts, floods, global warming, and ultimately climate change.
(a) spending (b) running (c) opening
(d) tying (e) meeting Tree planting is the best way to seek forgiveness of our Mother
Earth.
Answers And Solutions (a) Remarkable (b) Squad
191Ans.(b)
(c) Vagaries (d) Blossoms
abase [verb] means ‘behave in a way that belittles or degrades
(someone)’; (e) None of these
So, among the given words, the correct answer is the option (b).
192Ans.(a) Q198. The crusade against corruption should continue but the
On what type of rock, a person could slip? On a wet rock, a person ________________ should not kill businesses.
could slip is an appropriate answer. (a) Communities (b) Productivity
The word ‘wet’ also fits appropriately in the blank of the first (c) Connectivity (d) Zealousness
sentence.
(e) None of the above
Among the given options, the word ‘wet’ appropriately fits the
blank of both sentences. Hence, the option (a) is the correct answer.

Facebook Page- https://www.facebook.com/vishalpariharpage Youtube- https://www.youtube.com/vishalparihar


Follow
40 Vishal Sir Telegram Channel- https://t.me/englishbyvishalsirchannel Instagram- https://www.instagram.com/vishalthetrainer
By Vishal sir

SBI PO, SBI CLERK, IBPS PO, IBPS CLERK, RRB PO, RRB CLERK, NIACL,
- LIC RBI grade B, RBI ASSISTANT, & Other competitive Exams

Q199. The college has _______________ infrastructure, including a [II] Dean wasn't sure if it was her natural __________ to anything
massive sports complex and a library with digital check-in/check- involving law enforcement or concern for her boss's future
out. Another striking feature is the sanitation park, SRCC's waste (a) delight (b) elation
management facility.
(c) aversion (d) function
(a) Enviable (b) Expeditious
(c) Explicit (d) Excoriate (e) cognizance
(e) None of the above words are irrelevant and do not fit in the context of the sentence here.
Hence option (c) is the correct choice.
Delight means take great pleasure in.
Q200. To scale up research activities, the institute is _______________ Elation means great happiness and exhilaration.
existing lab and diagnostic facilities and setting up advanced Cognizance means knowledge or awareness
facilities to enhance new skill sets.
a) Refurbishing b) Ninjutsu Q202.
[I] I have never counted the number of posts, but
c) Bewailing d) Klutzy
I ____________ that there are less than five.
e) None of the above [II] One way of repairing the original ____________ is to force the
Answers And Solutions involved triangle-free graphs to have a large minimum degree
196Ans.(a) (a) conjecture (b) feint
(c) homage (d) case
sluggish means little motion. Marshal means officer, epilogue
(e) evidence
means concluding speech, panned means criticize severely
197Ans.(c) Q203.
vagaries mean unpredictable occurrences, other options are [I] It led to a certain ambivalence in their attitudes toward the
erroneous, Other options are erroneous. organized labor movement, despite their commitment to greater
198Ans.(d) economic equality.
zealousness which means intense passion for something, Other [II] The reaction of the American public to the invasion of Iraq
reveals a startling __________ to war in contemporary times
options are contextually wrong. This statement talks about putting
(a) illumination (b) glee
a lot of effort to end corruption. (c) triumph (d) significance
199Ans.(a) (e) ambivalence
enviable means desiring something which is possessed by another. not fit in the context of the sentence here. Hence option (e) is the
Excoriate means scraping the part of skin, expeditious means to correct choice.
speed up,explicit means directly expressed. Illumination means lighting or light.
Glee means great delight, especially from one's own good fortune
200Ans.(a) or another's misfortune.
refurbishing means to redecorate, ninjutsu means marrial art used Triumph means a great victory or achievement.
for Spying, bewailing means cry, klutzy means lack of skill
Q204.
Direction (201-205): There are two different sentences with a blank [I] He was evidently distressed, and breathed painfully, but could
space in each sentence. Choose the word from the given options not restrain the wild laughter that convulsed his
usually _____________ features.
which fits appropriately in both the blanks adding a proper and
[II] Gabriel sensed her emotions, even if she tried to keep her
logical meaning to the sentences. face ___________.
Q201.. (a) illusory (b) inherent
[I] The prominent features of his character seem to have been (c) imminent (d) impassive
cunning, ambition and avarice, combined with want of courage (e) ingenious
and __________ from effort.
Facebook Page- https://www.facebook.com/vishalpariharpage Youtube- https://www.youtube.com/vishalparihar
Follow
41 Vishal Sir Telegram Channel- https://t.me/englishbyvishalsirchannel Instagram- https://www.instagram.com/vishalthetrainer
By Vishal sir

SBI PO, SBI CLERK, IBPS PO, IBPS CLERK, RRB PO, RRB CLERK, NIACL,
- LIC RBI grade B, RBI ASSISTANT, & Other competitive Exams

Q205. 205Ans.(e)
[I] The inner surface of their cup-shaped mouth is armed with The blank can be filled with the word ‘perforate’ making the
pointed teeth, with which they ___________the integuments of the sentence meaningful.
fish attacked, scraping off particles of the flesh and sucking the Perforate means pierce and make a hole or holes in. Other words
blood. are irrelevant and do not fit in the context of the sentence here.
[II] In old specimens some of the longer, lower teeth work their tips Hence option (e) is the correct choice.
into deep pits, and ultimately even _____________ the
corresponding parts of the upper jaw.
(a) articulate (b) languish Directions (206-210): In each of the questions given below, a sentence
(c) linguistic (d) perceive is given with one blank. Below each sentence, FOUR words are
(e) perforate given out of which two can fit into the sentence. Five options
Articulate means having or showing the ability to speak fluently are provided with various combinations of these words. You
and coherently. Languish means be forced to remain in an have to choose the combination with the correct set of words
unpleasant place or situation. which can coherently fit into the given sentence.
Linguistic means relating to language or linguistics.
Perceive means become aware or conscious of (something); come Q206.. The idea that nuclear power can solve the coming energy crisis is
to realize or understand. therefore totally ___________.
Answers And Solutions (I) Virtuous (II) Scrupulous
201Ans.(c) (III) Flawed (IV) Fallacious
(a) (I) – (II) (b) (I) – (IV)
The blank can be filled with the word ‘aversion’ making the
(c) (II) – (III) (d) (III) - (IV)
sentence meaningful. Aversion means a strong dislike or (e) All of these
disinclination. Other
202Ans.(a) Q207. My sister's piano skills continued to ___________ mine, even
The blank can be filled with the word ‘conjecture’ making the though we practiced the same amount.
sentence meaningful. Conjecture means an opinion or conclusion (I) Surpass (II) Authorize
formed on the basis of incomplete information. Other words are (III) exceed (IV) Sanction
irrelevant and do not fit in the context of the sentence here. Hence (a) (I) – (II) (b) (I) – (IV)
option (a) is the correct choice. (c) (I) – (III) (d) (III) - (IV)
Feint means make a deceptive or distracting movement, especially (e) None of these
during a fight.
Homage means special honour or respect shown publicly. Q208 He was forced to fly to Shanghai last month in a last-minute bid to
203Ans.(e) head off a trade war about _____________________ textile
imports from China.
The blank can be filled with the word ‘ambivalence’ making the (I) Drizzling (II) Surging
sentence meaningful. Ambivalence means the state of having (III) Withered (IV) Gratifying
mixed feelings or contradictory ideas about something or someone. (a) (I) – (IV) (b) Only (II)
Other words are irrelevant and do (c) (I) – (II) (d) (III) - (IV)
204Ans.(d) (e) None of these
The blank can be filled with the word ‘impassive’ making the
sentence meaningful. Q209. The attack took place along an _____________________ trek into
Impassive means not feeling or showing emotion. Other words are the wildlife sanctuary on Kapiti Island which is heavily wooded at
irrelevant and do not fit in the context of the sentence here. Hence this time of year.
(I) Onerous (II) Arduous
option (d) is the correct choice.
(III) Mischievous (IV) Disastrous
Inherent means existing in something as a permanent, essential or (a) (I) – (IV) (b) (II) – (III)
characteristic attribute. (c) (I) – (II) (d) (III) - (IV)
Illusory means based on illusion; not real. (e) None of these
Imminent means about to happen.
Ingenious means (of a person) clever, original,
and inventive.
Facebook Page- https://www.facebook.com/vishalpariharpage Youtube- https://www.youtube.com/vishalparihar
Follow
42 Vishal Sir Telegram Channel- https://t.me/englishbyvishalsirchannel Instagram- https://www.instagram.com/vishalthetrainer
By Vishal sir

SBI PO, SBI CLERK, IBPS PO, IBPS CLERK, RRB PO, RRB CLERK, NIACL,
- LIC RBI grade B, RBI ASSISTANT, & Other competitive Exams

Q210. It is a conflict that occurs on the highest ________________ of The most appropriate combination of words is “realms and
thought and faces the fiercest of oppositions from the pragmatic, province”. All the other words fail to provide precise meaning to
dull-witted masses. the sentence. Hence, option (a) is the most suitable answer choice.
(I) Vicinity (II) Realms
Realms means a field or domain of activity or interest.
(III) Observations (IV) Province
Province means an area of special knowledge, interest, or
(a) (II) – (IV) (b) (I) – (III)
responsibility.
(c) (I) – (II) (d) (III) - (IV)
Vicinity means the area near or surrounding a particular place.
(e) None of these
Answers And Solutions Directions (211-214): Each question below has one blank, which is
206Ans.(d) indicating that something has been omitted. Find out which
The most appropriate combination of words is “Flawed and option can be used to fill up the blank in the sentence in the
Fallacious”. All the other words are irrelevant in context of the same sequence to make it meaningfully complete.
sentence. Hence, option (d) is the most suitable answer choice.
Virtuous means having or showing high moral standards.
Q211. Most teachers in school expect you to be ______ to both them and
Scrupulous means (of a person or process) careful, thorough, and
extremely attentive to details. the other students.
Flawed means having or characterized by a fundamental weakness (a)Exhausting (b)Polite (c)Supposing
or imperfection. (d)Commencing (e)Passing
Fallacious means based on a mistaken belief. Q212. There is a strong ________ that another earthquake will occur along
207Ans.(c) the fault.
The most appropriate combination of words is “surpass and (a) Inquiry (b) Unity
exceed”. All the other words are irrelevant in context of the (c) Probability (d) Security
sentence. Hence, option (c) is the most suitable answer choice.
(e) Restriction
Surpass means exceed; be greater than.
Sanction means give official permission or approval for (an action). Q213. Because of the waiter’s _____ over the meal, I decided to order the
208Ans.(b)
special.
The most appropriate word is “surging” that provides the absolute
(a)Neutrality (b)Opposition
context to the sentence. All the other words fail to provide precise
(c)Memory (d)Enthusiasm
meaning to the sentence. Hence, option (b) is the most suitable
(e)Frequency
answer choice.
Drizzling means rain lightly.
Surging means (of a crowd or a natural force) move suddenly and Q214. After losing his job, John realized he could not afford the _______
powerfully forward or upward. that came along with buying a new car.
Gratifying means give (someone) pleasure or satisfaction. (a) Downfall (b) Economy
209Ans.(c) (c) Safeguarding (d) Liability
As the sentence is providing information on rebuilding the (e) Betraying
infrastructure, the most suitable combination of words that fill the
blank is “onerous and arduous”. All the other words fail to provide
Answers And Solutions
211Ans.(b)
precise meaning to the sentence. Hence, option (c) is the most
suitable answer choice. The appropriate word to be used here is ‘polite’. If people are
Onerous means (of a task or responsibility) involving a great deal excessively polite in their behavior, feeling of warmth cannot be
of effort, trouble, or difficulty. expected from them. Exhausting, supposing and passing are
Arduous means involving or requiring strenuous effort; difficult irrelevant and cannot be used here. Commencing means beginning.
and tiring.
Mischievous means causing or showing a fondness for causing 212Ans.(c)
trouble in a playful way. The appropriate choice to be filled in blank is option (c).
Probability means possibility. Inquiry is a question which you ask
210Ans.(a) in order to get some information.
Facebook Page- https://www.facebook.com/vishalpariharpage Youtube- https://www.youtube.com/vishalparihar
Follow
43 Vishal Sir Telegram Channel- https://t.me/englishbyvishalsirchannel Instagram- https://www.instagram.com/vishalthetrainer
By Vishal sir

SBI PO, SBI CLERK, IBPS PO, IBPS CLERK, RRB PO, RRB CLERK, NIACL,
- LIC RBI grade B, RBI ASSISTANT, & Other competitive Exams

Unity means integration, togetherness. (c) planned. . . . increase (d) smart. . . .regress
Security means the state of being free from danger or threat. Q218. Historians have little evidence to show how a _____ farming
Restriction means limitation. community in Indian subcontinent took a giant _____ forward in
213Ans.(d) imagination.
The appropriate choice to be filled in blank is option (d). (a) backward. . . .idea (b) strong. . . .verdict
Enthusiasm is the most suitable word which means intense and (c) suppressed. . . . step (d) predominantly. . . .leap
eager enjoyment, interest, or approval.
Neutrality means absence of decided views, expression, or strong
feeling. Q219. The best place to _____ the spirit of a people is in their art,
Frequency means the rate at which something occurs over a especially literature, for creative artists are _____ to the
particular period of time or in a given sample. implications of the culture on life.
214Ans.(d) (a) discover. . . .privy (b) explore. . . .sensitive
Option (d) is the most appropriate choice. Liability means the state (c) dampen. . . . alert (d) strengthen. . . .averse
of being legally responsible for something.
The provision can only be protected or safeguarded. Q220. The twentieth century must surely be remembered as a time of
Downfall means a loss of power, prosperity, or status. _____ growth in the economics of the industrialized nations despite
Betraying means deceiving. the two most _____ wars ever fought in the whole of history.
(a) negative. . . . inhuman
Directions (215-230): Each question below has two blanks, which is (b) Staggered. . . . disastrous
indicating that something has been omitted. Find out which (c) Continued. . . . antagonistic
(d) Unprecedented. . . . destructive
option/s can be used to fill up the blank/s in the sentence in the
Continued means going on without stopping, antagonist means
same sequence to make it meaningfully complete. showing dislike to one another.
Q221. The revolutionaries of recent years cry out for _____ and
Q215. Nearly one in three Yeminis, more than seven million people,_____ individualism as did our forefathers, but the concept of liberty has
daily to find enough food to live a healthy and productive life, undergone a radical_____ along with the concept of morality.
leading to rates of ______ that are the third highest in the world. (a) freedom. . . . transformation
(a) struggles. . . .malnutrition (b) works. . . .employment (b) materialism. . . .amendment
(c) hunts. . . . decease (d) grieve. . . .nourishment (c) credentials. . . . change
(d) losses. . . .improvement
Q216. In our present culture crisis, with our materialistic values being
called into question, there are signs of a new interest in ____, in Q222. Conventional wisdom says truck demand is a fairly decent _____
those arts and disciplines which draw especially upon our
of economic health which is perhaps why last fiscal top gear
experience of _____ and meaning and the language representing
performances on the commercial vehicle front has _____ both
reality.
(a) science. . . .morals (b) morality. . . .god analyst and industry watchers alike.
(c) humanities. . . . value (d) philisophy. . . .life (a) incinerator. . . .bamboozled
Science means study of the structure and behaviour of the physical (b) indicator. . . .baffled
and natural world through observation and experiment, morals (c) incorporator. . . . reshuffled
means a lesson that can be derived from a story or experience. (d) incarnation. . . .fulfilled
Morality means principles concerning the distinction between right
and wrong or good and bad behaviour Q223. The______ earthquake which struck Haiti recently shows once
Philosophy means the study of the fundamental nature of again that we, as human beings, have always known: that even ____
knowledge, reality, and existence, especially when considered as the worst devastation, there is always hope.
an academic discipline. (a) destroying. . . .afore (b) huge. . . .enormous
(c) devastating. . . . amid (d) massive. . . .in
Q217. Government agencies, academia and local communities should
jointly develop ______ coastal zone management which will help Q224. For all the term signifies, the people seem pretty _____ with
_____ sustainable development from a desirable objective. modern democracy and they hold politicians in varying degree of
(a) strategic. . . .understand (b) integrated. . . .transform _____.
Facebook Page- https://www.facebook.com/vishalpariharpage Youtube- https://www.youtube.com/vishalparihar
Follow
44 Vishal Sir Telegram Channel- https://t.me/englishbyvishalsirchannel Instagram- https://www.instagram.com/vishalthetrainer
By Vishal sir

SBI PO, SBI CLERK, IBPS PO, IBPS CLERK, RRB PO, RRB CLERK, NIACL,
- LIC RBI grade B, RBI ASSISTANT, & Other competitive Exams

(a) displeased. . . .contempt (b) happy. . . .dislike Works means activity involving mental or physical effort done in
(c) disenchanted. . . . respect (d) disinterested. . . .adoration order to achieve a purpose or result, employment means the state
of having paid work.
Q225. The well-known judge who was earlier _____ to head the Hunts means search determinedly for someone or something, also
delegation has now given his _____. to pursue and kill (a wild animal) for sport or food, Decease a
(a) reluctant, consent (b) eager, approved person's death.
(c) requested, chances (d) forced, unwillingness Grieve means feel intense sorrow, the food necessary for growth,
(e) expected, assurances health, and good condition.
216Ans.(c)
Q226. People were ______ of the two brothers because their behavior Humanities means the quality of being humane; benevolence, value
seemed_____. the regard that something is held to deserve; the importance, worth,
or usefulness of something.
(a) anxious, notorious (b) suspicious, unusual
217Ans.(b)
(c) unmindful, gentle (d) aware, dispassionate
Integrated means to be united, to be together, transform means to
(e) cautious, generous change completely
Strategic means relating to the identification of long-term or overall
Q227. The army regime has announced firm steps on ______ democracy aims and interests and the means of achieving them, understand
and fighting _____ . means interpret or view (something) in a particular way.
(a) clamping, anarchy Planned means decide on and make arrangements for in advance,
(b) ruling, voters increase means to a rise in the size, amount, or degree of something.
(c) restoring, terrorism Smart means clean, tidy, and well dressed, regress means return to
(d) enlightening, anticorruption a former or less developed state.
(e) waiving, minorities 218Ans.(d)
Predominantly means having superior strength, influence, or
Q228. ______ the death of one of the contestants, the election to the seat authority : prevailing, leap means jump or spring a long way, to a
has been ______. great height, or with great force.
(a) Despite, postponed (b) Because of, conducted Backward means having made less progress than is normal or
(c) With, skipped (d) In spite of, stopped expected, Idea means a thought or suggestion as to a possible
(e) Owing to, countermanded course of action.
Strong means having the power to move heavy weights or perform
Q229. The captain of our team was _____ because of the _____ other physically demanding tasks, verdict means a decision on an
performance of our opponents. issue of fact in a civil or criminal case or an inquest.
(a) upset, poor (b) nervous, uneventful Suppresses means prevent the development, action, or expression
(c) happy, excellent (d) elated, bad of (a feeling, impulse, idea, etc.); restrain, step an act or movement
(e) annoyed, thorough of putting one leg in front of the other in walking or running.
. 219Ans.(b)
Explore means travel through (an unfamiliar area) in order to learn
Q230. These were arranged in accordance with their claim to about it, sensitive quick to detect or respond to slight changes,
_____________, especially considering their collaborative efforts. signals, or influences.
(a)priority, (b)underlining, Discover means find unexpectedly or during a search.
(c)prerogative, (d)perquisite, Dampen means make slightly wet, Alert means to be in the state of
(e)liberty, being watchful for possible danger.
Answers And Solutions Strengthen means make or become stronger, averse means having
a strong dislike of or opposition to something.
215Ans.(a) 220Ans.(d)
Struggles means make forceful or violent efforts to get free of Unprecedented means never having happened or existed before,
restraint or constriction, Malnutrition means lack of proper destructive means causing a lot of harm or damage.
nutrition, caused by not having enough to eat, not eating enough of Negative means bad or harmful, inhuman means very cruel and
the right things, or being unable to use the food that one does eat. without pity

Facebook Page- https://www.facebook.com/vishalpariharpage Youtube- https://www.youtube.com/vishalparihar


Follow
45 Vishal Sir Telegram Channel- https://t.me/englishbyvishalsirchannel Instagram- https://www.instagram.com/vishalthetrainer
By Vishal sir

SBI PO, SBI CLERK, IBPS PO, IBPS CLERK, RRB PO, RRB CLERK, NIACL,
- LIC RBI grade B, RBI ASSISTANT, & Other competitive Exams

Staggered means very surprised, disastrous means terrible, harmful 226Ans.(b)


or failing completely Suspicious-having or showing a cautious distrust of someone or
221Ans.(a) something.
Freedom means the right or ability to do what you want, 227Ans.(c)
transformation means process of changing completely Restoring - bring back or re-establish (a previous right, practice, or
Materialism means the belief that money and possession are the situation).Terrorism - the unofficial or unauthorized use of violence
most important things in life, amendment means a part that is added and intimidation in the pursuit of political aims.
or a small change that is made to a piece of writing, especially to a 228Ans.(e)
law. Countermanded - revoke or cancel (an order).
Credentials means a qualification, achievement, quality, or aspect 229Ans.(d)
of a person's background, especially when used to indicate their Elated - make (someone) ecstatically happy
suitability for something, change means make (someone or 230Ans.(a)
something) different; alter or modify. “priority” is the correct word that fits perfectly in the given blank.
Losses means the fact or process of losing something or someone, The word “priority” means “the fact or condition of being
improvement means make something better or is better than regarded or treated as more important than others.” Hence (a) is
something else. the correct option.
222Ans.(b) Prerogative means a right or privilege exclusive to a particular
Sol. Indicator means a thing that indicates the state or level of individual or class.
something, baffled means to be confused Perquisite means a benefit which one enjoys or is entitled to on
Incinerator means an apparatus for burning waste material, account of one's job or position.
especially industrial waste, at high temperatures until it is reduced
to ash, Bamboozled means cheat or fool. Direction (231-245): In the following question, the sentence given with
Incorporator means an incorporator is responsible for setting up a blank/blanks to be filled in with appropriate word/words.
corporation. This role is primarily executed before the business is Select the correct alternative out of the four and indicate it by
formed, interchange the positions of someone or something. selecting the appropriate option.
Incarnation means a person who embodies in the flesh a deity,
spirit, or quality, fulfilled means satisfied or happy because of fully Q231. George Bernard Shaw said that the contemporary social values
developing one's abilities or character. could be flippant and ___________ at some times.
(a) important (b) profound (c) inane
223Ans.(c) (d) riveting (e) absurd
Devastating means highly destructive or damaging, amid means Q232.
surrounded by; in the middle of. ___________ the general had been told by the field staff that
Destroying means end the existence of (something) by damaging
continuing warfare would be ___________ to his soldiers, he
or attacking it, afore means before.
Huge means very big, enormous means very big insisted on deploying more battle tanks.
Massive means very large. (a) Although...detrimental (b) Because...instrumental
224Ans.(a) (c) Although...formidable (d) Because...immediate
Displeased means feeling or showing annoyance and displeasure, (e) Because...incremental
contempt means the feeling that a person or a thing is worthless or
beneath consideration. Q233. The Senate warned the Prime Minister that if he did not accept their
Happy means feeling or showing pleasure or contentment, dislike advice, the differences between the Legislative and the Executive
means a feeling of distaste or hostility.
arms of the Government would be ___________ .
Disenchanted means disappointed by someone or something
previously respected or admired; disillusioned, adoration means (a) eliminated (b) eroded
deep love and respect. (c) exacerbated (d) exemplified
215Ans.(a) (e) exasperated
Reluctant-unwilling and hesitant; disinclined. Consent - permission .
for something to happen or agreement to do something. Q234.
Facebook Page- https://www.facebook.com/vishalpariharpage Youtube- https://www.youtube.com/vishalparihar
Follow
46 Vishal Sir Telegram Channel- https://t.me/englishbyvishalsirchannel Instagram- https://www.instagram.com/vishalthetrainer
By Vishal sir

SBI PO, SBI CLERK, IBPS PO, IBPS CLERK, RRB PO, RRB CLERK, NIACL,
- LIC RBI grade B, RBI ASSISTANT, & Other competitive Exams

Most journalistic writing could do with ___________ of (e) equal to the


paragraphs to make the prose more ___________ . Q241. Renu saw no ___________ in the exercises that had been set for
(a) suppression...legible (b) removal...argumentative her and worked at them in a __________ manner.
(c) simplification...abstruse (d) deletion...succinct (a) reason...organised (b) point...desultory
(e) exemplification...incoherent (c) strength...careless (d) meaning...dutiful
(e) sense...painful
Q235.
Varun was accused of murdering his wife, and though he was never Q242. Mother Teresa was ___________ about her achievements and
___________ , he never recovered from the shame and the scandal. unwilling to ___________ them before anyone.
(a) charged (b) booked (a) modest...discuss (b) proud...promote
(c) indicted (d) acquitted (c) unsure...reveal (d) ignorant...eulogise
(e) applauded (e) immodest...play

Q236. The earthquake warnings and the fragility of our houses make us Q243. Martin Luther strived for justice and fought against attempts to
certain that disaster is _________. ___________ ignorant peasants in the medieval ages.
(a) available (b) imminent (a) influence (b) change
(c) forthcoming (d) catastrophic (c) exploit (d) corrupt
(e) eminent (e) discharge

Q237. The television serials are characterized by an ______ story line and Q244. The instruction manual that came with the computer is no
_______ plots and sub-plots. masterpiece of ___________ prose because its instructions are so
(a) unending...convoluted ___________ that we still do not know how to set up the computer.
(b) infantile...simple (a) clear...garbled
(c) uniform...emotional (b) valuable...intelligent
(d) extempore...routine (c) worthless...clear
(e) erratic...forthcoming (d) confused...varied
(e) worth ... muffled
Q238. The ___________ transformation of the former Soviet Union or
Russia as it was popularly known, remains one of the biggest stories
Q245. The strike will not end because both the management and the
of the decade.
(a) smooth (b) singular workers are ___________ .
(c) tumultuous (d) prophetic (a) insipid (b) intransigent
(e) traumatic (c) intricate (d) inexplicable
(e) saddened
Q239. If mankind has to survive for long, it must ___________ and
deliberately renounce the fruits of ___________ and whirling Answers And Solutions
technology. 231Ans.(c)Inane means foolish
(a) wilfully...wild (b) honestly...responsible Flippant= not showing a serious or respectful attitude.
(c) sincerely...labor (d) effectively...sliding 232Ans.(a) The correct answer is: A
(e) knowingly...impending Detrimental= tending to cause harm.
Formidable= intimidating.
Q240. Although similar to moths in many ways, butterflies may be
___________ moths by the beauty of their wings. 233Ans.(c)The correct answer is: C
(a) akin to Exacerbate= to make worse
(b) in comparison with Exasperated= intensely irritated and frustrated
(c) listed as 234Ans.(d)
(d) distinguished from Legible= clear enough to read.

Facebook Page- https://www.facebook.com/vishalpariharpage Youtube- https://www.youtube.com/vishalparihar


Follow
47 Vishal Sir Telegram Channel- https://t.me/englishbyvishalsirchannel Instagram- https://www.instagram.com/vishalthetrainer
By Vishal sir

SBI PO, SBI CLERK, IBPS PO, IBPS CLERK, RRB PO, RRB CLERK, NIACL,
- LIC RBI grade B, RBI ASSISTANT, & Other competitive Exams

Abstruse = difficult to understand. (d) c-d (e) a-d


Exemplification=symbolization
Succinct= brief Q247. A good way to break the ______ of eating too quickly is to put your
Incoherent=unclear knife and fork down after each mouthful.
235Ans.(c) (a) Convention (b) Trauma
Indict=to charge with a crime. (c) Habit (d) Entree
Acquit= to free someone with charge. (A) a-c (B) b-c
236Ans.(b) (C) b-d (D) c-d
Imminent = certain to happen. (E) a-b
Catastrophic = disastrous.
237Ans.(a) Q248. In our house, our mother is a ____________ who ensures our
Convoluted means complicated, complex. constantly travelling father receives our messages.
Extempore= to speak without preparation. a) Conduit b) Greed
Infantile=childish c) Channel d) Inclination
238Ans.(c) (A) a-c (B) b-c
Tumultuous= loud, deafening, thunderous. (C) b-d (D) c-d
Prophetic= accurately predicting (E) a-d
239Ans.(a) Impending=be about to happen.
240Ans.(d) Q249. His encouragement, humour and optimism blotted out many of the
Akin to = of similar character. ____________ influences that had been present in my life as I was
241Ans.(b) growing up.
Desultory= lacking a plan, enthusiasm. a) Negative b) Unhealthy
242Ans.(a) c) Viral d) Thrift
Eulogise=to praise highly in speech or writing (A) a-c (B) b-a
243Ans.(c) (C) b-d (D) c-d
244Ans.(a) (E) a-d
Muffled=not loud, muted.
Garbled= unclear Q250. Though her advances were ____________, the married man
245Ans.(b) refused to accept the gorgeous woman’s advances.
a) Deceptive b) Tempting
Intransigent= refusing to change one's views c) Flawed d) Alluring
Intricate=very complicated, (A) a-b (B) b-c
Inexplicable=in a way that is impossible to separate. (C) b-d (D) c-d
Insipid=tasteless, flavourless. (E) a-c
Answers And Solutions
Direction (246-250): In the given question, one statement with a blank 246Ans.(e)
along with four words is given. Two of the given words can fit Restore means to bring back to a former, original, or normal
condition, as a building, statue, or painting.
in the blank. Five options with various combinations of these
Rejuvenate means to make (someone or something) look or feel
words are given. Choose the combination of the words that best better, younger, or more vital.
fit in the blank. Replace means to provide a substitute for (something that is broken,
old, or inoperative).
Q246. The government decided to _________ the failed policies of its Reinstate means restore (someone or something) to their former
predecessor. position or state.
(a) Restore (b) Rejuvenate Rejuvenate refers to a person, but the blank is talking about some
(c) Replace (d) Reinstate failed policies which is not a person. policies can be reinstated or
(a) a-b (b) b-c (c) b-d restored. Hence, the correct answer is E.

Facebook Page- https://www.facebook.com/vishalpariharpage Youtube- https://www.youtube.com/vishalparihar


Follow
48 Vishal Sir Telegram Channel- https://t.me/englishbyvishalsirchannel Instagram- https://www.instagram.com/vishalthetrainer
By Vishal sir

SBI PO, SBI CLERK, IBPS PO, IBPS CLERK, RRB PO, RRB CLERK, NIACL,
- LIC RBI grade B, RBI ASSISTANT, & Other competitive Exams

Q86. If we are not thoughtful about how we _______ our ideas at work, (a) aptitude, oppressive (b) ineptness, insurgent
we _____ not being heard or, worse, missing out on the credit we (c) capability, turbulence (d) potential, subversive
are due. (e) incompetence, riotous
(a) qualify, jeopardise Q93. Egypt was ruled not by kings and emperors as was common among
(b) project, produce other ........... at the time, but by a citizenry, which .............. fully in
(c) present, risk the affairs of the city.
(d) confess, endanger (a) populations, cooperated (b) societies, participated
(e) None of the above (c) cities, engaged (d) civilizations, supplemented
(e) monarchies, convalesced
Q87. Global carbon emissions ______ to be reduced to net zero by 2050
to have a good chance of holding global average temperature rises
to no more than 1.5oC, a level that would be __________, but not
Answers And Solutions
catastrophic for human civilization.
(a) needful, affliction (b) needs, adversity 71Ans. (e)
(c) needed, disaster (d) need, disastrous An adjective is needed in the blank. This eliminates option
(e) None of the above A(adverb), B(verb) and D(adverb). Option C is the correct answer.
72Ans. (c)
Q88. In a crisis that has become _________ dire over the past decade, An adjective is needed in the blank to qualify the noun
digital solutions - many with artificial intelligence (AI) at their core 'entrepreneurs'. This eliminates option A(verb/noun), B(verb in the
- offer ______ for reversing the decline in our mental wellness. past form) and D(verb/noun). Option C is the correct answer.
a) progressively, hope b) progression, hopeful 73Ans. (d)
c) progress, hopingly d) progressive, hope A noun is needed in the blank. This eliminates option A (gerund or
e) None of the above present participle) and C(adjective). 'Circulation' means the public
1st blank- an adverb is required here, which would qualify the availability or knowledge of something; it is contextually incorrect.
adjective 'dire'; 'progressively'- adverb, progression- noun, Option D is the correct answer. The sentence says that if you wait
progress- verb, progressive- adjective. 2nd blank- it requires a for perfection before switching roles or launching a new idea, it can
noun; only 'hope' fits in here. Hence, (a) is the right answer. be your greatest enemy.
Q89. Evidence from __________policies a decade ago suggests a 74Ans. (b)
_____________ multiplier effect, as spending cuts lead to falling An adverb is needed in the blank to qualify the adjective
incomes, leading to further spending cuts.
'struggling'. This eliminates option A(adjective), C(adjective) and
(a) austerity, substantial (b) extravagance, generous
(c) acerbity, slight (d) indulgence, consequential D(verb/noun). Option B is the correct answer.
(d) obduracy, trivial 75Ans.(c)
Q90. Politics is funded because political power provides an __________ The most appropriate choice. Safeguarding means protecting. The
of opportunity for ________________ aggrandizement of wealth, provision can only be protected or safeguarded.
in a way no regular business does. Downfall means a loss of power, prosperity, or status.
a) Plenitude, larcenous b) Opulence, voracious Liability means the state of being legally responsible for
c) Affluence, ferocious d) Myriad, aggressive
something.
e) Abundance, rapacious
Betraying means deceiving.
Q91. With the government being the __________ owners of the public 76Ans. (c)
sector banks, the finance In order to manage the traffic an approach of widening the roads is
Q92. The __________ for the ideas spread through print media to used that is the conventional or regular wisdom is used. It cannot
challenge official power structures led some political and religious be free-spirit or provided or groundbreaking or freakish.
authorities to actively suppress publications that they deemed 77Ans. (b)
__________. The appropriate word to be used here is ‘polite’. If people are
excessively polite in their behavior, feeling of warmth cannot be
Facebook Page- https://www.facebook.com/vishalpariharpage Youtube- https://www.youtube.com/vishalparihar
Follow
19 Vishal Sir Telegram Channel- https://t.me/englishbyvishalsirchannel Instagram- https://www.instagram.com/vishalthetrainer
By Vishal sir

SBI PO, SBI CLERK, IBPS PO, IBPS CLERK, RRB PO, RRB CLERK, NIACL,
- LIC RBI grade B, RBI ASSISTANT, & Other competitive Exams

247Ans.(a) Q252. The escaped defaulter is more than just an arrogant, elitist,
‘Habit’ is an action which is considered bad that someone does unashamed liar, and has been proved to work ____________ with
repeatedly and finds it difficult to stop doing, ‘Convention’ means the bankers.
a way in which something is usually done. Thus, both are I. hand in hand II. hand in glove
synonymous to each other. ‘Trauma’ means a deeply distressing or III. hand and foot
disturbing experience; ‘Entrée’ means the main course of a meal. (a) Only II
Both these words would be contextually incorrect in the given (b) Only I and III
blank. Hence, the correct answer is A. (c) Only I and II
248Ans.(a) (d) Only II and III
‘Conduit’ is a way of connecting two places thus making ‘Channel’ (e) All of these
its correct synonym. ‘Greed’ means intense and selfish desire for Ans.()The given statement talks about a defaulter who is more than
something, especially wealth, power, or food; ‘Inclination’ means an arrogant and unashamed liar who proved to work in alliance with
the fact or degree of sloping. Both these words would not fit in the the bankers. The structure of the sentence makes it clear that the
blank. Hence, the correct answer is A. words to be filled in complete the latter half of the statement.
249Ans.(b) Option I – The meaning of ‘hand– in–hand’ means in close
‘Viral’ means an image, video, piece of information, etc. that is association and cooperation’. It fit the sentence correctly.
circulated rapidly and widely on the Internet. ‘Thrift’ means the Option II – It makes the sentence grammatically correct. It also
quality of using money and other resources carefully and not means in close association and cooperation.
wastefully. Hence, the correct answer is B. Option III – It makes the sentence grammatically incorrect. ‘Hand
250Ans.(c)Neither ‘Deceptive’ which means misleading nor ‘Flawed’ is a and foot’ means with great and concentrated effort and is
positive trait, ‘Alluring’ means tempting and attractive. So, only unsuitable.
‘Tempting’ and ‘Alluring’ fit contextually. Other options don’t fit Thus, I and II mean the same and they both are suitable as part of
logically. Hence, option C is the correct answer. the sentence.

Q253. Yet, despite these far-reaching shifts, the sequences of words


Direction (251-255): A statement with one blank is given below.
Choose the set of words from the given options which can be themselves have been handed down _____________ from age to
used to fill the given blank. age.
I. about exact II. almost entire
Q251. The patentability and the scope of protection ____________ III. more or less intact
(a) Only I
Genetically Modified crops has been controversial from the very
(b) Only II
beginning.
(c) Only III
i. to be afforded to
(d) Only II and III
ii. to be commended to
(e) All of these
iii. to be plucked from
a) Only i Q254. At the time, I was ashamed about riding a bus; but looking back,
b) Only ii
I’ve come to see that the hours _______________ provided an
c) Both i and ii
education of their own.
d) Both ii and iii
I. I logged on public transit
e) All of the above
II. Of my daily commute
“To be afforded to” means to be provided to someone or something.
III. I strolled over the bus
In this sentence, some patentability and scope of protection is being
(a) Both I and II
provided to GM crops, which is termed as controversial. Hence, “to
(b) Both I and III
be afforded to” fits in correctly.
(c) Both II and III
“commendable” means praiseworthy, which doesn’t fit into the
(d) Only II
sentence. “Pluck” means to take away something, and nothing is
(e) None of these
being taken away from the crops here.

Facebook Page- https://www.facebook.com/vishalpariharpage Youtube- https://www.youtube.com/vishalparihar


Follow
49 Vishal Sir Telegram Channel- https://t.me/englishbyvishalsirchannel Instagram- https://www.instagram.com/vishalthetrainer
By Vishal sir

SBI PO, SBI CLERK, IBPS PO, IBPS CLERK, RRB PO, RRB CLERK, NIACL,
- LIC RBI grade B, RBI ASSISTANT, & Other competitive Exams

Q255. Application security encompasses measures or counter-measures Option I and option II can be used. Therefore, option A is the apt
that are taken _____ to protect applications from threats. answer.
i. During the development 255Ans.(a)
ii. For various reasons Option I is correct as it gives a time reference.
iii. Even during the upgradation of models Option II is incorrect as the part after the blank gives the reason as
to why are the measures taken.
(a) Both i and iii
Option III fits in as it also gives a time reference or cycle reference
(b) Both ii and iii to the measures.
(c) Only i Hence option A is the correct response.
(d) Only ii
(e) All of the above Direction (256-260): The following question consists of a sentence(s)
Answers And Solutions with one blank only. You are given four words as answer
choices and from the four choices you have to pick up two
correct answers, either of which will make the sentence
251Ans.(a) meaningfully complete.
252Ans.()
The given statement talks about a defaulter who is more than an Q.256. Since the postman had a _____________ attitude and often threw
arrogant and unashamed liar who proved to work in alliance with rocks at neighborhood dogs, he was fired from his position.
the bankers. The structure of the sentence makes it clear that the A. Belligerent B. Truculent
words to be filled in complete the latter half of the statement. C. Introvert D. Stupid
Option I – The meaning of ‘hand– in–hand’ means in close (a) CD (b) AC (c) AB
association and cooperation’. It fit the sentence correctly. (d) DB (e) AD
Option II – It makes the sentence grammatically correct. It also Q.257. Though she didn’t want to go on vacation with her parents, Jess
means in close association and cooperation. found it hard to ___________ their request.
Option III – It makes the sentence grammatically incorrect. ‘Hand A. Demur B. Obscene
and foot’ means with great and concentrated effort and is C. Object D. Herald
unsuitable. (a) AC (b) CD (c) BA
Thus, I and II mean the same and they both are suitable as part of (d) AD (e) BD
the sentence.
253Ans.() Q.258. She would have been justified to feel a sense of __________ after
The given statement talks about the sequence fo words that have losing her family, but instead she felt the need to keep going.
been handed down from age to age. The structure of the sentence
A. Despair B. Coddle
makes it clear that the words to be filled in complete the latter half
of the statement. C. Desperation D. Impost
Option I – It is incorrect as ‘about exact’ does not make sense. (a) BD (b) AC (c) AD
Option II – It makes the sentence grammatically incorrect. It would (d) BC (e) CD
have been correct if ‘entirely’ had been present instead of ‘entire’. Q.259. During the debate, the candidate implied his opponent’s
Option III – It makes the sentence grammatically correct. ‘Intact’ employment plan was not _____________ for the country.
means not damaged or impaired in any way; complete. A. Heedful B. Relevant
Thus only III is suitable as part of the sentence. C. Apposite D. Absorb
254Ans.(a) (a) BA (b) BD
(I): Logged on means to travel and to spend or accumulate time. (c) AD (d) AC
Public transit refers to public transportation. (e) BC
(II): Commute refers to a regular journey of some distance to and
Q.260. Because gifted students are more ____________ than students of
from one's place of work. lower intelligence, they usually turn in better assignments than their
(III): Stroll means to wander. It is quite bizarre to say that one peers.
gained experience of travelling in a bus by "strolling over" it. A. Waste B. Fecund

Facebook Page- https://www.facebook.com/vishalpariharpage Youtube- https://www.youtube.com/vishalparihar


Follow
50 Vishal Sir Telegram Channel- https://t.me/englishbyvishalsirchannel Instagram- https://www.instagram.com/vishalthetrainer
By Vishal sir

SBI PO, SBI CLERK, IBPS PO, IBPS CLERK, RRB PO, RRB CLERK, NIACL,
- LIC RBI grade B, RBI ASSISTANT, & Other competitive Exams

Q86. If we are not thoughtful about how we _______ our ideas at work, (a) aptitude, oppressive (b) ineptness, insurgent
we _____ not being heard or, worse, missing out on the credit we (c) capability, turbulence (d) potential, subversive
are due. (e) incompetence, riotous
(a) qualify, jeopardise Q93. Egypt was ruled not by kings and emperors as was common among
(b) project, produce other ........... at the time, but by a citizenry, which .............. fully in
(c) present, risk the affairs of the city.
(d) confess, endanger (a) populations, cooperated (b) societies, participated
(e) None of the above (c) cities, engaged (d) civilizations, supplemented
(e) monarchies, convalesced
Q87. Global carbon emissions ______ to be reduced to net zero by 2050
to have a good chance of holding global average temperature rises
to no more than 1.5oC, a level that would be __________, but not
Answers And Solutions
catastrophic for human civilization.
(a) needful, affliction (b) needs, adversity 71Ans. (e)
(c) needed, disaster (d) need, disastrous An adjective is needed in the blank. This eliminates option
(e) None of the above A(adverb), B(verb) and D(adverb). Option C is the correct answer.
72Ans. (c)
Q88. In a crisis that has become _________ dire over the past decade, An adjective is needed in the blank to qualify the noun
digital solutions - many with artificial intelligence (AI) at their core 'entrepreneurs'. This eliminates option A(verb/noun), B(verb in the
- offer ______ for reversing the decline in our mental wellness. past form) and D(verb/noun). Option C is the correct answer.
a) progressively, hope b) progression, hopeful 73Ans. (d)
c) progress, hopingly d) progressive, hope A noun is needed in the blank. This eliminates option A (gerund or
e) None of the above present participle) and C(adjective). 'Circulation' means the public
1st blank- an adverb is required here, which would qualify the availability or knowledge of something; it is contextually incorrect.
adjective 'dire'; 'progressively'- adverb, progression- noun, Option D is the correct answer. The sentence says that if you wait
progress- verb, progressive- adjective. 2nd blank- it requires a for perfection before switching roles or launching a new idea, it can
noun; only 'hope' fits in here. Hence, (a) is the right answer. be your greatest enemy.
Q89. Evidence from __________policies a decade ago suggests a 74Ans. (b)
_____________ multiplier effect, as spending cuts lead to falling An adverb is needed in the blank to qualify the adjective
incomes, leading to further spending cuts.
'struggling'. This eliminates option A(adjective), C(adjective) and
(a) austerity, substantial (b) extravagance, generous
(c) acerbity, slight (d) indulgence, consequential D(verb/noun). Option B is the correct answer.
(d) obduracy, trivial 75Ans.(c)
Q90. Politics is funded because political power provides an __________ The most appropriate choice. Safeguarding means protecting. The
of opportunity for ________________ aggrandizement of wealth, provision can only be protected or safeguarded.
in a way no regular business does. Downfall means a loss of power, prosperity, or status.
a) Plenitude, larcenous b) Opulence, voracious Liability means the state of being legally responsible for
c) Affluence, ferocious d) Myriad, aggressive
something.
e) Abundance, rapacious
Betraying means deceiving.
Q91. With the government being the __________ owners of the public 76Ans. (c)
sector banks, the finance In order to manage the traffic an approach of widening the roads is
Q92. The __________ for the ideas spread through print media to used that is the conventional or regular wisdom is used. It cannot
challenge official power structures led some political and religious be free-spirit or provided or groundbreaking or freakish.
authorities to actively suppress publications that they deemed 77Ans. (b)
__________. The appropriate word to be used here is ‘polite’. If people are
excessively polite in their behavior, feeling of warmth cannot be
Facebook Page- https://www.facebook.com/vishalpariharpage Youtube- https://www.youtube.com/vishalparihar
Follow
19 Vishal Sir Telegram Channel- https://t.me/englishbyvishalsirchannel Instagram- https://www.instagram.com/vishalthetrainer
By Vishal sir

SBI PO, SBI CLERK, IBPS PO, IBPS CLERK, RRB PO, RRB CLERK, NIACL,
- LIC RBI grade B, RBI ASSISTANT, & Other competitive Exams

C. Fertile D. Useless
(a) CD (b) BC (c) AD Direction (261-265): Two sentences with one blank in each, followed
(d) AB (e) BD by five alternatives, are given. Choose that option as the answer
Answers And Solutions which can fill both the blanks of both the sentences.
256Ans.(c) Q261.
Belligerent is hostile and aggressive. 1) Due to his status as a political ______, many people were eager
Truculent means eager or quick to argue or fight, aggressive. to vote for him in the next election.
Introvert is someone who is shy, quiet, and unable to make friends
2) Because he was a ______ in the fashion industry, he was always
easily.
Stupid is someone with a lack of intelligence or common sense. in need of reliable assistants.
Thus, both A and B make the sentence meaningful. (a) Maladroit (b) Malady
Thus option 3 is the correct answer. (c) Opprobrium (d) Magnate
(e) profound
257Ans.(a)
Demur means to raise objections or show reluctance. Q262. 1) The use of quadrature-formulae is important in _________ work,
Obscene means offensive or disgusting by accepted standards of where the fundamental tables are based on experience, and the
morality and decency. formulae applying these tables involve the use of the tabulated
Object means to feel or express opposition to something or values and their differences.
someone. 2) The government should release the __________ report on
Herald means to be a sign that something important, and often demand of the people.
good, is starting to happen. (a) Calculator (b) Analyst (c) Comptroller
Thus, both A and C make the sentence meaningful. (d) Auditor (e) Actuarial
Thus option 1 is the correct answer.
258Ans.(b) Q263. 1) He __________ his claim to the throne, as he wanted to serve as
Despair is the complete loss or absence of hope. a commoner for the development and betterment of his kingdom.
Coddle means to treat someone in an overprotective and kind way. 2) What were you thinking when you __________ how I should act
Desperation also means loss of hope and feeling of despair. with my wife and kids? They are mine, not yours.
Impost is a tax or similar compulsory payment. (a) Moved (b) Pronounced (c) Suggested
Thus, both A and C make the sentence meaningful. (d) Condemned (e) Announced
Thus option 2 is the correct answer.
259Ans.(e) Q264.
Heedful means aware and careful. 1) The decision to continue the investigation was not so much about
Relevant means closely connected or appropriate to what is being doubting the veracity of the witness’s statement, which had been
done or considered. corroborated by other reliable interviews, as it was about a
Apposite means suitable and right for the occasion. conviction that there was further evidence that could play a
Absorb means take in or soak up. _______ role in the case.
Thus, both B and C make the sentence meaningful. 2) In his letter, he claimed that he just wanted to say hello, but the
Thus option 5 is the correct answer. more _________ message to me was that he was lonely and badly
260Ans.(b) needed a friend to talk to."
Waste means something no longer of use. (a) Negligible (b) Salient
Fecund means able to produce a lot of crops, fruit, babies, young (c) Perjurious (d) Mendacious
animals, etc. (e) Marginal
Fertile means producing or capable of producing abundant Q265.
vegetation or crops. 1) Karen’s parents were serious people who did not seem to
Useless means having no ability or skill. appreciate her acts of _____ during church service.
Thus, both B and C make the sentence meaningful.
Thus option b is the correct answer.
Facebook Page- https://www.facebook.com/vishalpariharpage Youtube- https://www.youtube.com/vishalparihar
Follow
51 Vishal Sir Telegram Channel- https://t.me/englishbyvishalsirchannel Instagram- https://www.instagram.com/vishalthetrainer
By Vishal sir

SBI PO, SBI CLERK, IBPS PO, IBPS CLERK, RRB PO, RRB CLERK, NIACL,
- LIC RBI grade B, RBI ASSISTANT, & Other competitive Exams

2) As a student, I really liked Mrs. Walters because she was not too C) a publication containing a variety of works
stern and always used ______ in her lessons to make the class less D) a pressing or urgent situation
formal. E) Mockery
(a) Aristocracy (b) Levity Thus option B is the word that fits in both the sentences.
(c) Compendium (d) Exigency
(e) Derision Directions (266-270): The Following questions have two blanks, each
Answers And Solutions blank indicating that something has been omitted. Choose the
set of words for each blank that best fits the meaning of the
261Ans.(d) sentence as a whole.
The meaning of the words is as follows:
A) awkward and somewhat incompetent Q266. Much of the public discussion on the ill-effects of crop ................
B) an illness burning ignores the immediately affected ..................... population.
C) the disgrace incurred by conduct considered outrageously (a) grown, microbial (b) surplus, hispanic
shameful (c) residue, vulnerable (d) propagate, male
D) a wealthy and influential business person (e) indigenous, homogeneous
E) very great or intense; thoughtful
Thus option D is the word that fits in both the sentences. Q267. In a/an .................. search last week, the ED seized a large number
262Ans.(e) of documents ................ to the financial transactions of the bank
'Actuarial' means relating to actuaries or their work of compiling during Ms. Sharma’s tenure.
and analysing statistics to calculate insurance risks and premiums. (a) perfunctory, echoing
263Ans.(b) (b) exhaustive, pertaining
To pronounce means to declare, to authorize formally, comment or (c) appendage, declining
give up. And the synonym comment has been used in the second (d) unvaried, promising
sentence. Condemned means to say strongly that you think (e) kindred, leading
somebody/something is very bad or wrong
Q268. Suresh Borkar’s poems that he wrote during the 228 years of his
264Ans.(b) ................... became a mirror to his ..................... soul.
In first sentence, the blank describes the role played by the kind of (a) eviction, demarcated
evidence that would justify a decision to continue the investigation, (b) realisation, directed
so a word that means something like important would make sense. (c) accretion, evicted
Both negligible and marginal are nearly the opposite of what you’re (d) incarceration, reformed
looking for, so eliminate choices A. and E. Choices C. and D. give (e) innovation, rejected
Q269. The airline has been forced to cancel several flights due to
roughly synonymous
meanings as well, but nothing in the sentence supports the ……...…. of planes because of non-payment of lease ……… to
description of the role as dishonest. Salient means important. So aircraft lessors.
choice B. is appropriate. Similarly In second sentence, the blank (a) augur, exaction (b) presage, tariffs
suggests something main, something important, so Salient fits the (c) prognosticating, rates (d) review, reassess
best. (e) grounding, rentals
265Ans.(b)
Q270. The petitioners ………….. that the leeway offered by the
The second sentence conveys that Mrs. Walters was not too stern
means that she was lenient and not too serious while taking the class subordinate laws would further multiply the ………….. influx of
and made it less formal. The word in the blank should be similar to illegal migrants from Bangladesh to Assam.
lenient or a similar attribute. (a) slapped, subjected
The meaning of the words is as follows: (b) contended, uncontrolled
A) the highest class in certain societies, typically comprising (c) targeted, recorded
people of noble birth holding hereditary titles and offices.
(d) rolled, assaulted
B) a lack of seriousness
(e) accosted, shared
Facebook Page- https://www.facebook.com/vishalpariharpage Youtube- https://www.youtube.com/vishalparihar
Follow
52 Vishal Sir Telegram Channel- https://t.me/englishbyvishalsirchannel Instagram- https://www.instagram.com/vishalthetrainer
By Vishal sir

SBI PO, SBI CLERK, IBPS PO, IBPS CLERK, RRB PO, RRB CLERK, NIACL,
- LIC RBI grade B, RBI ASSISTANT, & Other competitive Exams

Answers And Solutions 270Ans.(b)


Sol. Here, 'contended' and 'uncontrolled' perfectly fit in the given
266Ans.(c)
Here, 'residue' and 'vulnerable' perfectly fit in the given blanks. blanks.
residue- a small amount of something that remains after the main contended- struggle to surmount (a difficulty).
part has gone or been taken or used. assaulted- make a physical attack on.
vulnerable- exposed to the possibility of being attacked or harmed, accosted- approach and address (someone) boldly or aggressively.
either physically or emotionally. Hence, option (b) is the correct answer.
microbial- relating to or characteristic of a microorganism,
especially a bacterium causing disease or fermentation.
Directions (271-275): In each of these questions, there is a blank
hispanic- relating to Spain or to Spanish-speaking countries,
indicating that something has been omitted. There are five
especially those of Central and South America.
alternatives given. Choose the one that best fits the meaning of
propagate- spread and promote (an idea, theory, etc.) widely.
the sentence or sentences as a whole.
indigenous- originating or occurring naturally in a particular place;
native.
Q271. ____________ by circumstances, Arun felt desperate thinking
homogeneous- of the same kind; alike.
about his fractured career prospects.
Hence, option (c) is the correct answer.
267Ans.(b) (a) Held (b) Thwarted
(c) Borne (d) Hopeless
Here, 'exhaustive' and 'pertaining' perfectly fit in the given blanks.
(e) Happy
exhaustive- including or considering all elements or aspects; fully
.
comprehensive.
pertaining - belong to something as a part, appendage, or accessory. Q272. If the means to make ____________ use of solar energy could be
appendage- a thing that is added or attached to something larger or found, it would reduce our dependence on non-renewable sources
more important. of energy.
perfunctory- (of an action) carried out without real interest, feeling, (a) right (b) good
or effort. (c) greater (d) efficient
unvaried- not involving change.kindred- similar in kind; related. (e) good
Hence, option (b) is the correct answer.
268Ans.(d) Q273. A land of many ____________, India can enchant visitors to it like
Sol. Here, 'incarceration' and 'reformed' perfectly fit in the given no other country can.
blanks. (a) colours (b) shades
incarceration- the state of being confined in prison; imprisonment. (c) hues (d) thrills
reformed- having been changed in such a way as to be improved. (e) cravings
accretion- growth or increase by the gradual accumulation of
additional layers or matter. Q274. Charlie Chaplin could create an amazing kind of comedy seeped in
demarcated - set the boundaries or limits of. ____________ that drew tears even as it evoked laughter.
Hence, option (d) is the correct answer. (a) triviality
269Ans.(e) (b) seriousness
Here, 'grounding' and 'rentals' perfectly fit in the given blanks.
(c) pathos
prognosticating- foretell or prophesy (a future event).
grounding- prohibit or prevent (a pilot or an aircraft) from flying. (d) bathos
presage- be a sign or warning of (an imminent event, typically an (e) fun
unwelcome one).
tariffs- a tax or duty to be paid on a particular class of imports or Q275. The movements of the waters ____________ a memory in her of a
exports. distant past which, nevertheless, felt near.
augur-- (of an event or circumstance) portend a good or bad (a) stoked (b) evoked
outcome.
(c) drew (d) stirred
Hence, option (e) is the correct answer.

Facebook Page- https://www.facebook.com/vishalpariharpage Youtube- https://www.youtube.com/vishalparihar


Follow
53 Vishal Sir Telegram Channel- https://t.me/englishbyvishalsirchannel Instagram- https://www.instagram.com/vishalthetrainer
By Vishal sir

SBI PO, SBI CLERK, IBPS PO, IBPS CLERK, RRB PO, RRB CLERK, NIACL,
- LIC RBI grade B, RBI ASSISTANT, & Other competitive Exams

(e) called (a) A-B (b) B-D


Answers And Solutions (c) A-C
(e) B-C
(d) A-D

Q279. After looking at the month’s revenue, the storeowner was happy
271Ans.(b) because the sizable profit appeared to ______ a good start for his
Thwart means prevent (someone) from accomplishing something new business.
272Ans.(d) (A) Receded (B) Augur
Efficient suits the most hence option (d) is the correct choice. (C) Promise (D) Instill
273Ans.(a) (a) A-D (b) B-C
Enchant means fill (someone) with great delight; charm. Hence (c) C-D (d) A-B
option (a) is the correct choice. (e) B-D
274Ans.(c)
Pathos means a quality that evokes pity or sadness and evoked Q280. The lab results _________ a high amount of cholesterol as the
means bring or recall (a feeling, memory, or image) to the cause of my uncle’s heart problem.
(A) comply (B) Gratify
conscious mind.
(C) Implicate (D) Involve
275Ans.(a) (a) A-C (b) B-D
Stoked means excited or euphoric. Hence option (a) is the correct (c) B-C (d) A-B
choice (e) C-D
Answers And Solutions
Directions (276-280): In each of the questions given below, a sentence
276Ans.(b)
is given with one blank. Below each sentence, FOUR words are
Option (b) is the correct choice as both Tangible and palpable fit
given out of which two can fit into the sentence. Five options
the blank appropriately.
are provided with various combinations of these words. You
Tangible means perceptible by touch.
have to choose the combination with the correct set of words
Palpable means (of a feeling or atmosphere) so intense as to seem
which can coherently fit into the given sentence.
almost tangible.
Mystic means inspiring a sense of spiritual mystery, awe, and
Q276. Although Jack had a good business plan, the bank was unwilling to
fascination.
give him a loan because he lacked __________ assets. Spectacular means beautiful in a dramatic and eye-catching way.
(A) Tangible (B) Mystic
Combination A-D is correct as it is adding the meaning that the
(C) spectacular (D) palpable
anger is so significant that the ruling class is terrified.
(a) A-B (b) A-D 277Ans.(d)
(c) B-C (d) B-D
Option (d) is the correct choice for the given question.
(e) A-C Allude means suggest or call attention to indirectly; hint at.
Wrest means forcibly pull (something) from a person's grasp.
Q277. Sally did not give up her phone easily as her father had to ______
Seize means take hold of suddenly and forcibly.
it from her hands. Smear damage the reputation of (someone) by false accusations;
(A) smear (B) Seize
slander.
(C) Wrest (D) Allude
Combination (B-C) is correct as these words are imparting the
(a) A-B (b) B-D
meaning that control is being taken away from the current directors.
(c) A-D (d) B-C
Other words are not contextually correct.
(e) C-D 278Ans.(d)
Option (d) is the correct choice for the given question.
Q278. The boy would ___________ his father's morning routine, from
Insightful means having or showing an accurate and deep
reading the newspaper to sipping coffee.
understanding; perceptive.
(A) insightful (B) ferocious Incisive means (of a person or mental process) intelligently
(C) emulate (D) Incisive
analytical and clear-thinking.

Facebook Page- https://www.facebook.com/vishalpariharpage Youtube- https://www.youtube.com/vishalparihar


Follow
54 Vishal Sir Telegram Channel- https://t.me/englishbyvishalsirchannel Instagram- https://www.instagram.com/vishalthetrainer
By Vishal sir

SBI PO, SBI CLERK, IBPS PO, IBPS CLERK, RRB PO, RRB CLERK, NIACL,
- LIC RBI grade B, RBI ASSISTANT, & Other competitive Exams

Emulate means match or surpass (a person or achievement), Q283. The situation in Syria, where so many proxies of so many nations
typically by imitation. are fighting each other, is very __________.
Ferocious means savagely fierce, cruel, or violent. (A) Incessant (B) Complicated
279Ans.(b)
(C) Constant (D) Perplexing
Option (b) is the correct choice for the given question.
Augur means (of an event or circumstance) portend a good or bad (a) B-D (b) A-D
outcome. (c) A-B (d) A-C
Instill means gradually but firmly establish (an idea or attitude) in (e) B-C
a person's mind. Q284. Earlier, people used to think that clearing the English section of the
Receded means go or move back or further away from a previous SSC CGL Mains exam was __________, but it isn’t as confirmed
position. by the low cut-off marks for the English section.
(A) Easy (B) Exacting
280Ans.(e) (C) Formidable (D) Facile
Option (e) is the correct choice for the given question. (a) A-B (b) B-C (c) A-D \
C-D is the correct word combination. (d) A-C (e) B-D
Implicate means to hint a person or object is responsible for
something, often an illegal deed Q285. The family thought it was more __________ to eat in the kitchen.
Gratify means give (someone) pleasure or satisfaction. (A) Perpetual (B) Apt (C) Opportune (D) Fore
Involve means have or include (something) as a necessary or (a) B-C (b) A-B (c) B-D
integral part or result. (d) A-C (e) C-D
Comply means act in accordance with a wish or command. Answers And Solutions
Implicate and Involve are similar in meaning and fits the blank
281Ans.(d)
appropriately.
Titbit [noun] and Appetizer [noun] are synonyms which means ‘the
first course of a meal. It consists of a small amount of food.
Directions (281-285): In each of the questions given below a sentence Bite means an act of biting something in order to eat it.
is given with one blank. Below each sentence FOUR words are Sweetness means the quality of being sweet.
given out of which two can fit the sentence. Five options are Among the given options, options (A) and (C) are relevant to the
given with various combinations of these words. You must context of the sentence.
Hence, the option (d) is the correct answer.
choose the combination with the correct set of words which can
fit in the given sentence. 282Ans.(b)
Q281. At 6.30 everyone gathered for drinks and __________ in the Flay [verb] means ‘to strip the skin off’;
lounge.
Drub [verb] means ‘defeat, beating or thrashing’;
(A) Titbit (B) Bite
(C) Appetizer (D) Sweetness Testimony [noun] means ‘confirmation, proof etc.’
(a) A-B (b) B-C Among the given options, options (B) and (C) satisfy the context
(c) A-D (d) A-C of the sentence.
(e) B-D Hence, the option (b) is the correct answer.
Q282. They took her resignation from TCS as ___________ of their 283Ans.(a)
suspicions. The hint for the phrase is given by the subordinate clause ‘where
(A) Flaying (B) Confirmation so many proxies of so many nations are fighting each other’. The
(C) Testimony (D) Drubbing correct adjective which could correctly qualify the noun ‘situation’
(a) C-D (b) B-C is ‘complicated’ and ‘perplexing’.
(c) A-D (d) A-C The correct answer is the option (a).
(e) B-D 284Ans.(c)
Facile [adjective] and easy [adjective] are synonyms.

Facebook Page- https://www.facebook.com/vishalpariharpage Youtube- https://www.youtube.com/vishalparihar


Follow
55 Vishal Sir Telegram Channel- https://t.me/englishbyvishalsirchannel Instagram- https://www.instagram.com/vishalthetrainer
By Vishal sir

SBI PO, SBI CLERK, IBPS PO, IBPS CLERK, RRB PO, RRB CLERK, NIACL,
- LIC RBI grade B, RBI ASSISTANT, & Other competitive Exams

Exacting [adjective] and formidable [adjective] are too synonyms, (d) situations, hold
but they mean ‘to be difficult’. (e) systems, environment
The first part of the sentence before the conjunction ‘but’ is
showing a contrast to the second part of the sentence after the Q289. We need to have proper laws to ensure scientific _____ of our land
conjunction ‘but’. Having ‘a low cut-off marks for the English mass for a variety of ______.
section’ means that the English section of the SSC CGL Mains a) treatment, yield b) cure, appeal
exam isn’t easy. The adjective to be used in the first part of the c) experiment, genre d) planning, activities
sentence, when the first part of the sentence is in contradiction to e) norms, avenues
the second part of the sentence, is ‘easy’.
Among the given options, the options (A) and (D) are the most Q290. The highly ____ goals of literacy achievement have been ____ by
appropriate. Hence, the option (c) is the correct answer. the State-run schools so far.
285Ans.(a) (a) targeted, borne (b) appreciated, strengthened
Perpetual [adjective] means ‘continuing without pause or (c) ambitious, shouldered (d) debated, raised
interruption’; (e) demanding, forward
Fore [adjective] means ‘situated or place in front’; Answers And Solutions
Apt[adjective] means ‘suitable or convenient’; 286Ans.(b)
Opportune [adjective] means ‘convenient’;
Procurement- the act of obtaining goods or services
From above, we find that only two words, which are also
Enforcement- the act of compelling observance of or compliance
synonyms, ‘apt’ and ‘opportune’ have a meaning and grammatical
with a law, rule, or obligation.
form which is appropriate to fill the blank.
Avid- having or showing a keen interest in or enthusiasm for
Hence, the correct answer is the option (a).
something.
Opaque- not able to be seen through; not transparent.
Directions (286 – 290): In each of the following sentences there are two Deciphering- convert (a text written in code, or a coded signal) into
blank spaces. Below each sentence there are five pairs of words normal language.
denoted by numbers A), B), C), D) and E). Find out which pair 287Ans.(e)
of words can be filled up in the blanks in the sentence in the Empaneled- enrol (someone) on to a jury.
same sequence to make it meaningfully complete. Authorized- having official permission or approval.
288Ans.(a)
Q286. An ____ law ____ system is essential for India to attain and sustain 289Ans.(d)
economic and social prosperity. Yield- produce or provide (a natural, agricultural, or industrial
(a) effective, procurement product).
(b) efficient, enforcement Cure- relieve (a person or animal) of the symptoms of a disease or
(c) avid, delivery condition.
(d) opaque, adhering Genre- relieve (a person or animal) of the symptoms of a disease
(e) outstanding, deciphering or condition.
Avenues- a broad road in a town or city, typically having trees at
Q.287. ______ of crimes to agencies _____ by the state is an indication of regular intervals along its sides.
the maturity of a society. 290Ans.(c)
a) Informing, held b) Dealing, sentenced Borne- carried or transported by the thing specified.
c) Alerting, empowered d) Monitoring, empaneled Ambitious- having or showing a strong desire and determination to
e) Reporting, authorized succeed.
Shouldered- (of a person or garment) having shoulders of a
Q288. Cropping pattern should be decided by taking land quality and
other agro-climatic ____ into ____. specified kind.
(a) conditions, account
(b) spheres, domain Direction (291-295): In each of these questions, there are two blanks
(c) features, confidence indicating that something has been omitted. There are five

Facebook Page- https://www.facebook.com/vishalpariharpage Youtube- https://www.youtube.com/vishalparihar


Follow
56 Vishal Sir Telegram Channel- https://t.me/englishbyvishalsirchannel Instagram- https://www.instagram.com/vishalthetrainer
By Vishal sir

SBI PO, SBI CLERK, IBPS PO, IBPS CLERK, RRB PO, RRB CLERK, NIACL,
- LIC RBI grade B, RBI ASSISTANT, & Other competitive Exams

alternatives given. Choose the one that best fits the meaning of Thus, option b is the correct answer.
the sentence or sentences as a whole. Ans.(c)292

Q.291. I ________ miss my friends from school even ______ we weren’t In the above sentence, the filler has to be an adverb of time, as the
sentence describes the frequency with which people believe in
that close.
superstitions.
(a) Seldom, because (b) Often, though Moreover, the latter filler is to be an preposition since it will be
(c) Want to, when (d) Don’t, because explaining the relationship between the noun and the pronoun or
(e) Wont, if clause.
Let us know the meaning of some of the words:
Q.292 People are __________ superstitious ___________ numbers, Seldom means something or an event which doesn't occur
days and colors. frequently.
(a) Seldom, more (b) Enjoying, about Often means something or an event which occurs frequently.
(c) Often, about (d) Jealous, in Thus, option c is the correct answer.
(e) Stuck-up, between 293Ans.(a)
In the above sentence, the subject seems to be ordering for another
Q.293. The old man ___________ the waiter to bring ________ another round of drinks.
round of drinks. “Asked” is the most apt filler, as the other fillers do not sit in
a) Asked, in b) Enquired, around accordance with the connotation of the sentence. The preposition
c) Ordered, about d) Pleaded, on that will complete the phrase will be 'bring in' which means 'to get'.
e) Refused, in “Enquired” is used for asking and doesn’t match the meaning of the
clause, thus it is ruled out.
Q.294 We are going to meet tomorrow_________ dinner _________ my “Ordered” would be apt if the subject was ordering for the first
parents’ house. time, however the use of “another round of drinks” clearly indicates
(a) During, in (b) After, outside that it was the second order.
(c) For, at (d) Before, toward “Pleaded” doesn’t sit in accordance with the connotation, as diners
(e) At, near never generally plead their servers.
“Refused” doesn’t agree with the clause and is thus ruled out.
Q.295. I _________ taken my lunch and decided to ________ in a box of Let us know the meanings of some of the words given above:
old books and family photographs. Asked means to say something in order to obtain an answer or some
(a) Have, look (b) Had, rummage information.
(c) Has, skimmed (d) Want, search Enquired means to ask for information from someone.
(e) Already, saw Ordered means to give an authoritative instruction to do something.
Pleaded means to make an emotional appeal.
Answers And Solutions Refused means to indicate or show that one is not willing to do
291Ans.(b) something.
The above sentence, carries a sense of nostalgia and reminiscence. Thus, option a is the correct answer.
“Seldom” is an adverb of time which doesn’t fit the connotation of 294Ans.(c)
the sentence and is thereby ruled out. In the above sentence, the subjects are talking about meeting
someone at dinner time.
“Often” sits in with the meaning the sentence wishes to express
The first filler, is to be a preposition as it would show the
which is the frequency with which the subject thinks about her relationship between the nouns and the clause.
friends back at school. The latter filler is to be a preposition of place.
“Though” is the only conjunction that sits with the adverb “even”. Thus, option c is the correct answer.
Let us know the meanings of some of the words given above: 295Ans.(b)
Seldom means something or an event that doesn't occur frequently. In the above sentence, the first filler will be “have” in accordance
Often means something or an event that occurs quiet frequently. with the rules of present perfect.

Facebook Page- https://www.facebook.com/vishalpariharpage Youtube- https://www.youtube.com/vishalparihar


Follow
57 Vishal Sir Telegram Channel- https://t.me/englishbyvishalsirchannel Instagram- https://www.instagram.com/vishalthetrainer
By Vishal sir

SBI PO, SBI CLERK, IBPS PO, IBPS CLERK, RRB PO, RRB CLERK, NIACL,
- LIC RBI grade B, RBI ASSISTANT, & Other competitive Exams

The second filler would be a verb which would mean to look Option (c) is the appropriate answer.
through something without the intention of finding something . Here Enchant means delightfully charming or attractive, is the
Let us know the meanings of some of the words given above: correct word.
Rummage means to search unsystematically and untidily through Deactivate means make (something) inactive by disconnecting or
something. destroying it.
Skimmed means to remove a substance from the surface of a liquid. 297Ans.(c)
Thus, option b is the correct answer. Option (c) is the most apt choice to be filled in the blank. Here word
desired is wrong in terms of context as the word that will be used
instead of desired should be ‘expected.’
Directions (296-300): In the following questions a sentence is given
with a blank and bold word. Against each question five Peeling means removing the outer covering or skin from (a fruit,
alternatives are provided; reflecting the word that should fill vegetable, or prawn).
the blank and conveying whether the word in bold is Agreeable means willing to agree to something. Expected means
contextually correct or incorrect. Choose the best option as regard (something) as likely to happen.
298Ans.(d)
your answer.
Option (d) is the most suitable choice to be filled in the blank. Here
treat means an event or item that is out of the ordinary and gives
Q296. A land of many ____________, India can enchant visitors to it like
great pleasure, is the correct word. Probable means likely to happen
no other country can.
or be the case.
(a) shades, incorrect (b) hues, incorrect 299Ans.(d)
(c) colors, correct (d) thrills, correct
Option (d) is the most suitable choice to be filled in the blank. And
(e) cravings, incorrect the usage of word addressed is correct contextually.
Q297. Moving house is often thought of as an ____________ exercise but ‘compounded — addressed’ is the correct use where ‘compounded’
it does not always turn out as desired. means make (something bad) worse; intensify the negative aspects
of and ‘addressed’ means to fix or solve an issue.
(a) Peeling, incorrect (b) exciting, correct 300Ans.(b)
(c) agreeable, incorrect (d) irritating, correct Option (b) is the most appropriate choice to be filled in the blank.
(e) exhausting, correct And the usage of word familiar is inappropriate here. Instead
flamboyant has to be used. Nuances- a subtle difference in or
Q298. The children explored the ____________ pleasures of the garden shade of meaning, expression, or sound
which, lush and colourful, was a treat to the sight. Accustomed means customary; usual. Flamboyant means (of a
(a) little, incorrect (b) mutant, incorrect person or their behaviour) tending to attract attention because of
(c) thorough, correct (d)probable, correct their exuberance, confidence, and stylishness.
(e)Published, correct
Direction (301-305): Each sentence below has two blanks, each blank
Q299. The problem of housing shortage _______ with the population indicating that something has been omitted. Choose the words
explosion has also been addressed by this policy. that best fit the meaning of the sentence as a whole.
(a) highlighted, incorrect (b) coped, incorrect Q301. The _____________ condition of her family was evident from her
(c) discussed, incorrect (d) compounded, correct _____________ nature.
(e) projected, incorrect (a) Corpulent, Dilatory (b) Execrable, Dilatory
(c) Bellicose, Corpulent (d) Bellicose, Dilatory
Q300. As our eyes become ____________ to the dazzle, we could clearly (e) Execrable, Bellicose
Q302. The once _____________ boy became __________________ on
make out the familiar decor and elaborate arrangements.
being cheated by his closest friends.
(a) Contrition, correct (b) accustomed, incorrect
(a) Effulgent, Arcadian (b) Effulgent, Contumacious
(c) Dispute, correct (d) Nuances, correct
(c) Guileless, Contumacious. (d) Guileless, Effulgent
(e) restless, correct
(e) Contumacious, Arcadian
Answers And Solutions
296Ans.(c)

Facebook Page- https://www.facebook.com/vishalpariharpage Youtube- https://www.youtube.com/vishalparihar


Follow
58 Vishal Sir Telegram Channel- https://t.me/englishbyvishalsirchannel Instagram- https://www.instagram.com/vishalthetrainer
By Vishal sir

SBI PO, SBI CLERK, IBPS PO, IBPS CLERK, RRB PO, RRB CLERK, NIACL,
- LIC RBI grade B, RBI ASSISTANT, & Other competitive Exams

Q303. Before she faced her _____________ mother-in-law at dinner, she has a scathing wit. Hence the correct answer is option D
had to brace herself for her ____________ comments about her
cooking. 304Ans.(e)
(a) Jocular, Histrionic (b) Mordant, Histrionic The meanings of the words are as follows:
(c) Mordant, Jocular (d) Mordant, Abrasive Limpid means simple.
(e) Histrionic, Abrasive Nefarious means wicked.
Q304. His ________________ nature of giving cheap gifts to people Parsimonious means frugal.
made him ___________ to all. Irksome means annoying.
(a) Limpid, Nefarious (b) Parsimonious, Nefarious The phrase "cheap gifts" directly reflects the use of word
(c) Nefarious, Irksome (d) Limpid, Parsimonious "parsimonious" in the first blank following the use of word
(e) Parsimonious, Irksome "irksome" in the second blank which means annoying. Hence the
Q305. The famous artist released a new album after a five year of correct answer is option E
______________, though the album was yet ___________. 305Ans.(b)
(a) Tenet, Torpor (b) Torpor, Inchoate The meanings of the words are as follows:
(c) Tenet, Perusal (d) Palliative,Perusal Tenet means a rule or law.
(e) Palliative, Inchoate Torpor means a state of being inactive.
Inchoate means beginning to develop or not properly formed.
Answers And Solutions Perusal means examination.
301Ans.(e) Palliative means remedy.
The meanings of the words are as follows: Hence the correct answer is option B which fits into the blanks both
Corpulent means obese.
Dilatory means causing delay. grammatically and contextually.
Execrable means wretched, detestable.
Bellicose means quarrelsome. Directions (306-310): Each sentence below has a blank. Each blank
The context of the sentence talks about the condition of the family indicates that something has been omitted. Choose the
which seems to be unpleasant if we go with the contextual meaning. word/group of words that best fit/s the meaning of the sentence
Thus, "execrable" is the appropriate word to fit in the blank as a whole.
followed by "bellicose" in the second one. Hence the correct
answer is option E Q306. When I was training for the marathon, I ______ run over 500
302Ans.(c) kilometers a week.
The meanings of the words are as follows: (a) have to (b) would
Effulgent means brilliantly radiant. (c) will (d) destined
Arcadian means serene.
(e) use to
Contumacious means rebellious.
Guileless means naive.
The context is all about the consequences followed by an event Q307. My batch mate is one of the kindest people _______.
which seems to reflect a negative tone. Thus, "guileless" which best (a) that I knows (b) I know
preceded the word "boy" is the most suitable filler following by the (c) who I know (d) which I know
word "contumacious". Hence the correct answer is option C (e) I had known
303Ans.(d)
The meanings of the words are as follows: Q308. The lawyer's ________ led to the resolution of the problem.
Jocular means playful. (a) behaviour (b) fees
Histrionic means affected. (c) advice (d) impact
Mordant means having a scathing wit. (e) approval
Abrasive means harsh.
The context is all about characterizing a person, .i.e., mother-in-law
who has been called as the one with a scathing wit. Thus, the person Q309. The government claims that _________ in the Automobile industry
(referred as "she" in the context) has to prepare herself for the will mean lower prices for customers.
unpleasant situation where she has to be ready for the harsh (a) budget (b) finance (c) installments
comments on her cooking before facing her mother-in-law who (d) decrease (e) competition
Facebook Page- https://www.facebook.com/vishalpariharpage Youtube- https://www.youtube.com/vishalparihar
Follow
59 Vishal Sir Telegram Channel- https://t.me/englishbyvishalsirchannel Instagram- https://www.instagram.com/vishalthetrainer
By Vishal sir

SBI PO, SBI CLERK, IBPS PO, IBPS CLERK, RRB PO, RRB CLERK, NIACL,
- LIC RBI grade B, RBI ASSISTANT, & Other competitive Exams

Q310. Samaya was walking along the street ________ she tripped over. (a) confirm, indication (b) raise, real
(a) when (b) as (c) while (c) hear, smart (d) investing, upskilling
(d) then (e) however (e) none of these
Answers And Solutins Answers And Solutions
306Ans.(b) 311Ans.(a)
307Ans.(b) Only option A fills both the blanks appropriately making it both
308Ans.(c) grammatically as well as contextually correct. Other words like
309Ans.(e) “allows”, “calling” and “maintains” are not grammatically fit
310Ans.(a) because of the presence of “be” before the blank 1.
Hence, option A is the best suited answer
312Ans.(d)
Direction (311-315): A sentence with two blanks is given, each blank Galling means ‘causing annoyance or resentment; annoying’ which
indicating that something has been omitted. Choose the pair of seems to be filling up the blank appropriately.
words that best fits in the sentence. Implementing means put (a decision, plan, agreement, etc.) into
effect. Thus, it will get eliminated because of no contextual sense.
Appalled means shocked but this is grammatically incorrect and
Q311. This needs to be __________ in classification, otherwise the
hence, will get eliminated.
arbitrariness with which the classification is made somewhat hints Also, “knowing” in option A doesn’t fit into the blank.
at a __________ designation of a few States in two extremes and a Hence, option D would be the correct response as per the choice.
major share of them in between. 313Ans.()
(a) recognised, purposive (b) allows, purposeful For the first blank, the indicator is 'seven week low'. Announcement
(c) calling, desirous (d) maintains, acquiring refers to a formal statement given by someone.
(e) none of these 314Ans.(d)
. 315Ans.(d)
Hence, option D is the most suited response.
Q312. What was the most _________ was the Central government’s
apathy in the face of this brazen act of mob violence despite the fact
Directions (316-320): Each question below has one blank, which is
that it had been _________.
indicating that something has been omitted. Find out which
(a) knowing, forward (b) implementing, forlorn
(c) appalled, foregone (d) galling, forewarned option can be used to fill up the blank in the sentence in the
(e) none of these same sequence to make it meaningfully complete.

Q313. Stocks slid sharply in afternoon trade and the rupee ________ to a Q316. Unless management accepts mediation, the strike will never
seven-week low after the Indian Army’s ________ that it had be __________.
destroyed Pakistani posts heightened investor fears of border (a)Resolved (b)Concludes
tensions running out of control. (c)Resolves (d)Commenced
(a) fell, announcement (b) tumbled, notice
(e)Passed
(c) rose, procession (d) stuck, challenge
(e) stuck, Possession
Q314. Increased interest has recently __________on identifying and Q317. Because he refused to listen to others, everyone considered the
understanding the specific role of the intestinal microbiota in politician to be too ________.
various __________ diseases. (a)Tentative (b)Hasten
(a) arrangements, symbols (b) changes, examples (c)Unlikely (d)Provided
(c) alterations, blueprints (d) focused, metabolic (e)Dogmatic
(e) none of these
Q318. Many hours of __________ preparation have gone into writing the
Q315. California-based digital transformation solutions company UST book.
said it has been __________in building a future-ready workforce, (a)Careless (b)Slapdash
with a focus on __________and retraining talent. (c)Meticulous (d)Sloppy

Facebook Page- https://www.facebook.com/vishalpariharpage Youtube- https://www.youtube.com/vishalparihar


Follow
60 Vishal Sir Telegram Channel- https://t.me/englishbyvishalsirchannel Instagram- https://www.instagram.com/vishalthetrainer
By Vishal sir

SBI PO, SBI CLERK, IBPS PO, IBPS CLERK, RRB PO, RRB CLERK, NIACL,
- LIC RBI grade B, RBI ASSISTANT, & Other competitive Exams

(e)Vexed Q321. The officer refused to …….. any more elusive replies and decided
to reconcile the case within the …….. of the city.
Q319. The jury must take into account any __________ circumstances (a) notice, eccentricity (b) brook, jurisdiction
presented by the defense, such as previous good character. (c) bloom, indulgence (d) suffer, fortitude
(a)Sharp (b)Intensify (e) allow, fortune
(c)Mitigating (d)Increase Q322. Chanting and praying are _______the most popular religious
(e)Aggravate activities, which are said to be able to __________people’s
negative emotions.
Q320. The crowd was so well - behaved that the police presence (a) However, represent (b) among, alleviate
was ____________. (c) Moreover, reflect (d) In addition, convey
(a)Obtainable (b)Indispensable (e) Hence, allow
(c)Wanted (d)Superfluous
(e)Essential Q323. After listening to the pleas for ………., the judge was ……….. and
Answers And Solutions gave the convicted man the maximum punishment allowed by law.
(a) activity, abrasive (b) fortitude, formidable
316Ans.(a) (c) clemency, unmoved (d) multiplicity, noisome
Sol. The appropriate word to be used here is ‘Resolved’. (e) adequacy, angry
‘Concludes’ ‘resolves’ are irrelevant as they are different part of
speech and cannot be used here. Commencing means beginning. Q324. First, women were probably regarded as more hopelessly
Resolved- firmly determined to do something. _________, more totally __________when fallen.
317Ans.(e) (a) congenial, creditable (b) incorrigible, irredeemable
The appropriate word to be used here is ‘Dogmatic.’ (c) convincing, illogical (d) blatant, incongruous
Dogmatic- inclined to lay down principles as undeniably true (e) intelligent, correct
All other given options don’t fit contextually.
318Ans.(c) Q325. Even though the city was prone to natural calamities, the ………
The appropriate word to be used here is ‘Meticulous.’ of the flood last year was ……… .
Meticulous- showing great attention to detail; very careful and (a) levity, lethal (b) portent, poignant
precise, option (a) (b) and (d) are opposite of the correct choice (c) inevitability, mercurial (d) ferocity, unprecedented
‘meticulous.’ (e) variety, decent
Vexed- annoy Answers And Solutions
319Ans.(c) 321Ans.(b)
The appropriate word to be used here is ‘Mitigating.’ To ‘brook’ is ‘to tolerate’ or ‘put up with’. ‘Suffer’ in option (d)
Mitigating- make (something bad) less severe, serious, or painful, will suit the first blank but ‘fortitude’ will not fit the second blank.
all other options except (c) are the opposite of the correct choice to 322Ans.(b) the correct answer is among and alleviate where alleviate
be filled i.e. ‘mitigating’ means reduce, covey means to carry
320Ans.(d) 323Ans.(c)
The appropriate word to be used here is ‘Superfluous.’ ‘Clemency’ is ‘mercy’; to be ‘unmoved’ is to be not affected by
Superfluous- unnecessary, especially through being more than emotion’.
enough, option (a) (b) (c) and (e) are opposite of the correct choice 324Ans.(b) congenial means compatible, creditable means admirable,
‘Superfluous.’ incorrigible means habitual, irredeemable means not able to
recover, blatant means obvious, incongruous means inappropriate
Directions (321-325) : In each of the sentences given in these questions, 325Ans.(d) levity means high in spirits, lethal means deadly, portent
there are two blanks indicating that some words have been means sign, poignant means touching, inevitability means certain
omitted. There are four alternatives given. Choose the one that to happen, mercurial means temperamental, ferocity means
best fits the meaning of the sentence as a whole. violently, unprecedented means unparalled

Facebook Page- https://www.facebook.com/vishalpariharpage Youtube- https://www.youtube.com/vishalparihar


Follow
61 Vishal Sir Telegram Channel- https://t.me/englishbyvishalsirchannel Instagram- https://www.instagram.com/vishalthetrainer
By Vishal sir

SBI PO, SBI CLERK, IBPS PO, IBPS CLERK, RRB PO, RRB CLERK, NIACL,
- LIC RBI grade B, RBI ASSISTANT, & Other competitive Exams

Directions (326-330): In each of the following questions, a sentence is Call back means to return to a place in order to see someone or
given with a blank. There are five phrases given below each collect something.
sentence, one out of which can be used in the blank to form a Chew up means cut into small pieces with your teeth.
meaningful sentence. Choose the most appropriate phrase 227Ans.(c)
among the five options that makes the sentence grammatically The correct phrase which makes the sentence meaningful is “eke
and contextually meaningful. out”. The phrasal verb “eke out” means to obtain or win something
only with difficulty or great effort. Hence option (c) is the correct
Q326. They ___________________ our objections and carried on with choice.
the plan to detain the accused. Fall out means (of an object) to drop from a place where it was
(a) add on attached or contained.
(b) waved aside Mix up means a mistake that causes confusion.
(c) agree with Move ahead means make progress, often after a pause or delay.
(d) called back Nose out means narrowly beat someone.
(e) chewed up 228Ans.(a)
The blank should be filled up with the phrase “abide by” to form a
Q327. Most students have to __________________ their income because meaningful sentence. The phrasal verb “abide by” means to accept
they have so little money to live on. or obey an agreement, decision, or rule. Hence option (a) is the
(a)fall out (b)mix up correct choice.
(c)eke out (d)move ahead Bring off means to succeed in doing something difficult.
(e)nose out Cast up means be left on the shore by the sea.
Dig in means to start eating.
Q328. We have to __________________ what the court says regarding Drop off means to begin to sleep.
the case.
(a)abide by (b)bring off 229Ans.(d)
(c)cast up (d)dig in The phrase “do up” gives a meaningful sense to the sentence. The
(e)drop off phrasal verb “do up” means repair and renovate. Hence option (d)
Q329. It took them six months to ___________________ the house before is the correct choice.
they could actually move in. Aim at means to produce something for a particular purpose or a
(a)aim at (b)freeze up particular group of people.
(c)go around (d)do up Freeze up means stop working because the parts of a machine won't
(e)head out move.
Go around means to be enough for everyone.
Q330. She will be back in a minute- she has just __________________ to Head out means go out.
the shops. 230Ans.(a)
(a)nipped out (b)noted down The phrase ‘nipped out” makes the sentence meaningfully correct.
(c)pass by (d)poke around The phrasal verb “nip out” means go somewhere quickly. Hence
(e)ran after option (a) is the correct choice.
Answers And solutions Note down means to write something so that you do not forget it.
Pass by means go past without stopping.
Poke around means move things around or search in a casual way
226Ans.(b) to try to find something.
The correct phrase which can make the sentence meaningful is Run after means chase, pursue.
“waved aside”. The phrasal verb “wave aside” means to refuse to
consider what someone says. Hence option (b) is the correct Directions (331-335): Each question below has two blanks. There are
choice. five pairs of words below the sentence. Each pair is lettered.
Add on means to include an extra thing or amount. Choose the pair of words which can be filled up in the blanks
Agree with means to think that something is morally acceptable. in the sentence in the same order so as to complete the sentences
meaningfully.
Facebook Page- https://www.facebook.com/vishalpariharpage Youtube- https://www.youtube.com/vishalparihar
Follow
62 Vishal Sir Telegram Channel- https://t.me/englishbyvishalsirchannel Instagram- https://www.instagram.com/vishalthetrainer
By Vishal sir

SBI PO, SBI CLERK, IBPS PO, IBPS CLERK, RRB PO, RRB CLERK, NIACL,
- LIC RBI grade B, RBI ASSISTANT, & Other competitive Exams

Q331. In that country, bureaucracy is …………. as a group of men and Engulfed- sweep over (something) so as to surround or cover it
women that had arrogated to itself power ………… responsibility. completely.
(a) perceived, without (b) believed, within 234Ans.(e)
(c) held, into (d) known, unto ‘affect, suffer’ is the correct use.
(e) allowed, for 235Ans.(d)
Q332. The employees of the factory owing ………….. to the workers ‘stipulate, remunerative’ is the correct use.
union, have …………. of an agitation from next week in support of Remunerative -financially rewarding.
their demands. Stipulate- demand or specify (a requirement), typically as part of
(a) allegiance, warned (b) respect, called an agreement.
(c) shelter, started (d) pressure, proposal
(e) assurance, sought Direction (336-340): A sentence with one blank is given, indicating
that something has been omitted. Choose the word that best fits
Q333. Yesterday, around 400 huts were …………… in a major fire which the blank appropriately.
……………. the slum area.
(a) burn, hit (b) gutted, engulfed Q336. Hunters wear camouflage clothing so that they appear less
(c) fired, took (d) demolished, entered __________ around the animals.
(e) smoldered, enmeshed (a) Diligent (b) Conspicuous
(c) Available (d) Accessible
Q334. The television boom might not …………… newspaper reader but (e) Persistent
magazines which are not leaders in their segment might …………..
(a) link, close (b) effect, prosper Q337. The most prevalent ___________ about a race of people usually
(c) reduce, encourage (d) attract, continue become stereotypes.
(e) affect, suffer (a) Beliefs (b) Casts
(c) Pins (d) Propensity
Q335. The rules of the institute ………….. that employees may undertake (e) Instant
any outside …………… work without seeking written permission Q338. Even under great pressure, the emergency room doctor was the
of the institute. __________ of professionalism.
(a) conclude, devoted (a) Concurrence (b) Jargon
(b) strengthen, assignment (c) Notion (d) Epitome
(c) allow, important (e) Apprehension
(d) stipulate, remunerative
(e) direct, extra Q339. Experts cite the country as an __________ where human rights
Answers And Solutions violations could lead to international intervention.
(a)Outcome (b) Instance
231Ans.(a) (c)Implication (d) Association
‘perceived, without’ is the correct use. (e) Alliance
Perceived- interpret or regard (someone or something) in a
particular way. Q340. They reported a sudden __________ of the disease in the south of
the country.
232Ans.(a) (a)Consequence (b)Humility
‘allegiance, warned’ is the correct use. (c)Application (d)Utilization
Allegiance -loyalty or commitment to a superior or to a group or (e)Outbreak
cause.
233Ans.(b) Answers And Solutions
‘gutted, engulfed’ is the correct use. 236Ans.(b)
Gutted- if a building is gutted, it is badly damaged or completely The sentence implies that the hunters wear disguising clothes to
destroyed. blend in with the surroundings so that they are not visible around
the animals.

Facebook Page- https://www.facebook.com/vishalpariharpage Youtube- https://www.youtube.com/vishalparihar


Follow
63 Vishal Sir Telegram Channel- https://t.me/englishbyvishalsirchannel Instagram- https://www.instagram.com/vishalthetrainer
By Vishal sir

SBI PO, SBI CLERK, IBPS PO, IBPS CLERK, RRB PO, RRB CLERK, NIACL,
- LIC RBI grade B, RBI ASSISTANT, & Other competitive Exams

Out of the given options, conspicuous meaning ‘clearly visible’ is (ii) We drove home in an uncomfortable silence, Grandma sensing
the correct fit. my ..................... mood.
Hence, option B is the correct answer. (a)utopian (b)resurgent
237Ans.(a) (c) saturnine (d)inevitable
Note that a “stereotype” is an over-generalized belief about a (e)ineludible
particular category of people. “Stereotypes” are generalized
because one assumes that the thoughts about a person, group of Q342. (i) There has been an extraordinary ...................... in the parties'
people or a particular thing(s) is true for each individual person in positions on Europe.
the category. So, among all the options, only “beliefs” fit in the (ii) The dramatic ........................ illustrates the extent to which the
black indicating that if certain beliefs about a race are held to be media's polling results are simply measuring the impact of their
true, they gradually become stereotypes. own coverage.
* Propensity- an inclination or natural tendency to behave in a (a) regime (b) reversal
particular way. (c) regimen (d) refuge
238Ans.(d) (e) extern
The sentence implies that the emergency room doctor was the
perfect example of professionalism who served his duty Q343. (i) A code is produced based upon the Product key and various
excellently. details ................. to your PC hardware.
Out of the given options, epitome meaning ‘a person or thing that (ii) As with any association, certain matters .................. to codes of
is a perfect example of a particular quality or type’ is the perfect fit. conduct and standards need to be addressed.
Therefore, option D is the correct answer. (a) Pertaining (b) influencing
* Jargon means special words or expressions used by a profession (c) suspending (d) challenging
or group that are difficult for others to understand. (e) lacking
* Apprehension means anxiety or fear that something bad or
unpleasant will happen. Q344. (i) The wounds were still too raw and painful for the .....................
239Ans.(b) bandages to be removed.
The sentence implies that the experts refer to this country as an (ii) Fuchsias come in wide varieties ranging from tall shrubs to very
evidence where human rights violations lead to international ..................... greenhouse specimens.
intervention. (a) Elaborate (b) fragile
Out of the given options, instance meaning ‘an example or single (c) dissent (d) gratitude
occurrence of something’ is the correct fit. (e) catechism
Therefore, option B is the correct fit.
240Ans.(b) Q345. (i) There is .................. among the security officials that there could
The sentence implies that a sudden occurrence of the disease was be a series of simultaneous bombings across the country.
reported in the south of the country. (ii) The ruins of this building are still the cause of much ...................
Out of the given options, outbreak meaning ‘a sudden occurrence as to their purpose.
of something unwelcome, such as war or disease’ is the correct fit. (a) Withdrawal (b) nomination
Therefore, option E is the correct fit. (c) speculation (d) collaboration
Humility means the quality of having a modest or low view of one's (e) decision
importance. Answers And Solutions
341Ans.(c)
Direction (341-345): There are two different sentences with a blank Here, option (b), ‘resurgent’ and option (d), ‘inevitable’ are out of
space in each sentence. Choose the word from the given options the context. Also, option (a), ‘utopian’ which refers to an idealistic
which fits appropriately in both the blanks adding a proper and state cannot be the correct answer for the given blanks. Something
logical meaning to the sentences. which is ‘ineludible’ is inescapable, which is why we can omit
option (e). Therefore, option (c), ‘saturnine’ perfectly fits in the
Q341. (i) My former roommate was a ...................... scholar who said very given blanks making them grammatically correct and contextually
little and smiled rarely. meaningful.

Facebook Page- https://www.facebook.com/vishalpariharpage Youtube- https://www.youtube.com/vishalparihar


Follow
64 Vishal Sir Telegram Channel- https://t.me/englishbyvishalsirchannel Instagram- https://www.instagram.com/vishalthetrainer
By Vishal sir

SBI PO, SBI CLERK, IBPS PO, IBPS CLERK, RRB PO, RRB CLERK, NIACL,
- LIC RBI grade B, RBI ASSISTANT, & Other competitive Exams

utopian- modelled on or aiming for a state in which everything is catechism- a summary of the principles of Christian religion in the
perfect; idealistic. form of questions and answers, used for religious instruction.
resurgent- increasing or reviving after a period of little activity, gratitude - the quality of being thankful; readiness to show
popularity, or occurrence. appreciation for and to return kindness.
saturnine- (of a person or their manner) gloomy. Hence, option (b) is the correct answer.
inevitable- certain to happen; unavoidable. 345Ans.(c)
ineludible- unavoidable; inescapable. Sol. 'Speculation' fits in perfectly in the given blanks. Here, both
Hence, option (c) is the correct answer. the statements mention a guesswork/ opinion but without any
342Ans.(b) compact evidences. Option (b) ‘nomination’ and option (a)
Sol. Here, option (a) ‘regime’ and option (c), ‘regimen’ are out of ‘withdrawal’ can be omitted as they are out of context. Also,
the context and can therefore be omitted. Also, option (e), ‘extern’ collaboration and decision do not make a grammatically correct
related to person who is a non resident but a worker at an statement. Therefore, option (c) ‘speculation’ will perfectly fit in
institution/ organization and is thus cannot be the answer. the blanks making them grammatically correct and contextually
Therefore, option (b), ‘reversal’ perfectly fits in the given blanks meaningful.
making them grammatically correct and contextually meaningful.
regime - a government, especially an authoritarian one. withdrawal- the action of withdrawing something.
reversal- a change to an opposite direction, position, or course of nomination-- the action of nominating or state of being nominated.
action. speculation- the forming of a theory or conjecture without firm
regimen- a system of government. evidence.
refuge- a place or situation providing safety or shelter. collaboration- the action of working with someone to produce
extern - a person working in but not living in an institution, such as something.
a non-resident doctor or other worker in a hospital. decision- a conclusion or resolution reached after consideration.
Hence, option (b) is the correct answer. Hence, option (c) is the correct answer.
343Ans.(a)
Here, we can omit ‘challenging’ and ‘influencing’ as they do not
carry preposition ‘to’ with them. Also, ‘lacking’ and ‘suspending’ Direction (346-350): The following question carries a sentence with a
do not make the statement contextually correct. Therefore, option blank. Choose the most suitable word from the given options
(a), ‘pertaining’ perfectly fits in the given blanks making them that would make the sentence meaningful and complete.
grammatically correct and contextually meaningful.
pertaining- be appropriate, related, or applicable to. Q346. Every state should have its particular rescue force at their disposal
influencing - have an influence on.
as ____________ occur without signalling a warning.
suspending- temporarily prevent from continuing or being in force
(a) Tragedy (b) accident
or effect.
(c) misfortune (d) trouble
challenging- testing one's abilities; demanding.
(e) disasters
lacking - not available or in short supply. Q347. Direction: The following question carries a sentence with two
Hence, option (a) is the correct answer.
344Ans.(b) blanks. Choose the most suitable pair of words from the given
options that would make the sentence meaningful and complete.
Sol. Here, ‘catechism’, ‘dissent’ and ‘gratitude’ are completely out
The volunteers will not only raise ________ but also help these
of the context. Also, ‘elaborate’ doesn’t makes the correct sense in
families with the application process.
the statement (i). Therefore, option (b), ‘fragile’ perfectly fits in the
(a) Alertness (b) awkwardness
given blanks making them grammatically correct and contextually
(c) awareness (d) defence
meaningful.
(e) stiffness
Elaborate- involving many carefully arranged parts or details;
detailed and complicated in design and planning. Q348. ________ hearing the news he felt dejected because his hard work
fragile- (of an object) easily broken or damaged.
had not borne fruit.
dissent - the holding or expression of opinions at variance with
(a) After (b) At
those commonly or officially held.
(c) On (d) While

Facebook Page- https://www.facebook.com/vishalpariharpage Youtube- https://www.youtube.com/vishalparihar


Follow
65 Vishal Sir Telegram Channel- https://t.me/englishbyvishalsirchannel Instagram- https://www.instagram.com/vishalthetrainer
By Vishal sir

SBI PO, SBI CLERK, IBPS PO, IBPS CLERK, RRB PO, RRB CLERK, NIACL,
- LIC RBI grade B, RBI ASSISTANT, & Other competitive Exams

(e) At Direction (351-355): There is a sentence with a blank space in it.


Choose the word from the given options which fits
Q349. Norway has stolen a march over other developed countries by appropriately in the blank adding a proper and logical meaning
_______ that it would reduce 40% of its greenhouse gas emissions to the sentences.
by 2020 and become carbon neutral by 2030.
(a)allowing (b) posing Q351. The new age scooter is perfect not just as a commute option for
(c) estimating (d) perceiving women, but is functional enough for men __________.
(e) declaring (a) also (b) too
(c) well (d) nor
Q350. Ten years later, James Starley, an English inventor, made several (e) additionally
innovations that ____ the bicycle design.
(a) revolutionized (b) reformed Q352. The police __________ a gang specialized in duping telebanking
(c) invented (d) revolutionizes customers and arrested 10 persons.
(e) created (a) told (b) busted
Answers And Solutions (c) seized
(e)arrested
(d)caught
346Ans.(e) Q353. The arrested persons were produced in court and __________
The blank is followed by a plural form of verb and thus the noun Judicial custody.
needs to be plural. Out of the available options, only option E has (a) persuaded (b) sentenced
a plural noun and thus is the correct response. (c) left (d) remanded
347Ans.(c) (e) kept
The context here is of the volunteers raising something; it can be Q354. The school students in the vicinity will be taken to the nearby
slogans, funds or theatres __________the festival.
awareness. Thus, option C is the most accommodative of the (a) during (b) for (c) in
available options. (d) at (e) on
348Ans.(c)
For hearing the news, we use 'on'. So the correct response is c. Q355. A plethora of cultural talent caught the spectators busy at a talent
349Ans.(c) programme where students from various colleges get together to
'to steal a march on/over' means to gain an advantage over __________ their mettle in various contests.
(someone) by acting before they do. (a) show (b) prove
This means that the sentence talks about some step taken by (c) puzzle (d) learn
Norway which has given it some advantage over other developed (e) mention
countries. Answers And Solutions
Allowing and posing don't fit into the first blank, hence options A 351Ans.(b)
and B can be eliminated. The correct answer is: B
Just estimating a certain reduction will not be enough to gain an 352Ans.(b)
advantage over others, hence option C is eliminated. The correct answer is: B
Declaring a reduction will make the country liable to achieve the To bust means to arrest or detain.
target making it carbon neutral by the proposed year, hence option 353Ans.(d)
E is the most suitable response. The correct answer is: D
350Ans.(a)
When a new thing is introduced, it is termed as 'invention' but when 354Ans.(a)
changes in the existing thing is made, it is termed as 'innovations'. The correct answer is: A
Hence, the most suitable filler is 'revolutionized'. 355Ans.(c)
The correct answer is: C

Facebook Page- https://www.facebook.com/vishalpariharpage Youtube- https://www.youtube.com/vishalparihar


Follow
66 Vishal Sir Telegram Channel- https://t.me/englishbyvishalsirchannel Instagram- https://www.instagram.com/vishalthetrainer
By Vishal sir

SBI PO, SBI CLERK, IBPS PO, IBPS CLERK, RRB PO, RRB CLERK, NIACL,
- LIC RBI grade B, RBI ASSISTANT, & Other competitive Exams

360Ans.(c)
Direction (356-360): There is a sentence with blank/s space/s in it. Rate is the correct term so as to define both literacy and population.
Choose the word/s from the given options which fits Rate means a measurement of the number of times something
appropriately in both the blanks adding a proper and logical happens or exists during a particular period
meaning to the sentences.

Q356. Fortunately no one was hurt _______ the accident being ______. Directions (361-365): Each question below has one blank, which is
(1) although, fatal (2) though, damage indicating that something has been omitted. Find out which
(3) however, critical (4) as, serious option can be used to fill up the blank in the sentence in the
(5) in spite of, serious. same sequence to make it meaningfully complete.

Q357. As a _______ of the foreign rule the country was ______ into mass Q361. The penetration went __________ and was more devastating than
poverty. anyone had imagined.
(1) sequel, going (2) consequence, steeped (a) Converged (b) Climatic (c) Deeper
(3) consequence, sliding (4) fall-out, slipping (d) Managerial (e) Preclude
(5) sequence, much.
Q358. ______ of the farmers has lead to their _______. Q362. What I'm trying to do is to _________ people, to give them ways
(1) exploitation, insecurity (2) loan, progress to help them get well.
(3) progress, output (4) bonding, misery (a)Populates (b)Empower
(5) joy, progress. (c)Interprets (d)Adjourns
(e)Reschedule
Q359 ______ is the result of ______ growth.
(1) this, burden (2) corruption, corrupt Q363. The Company decided to _________ the project, along with the
(3) poverty, repressive (4) economy, planned jobs of the design team.
(5) unemployment, unplanned (a)Aspect (b)Ponder
. (c)Terminate (d)Appease
(e)Increase
Q360. The ____ of literacy determines the _____ of population. Q364. Despite the decay the mosque somehow retained a _________
(1) rate, quality (2) share, number grandeur.
(3) rate, rate (4) share, number (a)Disfiguring (b)Yearning
(5) lack, control (c)Blameless (d)Profound
Answers And Solutions (e)Aloof
356Ans.(e) Q365. Following a determined __________ in the east, there was
The sentence is showing a contradiction that although a serious eventually a popular uprising in the capital.
accident took place of nobody was hurt, therefore, in spite of is used (a)Reaction (b)Acceptance
along with serious. (c)Resistance (d)Magnificence
357Ans.(c) (e)Reliability
Consequence means result and sliding means a change to a lower
or worse condition. Answers And Solutions
358Ans.(a) 361Ans.(c)
Exploitation means to treat somebody unfairly by making them Option (c) is the most appropriate choice to be filled in the
work and not giving them much in return and insecurity means not sentence. According to the context of the sentence deeper is the
safe or protected. only word that fits in the blank.
359Ans.(e) Converging means tending to meet at a point.
Unemployment means the fact of a number of people not housing Managerial means relating to management or managers.
a job and Unplanned means not planned in advance Preclude means prevent from happening; make impossible.

Facebook Page- https://www.facebook.com/vishalpariharpage Youtube- https://www.youtube.com/vishalparihar


Follow
67 Vishal Sir Telegram Channel- https://t.me/englishbyvishalsirchannel Instagram- https://www.instagram.com/vishalthetrainer
By Vishal sir

SBI PO, SBI CLERK, IBPS PO, IBPS CLERK, RRB PO, RRB CLERK, NIACL,
- LIC RBI grade B, RBI ASSISTANT, & Other competitive Exams

362Ans.(b)
Option (b) is the most appropriate choice to be filled in the blank. Q368. We will save our lawn business from closing by allowing a much
According to the context of the sentence, the word that fits most larger company to ___________ the brand into theirs and buy us
suitably in the blank is empower. Empower means make (someone) out.
stronger and more confident, especially in controlling their life and (a)Barren (b)Irate
claiming their rights. (c)Resume (d)Subsume
Interprets means explain the meaning of (information or actions). (e)Assume
Adjourns means put off or postpone (a resolution or sentence)
Reschedule means change the time of (a planned event). Q369. With his informative presentation, William showed his peers how
363Ans.(c) __________ he truly was.
Option (c) is the most suitable choice to be filled in the blank. (a)Ecstasy (b)Erudite
According to the sentence and the usage , terminate is the most (c)Acclimate (d)Aesthetic
appropriate word to be used. Rest all the words do not fit in the (e)Bovine
blank. Option (a) and (b) are totally out of context. Hence the best answer
Terminate means bring to an end. choice is option (d)
Aspect means the side of a building facing a particular direction.
Ponder means think about (something) carefully, especially before Q370. The committee met for several hours, but ________________
making a decision or reaching a conclusion. nothing because they argued the whole time.
But ponder will take the preposition ‘over’. So it does not fit here. (a)Attain (b)Acquire
Appease means assuage or satisfy (a demand or a feeling). (c)Administer (d)Attributed
364Ans.(d) (e)Accomplished
Profound is the most appropriate option to be filled in the blank.
Profound means very great or intense.
Answers And Solutions
366Ans.(a)
Disfiguring means spoil the appearance of.
Yearning means a feeling of intense longing for something. Pragmatic- dealing with things sensibly and realistically in a way
that is based on practical rather than theoretical considerations
Aloof means not friendly or forthcoming ; cool and distant.
Prolific- (of a plant, animal, or person) producing much fruit or
365Ans.(c)
foliage or many offspring
Resistance is the most suitable option that fits in the blank among Portend- be a sign or warning that (something, especially
all of the given options. Resistance means the refusal to accept or something momentous or calamitous) is likely to happen
comply with something. Option (d) and (e) are totally out of context. Hence the best answer
Magnificence means glory, opulence. choice is option (a).
Reliability means the quality of being trustworthy or of performing
consistently well. 367Ans.(c)
Lethargic- affected by lethargy; sluggish and apathetic
Direction (366-370): In the following given questions there is a Aloft-up in or into the air; overhead
sentence with a blank space in it. Choose the word from the Prelate-a bishop or other high ecclesiastical dignitary
given options which fits appropriately in the given blank
Nocturnal- done, occurring, or active at nigh
adding a proper and logical meaning to the sentence.
Lofty- of imposing height
Q366.The tent is a choice of architectural strategy which is not merely Option (b) and (d) are totally out of context. Hence the best answer
choice is option (c).
_________.
368Ans.(d)
(a) Pragmatic (b) Prolific (c) Portend
(d) Believe (e) Cash Subsume- include or absorb (something) in something else
Q367. From their extremely slow movements and _______________ Barren-(of land) too poor to produce much or any vegetation
habits in the daytime these weird little creatures are commonly Irate- feeling or characterized by great anger
called sloths by Anglo-Indians. Resume- begin again or continue after a pause or interruption
(a) Aloft (b) Prelate (c) Lethargic Assume- suppose to be the case, without proof
(d) Nocturnal (e) Lofty

Facebook Page- https://www.facebook.com/vishalpariharpage Youtube- https://www.youtube.com/vishalparihar


Follow
68 Vishal Sir Telegram Channel- https://t.me/englishbyvishalsirchannel Instagram- https://www.instagram.com/vishalthetrainer
By Vishal sir

SBI PO, SBI CLERK, IBPS PO, IBPS CLERK, RRB PO, RRB CLERK, NIACL,
- LIC RBI grade B, RBI ASSISTANT, & Other competitive Exams

Option (a) and (b) are totally out of context. Hence the best answer Q375. A plethora of cultural talent __________ the spectators busy at a
choice is option (d). talent programme where students from various colleges get
369Ans.(b) together to __________ their mettle in various contests.
Ecstasy- an overwhelming feeling of great happiness or joyful (a) tried, show (b) kept, prove
excitement (c) caught, puzzle (d) held, learn
Erudite- having or showing great knowledge or learning. (e) helped, mention
Acclimate- respond physiologically or behaviourally to a change in Q376. The most important factor is the __________ of a wealthy Indian
a single environmental factor middleclass which can now _______ to send their children abroad
Aesthetic-concerned with beauty or the appreciation of beauty for education.
Bovine- relating to or affecting cattle (a) emergence, afford (b) advent, focus
370Ans.(e) (c) decline, manage (d) rise, wish
Accomplished- well educated and having good social skills (e) perception, go
The best answer choice is option (e)
All other given options are totally out of context and doesn’t fit in Q377. Crores of public money is __________ on parks in the city and yet
the blank. most of them are out of __________ for the public.
Attain- succeed in achieving (something that one has worked for) (a)invested, limits (b)spent, bounds
Administer- manage and be responsible for the running of (a (c)bet, reach (d)put, areas
(e)made, boundaries
business, organization, etc.)
Attributed- regard something as being caused by. Q378. Rules are for those who cannot __________ them and not for the
rich and influential who can __________ to ignore them.
Direction (371-430): There is a sentence with blank/s space/s in it. (a)follow, demand (b)set, opt
Choose the word/s from the given options which fits (c)break, suggest (d)find, ask
(e)challenge, choose
appropriately in both the blanks adding a proper and logical
meaning to the sentences. Q379. Experts cannot __________ enough on the benefits of __________
more fruit and vegetables In your daily diet.
Q371. The new age scooter is perfect __________ just as a commute (a) pressure, involving (b) strain, adding
option for women, but is functional enough for men __________ . (c) emphasize, contributing (d) state, mixing,
(a)vehicle, also (b) not, too (e) examine, producing
(c)nor, well (d) neither, nor
(e) either, additionally Q380. For the last three months, the dengue __________ has put southern
districts on high alert and has been keeping local administrations
Q272. The police __________ a gang __________in duping telebanking and the health department on their __________
customers and arrested 130 persons. (a) outbreak, toes (b) fever, hinges
(a) told, mastered (b) busted, specialized (c) epidemic, feet (d) mosquito ground
(c) seized, active (d)caught, dealing
(e)arrested, setting (e) disease, beds
,
Q373. The arrested persons were __________ in court and __________ Q381. It is __________ to drive in the evening and night when the
Judicial custody. __________ of accidents looms large due to nonfunctional
(a)demanded, persuaded (b)questioned, sentenced streetlights.
(c)taken, left (d)produced, remanded (a)juvenile, planning (b)easy, risk
(e)sent, kept (c)instinctive, fame (d)difficult, threat
(e)natural, feature
Q374. The school students __________ the vicinity will be taken to the Q382. An overcast sky made Sunday less than __________ for city
nearby theatres __________the festival. residents, who going by weather experts have more in __________
(a) in, during (b) on, for (c) at, in (a)difficult, fate (b)perfect, store
(d) among, at (e) around, on (c)holiday, sky (d)dream, sharing
(e)steady, backyard

Facebook Page- https://www.facebook.com/vishalpariharpage Youtube- https://www.youtube.com/vishalparihar


Follow
69 Vishal Sir Telegram Channel- https://t.me/englishbyvishalsirchannel Instagram- https://www.instagram.com/vishalthetrainer
By Vishal sir

SBI PO, SBI CLERK, IBPS PO, IBPS CLERK, RRB PO, RRB CLERK, NIACL,
- LIC RBI grade B, RBI ASSISTANT, & Other competitive Exams

Q390. The minister __________ about various __________ being


Q383. Many teachers __________ the lack of professional freedom as the provided by officials for the pilgrims including direction wise
__________ for leaving the job. colour coded passenger enclosures.
(a) cited, reason (b) explained, force (a) ceased, functions (b) enquired, facilities
(c) claimed, understanding (d) argued, culprit (c) relied, opportunities (d) informed, evidence
(e) believe, effect (e) asked, deity

Q384. Skeptics would not __________ that the earth actually moves, let Q391. Since foggy weather __________ visibility by several metres, the
alone that it __________ around the sun. railways has either partially _________ or diverted some of the
(a) permit, orbits (b) accept, revolves trains.
(c) experience, circles (d) assume, went (a)improves, started (b)impairs, called off
(e) challenge, spins (c)hampers, withdrawn (d)diminishes, stopped
(e)reduces, cancelled
Q385. Much of the __________ that cricket has is due to the fact it is a
__________ sport. Q392. The once __________ district is gradually being __________ of its
(a) allure, lucrative (b) criticism, controversial green cover.
(c) attraction, unpopular (d) flak, great (a)remote, eroded (b)arid, replenished
(e) comments, unusual (c)beautiful, devoid (d)picturesque, depleted
Q386. The __________ of Chinese plastic thread should be banned as it (e)lush, rob
__________ thousands of birds every year during the kite flying
season. Q393. The pilot knew she would be able to see the __________ lights of
(a) sale, cripples (b) sell, kills the city from her cockpit window, but she would not see the
(c) sale, forbids (d) sell, disables fireworks explode to welcome the new year as she would have
(e) selling, saves __________ to cruising altitude.
(a)few, soared (b)divine, escalate
Q387. Harish __________ with me some basic techniques that one can (c)glistening, jumped (d)shining, reached
__________ at home without even owning a drum set. (e)glittering, climbed
(a) deviced, shred (b) imparted, try
(c) learned, balance (d) fits, rehearses Q394. The New Year has __________ in good news for City hotels as
(e) shared, practice most properties are __________ for the whole month.
(a) brought, deserted (b) ushered, packed
Q388. In the present case, the facts clearly __________ that the required (c) pushed, full (d) steered, renovating
reasonable degree of care and caution was not __________ by (e) escorted, vacant
hospital in the treatment of the patient.
(a) reflect, compressed (b) fix, advancement Q395. Manufacturing growth across Asia, Europe and the Americas eased
(c) show, proceeded (d) observe, considered in current month as heavy price cutting failed to revive __________
(e) indicate, taken providing more evidence that a feeble global economic recovery
may be __________ to a halt.
(a) supply, grind (b) demand, grinding
(c) supply, grounding (d) demanding, grind
Q389. Students from the Middle East and the African countries have (e) demanded, going
__________ been __________ contributors to the pool of foreign
students in the university. Q396. The five-judge Bench had ruled that the L-G has to
(a) traditionally, major (b) conservatively, crucial _________act on the ‘aid and advice’ of the Council of
(c) mostly, most (d) mainly, salient Ministers, or refer to the President for a decision any matter on
(e) annually, lucid which there is a ___________with the Ministry, but has no
independent decision-making powers.
a) either, difference

Facebook Page- https://www.facebook.com/vishalpariharpage Youtube- https://www.youtube.com/vishalparihar


Follow
70 Vishal Sir Telegram Channel- https://t.me/englishbyvishalsirchannel Instagram- https://www.instagram.com/vishalthetrainer
By Vishal sir

SBI PO, SBI CLERK, IBPS PO, IBPS CLERK, RRB PO, RRB CLERK, NIACL,
- LIC RBI grade B, RBI ASSISTANT, & Other competitive Exams

b) neither, comparison (c) wait, handling (d) waiting, open


c) either, similarity (e) waited, open
d) neither, divulgence
e) none of these Q404. Operations in steel plant which __________a major accident Is
Q397. Crude oil markets rose after __________ a five year low __________gradu
rebounding after data suggested that tumbling prices may have (a) suffer limp (b) occur, limp
started to affect __________ activity in the fast growing US shale (c) suffer, coming (d) suffered, limping
oil industry.
(a) hit, drill (b) hitting, drilled (e) suffered, limped
(c) hitting, drilling (d) going, drilled
(e) touching, exhuming Q405. The new government is __________ a comprehensive revamp of
the health insurance scheme for the poor in a move that will
Q398. In fact, there is a strong case to promote liquor __________ in high __________ further many people.
quality factories under regulation so that the __________ of illicit (a) propose, loss (b) proposing, harm
liquor and hooch goes down. (c) proposing, benefit (d) proposed, benefit
(a)manufacturing, supply (b)production, consuming (e) proposed, harm
(c)generation, supply (d)generated, demand Q406. A new study has __________the five most common health Issues
(e) hitting, produce in men and how to __________ them.
Q399. The government ease foreign direct __________ norms for the (a) said, arrest (b) stated, shrink
construction development sector , which is expected to provide a (c) revealed, prevent (d) exposed, react
__________boost to the sector in terms of greater foreign capital (e) laid, restrict
inflows.
(a) investment, substance (b) invention, substantial Q407. __________ to the challenge of fighting drug addiction, children
(c) investment, substantial (d) divestment, meagre have __________ up the task of spreading awareness against drugs,
(e) investment, superficial (a) Taking, set (b) Respond, taken,
(c) Responding, taken (d) Resolving, set
Q400. The banks were nationalized in __________ to take banking to the (e) Contrary, set
__________ country and all its citizens. Q408. The night fabric can do __________ for your frame and as a
(a)aim, whole (b)order, backward __________ your confidence.
(c)orderly, entirely (d)order, entire (a) wonders, consequence (b) marvels, side
(e)view, wholly (c) good, process (d) remarkably, finish
(e) superb, result
Q401.Cassava grows __________ in South India, but has one problem it
contains a toxin that must be __________ by long so kiting in Q409. The wheat grains do not meet the __________ standards
water. __________ for procurement of food grains by the government
(a) hardly, remove (b) easy, remove agencies.
(c) easily, remove (d) hard, removed (a) total, kept (b) most, meant
(e) easily, removal (c) excellence, allowed (d) optimistic, placed
Q402. A human journey is __________ when there is someone at the door (e) quality, prescribed
to __________ them when they get at home.
(a) incomplete, greet (b) complete, ignore Q410. Students will be __________ on their reading and writing abilities
(c) incompetent, ignore (d) complete, greet through question papers __________ by the government.
(e) complete, great (a) total, kept (b) kept, set
, (c) tested, provided (d) evaluated, asked
Q403. An adult reader is __________ for his tutor to come along and show (e) calculated, assigned
hit n how to __________ a book.
(a) waiting, handle (b) wait, handle
Facebook Page- https://www.facebook.com/vishalpariharpage Youtube- https://www.youtube.com/vishalparihar
Follow
71 Vishal Sir Telegram Channel- https://t.me/englishbyvishalsirchannel Instagram- https://www.instagram.com/vishalthetrainer
By Vishal sir

SBI PO, SBI CLERK, IBPS PO, IBPS CLERK, RRB PO, RRB CLERK, NIACL,
- LIC RBI grade B, RBI ASSISTANT, & Other competitive Exams

Q411. Although we __________ stiff competition from countries such as Q418. The human mind is never __________it advances or it
the Philippines, BPO industry is still huge and__________ __________
hundreds of Indians. (a) absolute, diminishes (b) dynamic, stops
(a) face, employs (b) experience, maintain (c) perfect, disintegrates (d) stationary, retrogrades
(c) prove, engages (d) witness, services (e) happy, decomposes
(e) note, helps
Q412. A language is considered nearly __________ when only a few Q419. If misery is the, effect of ill fortune, It ought to be pitied if of
native speakers __________ it and it is no longer being taught to __________ to be __________
children. (a) virtue, criticised (b) calamity, reverenced
(a) exhausted, say (b) abolished, speak (c) virtue, protected (d) vice, reverenced
(c) extinct, use (d) pass, treat (e) virtue, reverenced
(e) past, need
Q420. It would be impossible for us to continue living in this world if each
Q413. We __________ do not pay attention to the 30 minute gaps of us __________ exactly, what fate had in __________ for him.
sandwiched __________ two meetings. (a) follow, plan (b) appreciate, strategy
(a) seldom, in (b) regularly, middle (c) design, anticipation (d) visualize, hidden
(c) frequently, within (d) often, between (e) knew, store
(e) at least, among
Q421. It is the __________ , of selfishness for men, who fully
Q414. __________other cities where lack of education or civic sense __________ in their own case the great advantages of good
might be one of the reasons that may have contribute to issues education, to deny these advantages to women.
related to solid waste management, most residents here are (a) parody demand (b) height, appreciate
__________ of the pros and cons. (c) height, assimilate (d) degree, appreciate
(a) Unlike, aware (b) Inconsistent, conscious (e) level, advance
(c) Similarly, known (d) Contrary, careful
(e) Likewise, sensible Q422. The learner should be __________ to take a small first step one that
will provide immediate Success and __________ the learning.
Q415. The fact that Saudi Arabia __________ nearly 10 million barrels of (a) encouraged, reinforce (b) forced, organize
crude oil per day is not the __________ reason it has been called (c) directed, recognize (d) cautioned, reinforce
the Central Bank of Oil. (e) encouraged, acknowledge
(a) removes, alone (b) harvests, just
(c) saves, solely (d) yield, single Q423. His death __________ many tributes than have been paid at the
(e) produces, only __________ of any other human being in history.
(a) brought, passing (b) directed, helm
Q416. Vision is usually __________ most effectively when many (c) delivered, description (d) invited, living
different __________ are used. (e) acknowledged, perpetuation
(a) developed, manifestations (b) adapted, organizations .
(c) communicated vehicles (d) exhibited, forms
(e) described, thought Q424. Only with executive __________ can the organization concentrate
, its energies on __________ competitive advantage over time.
Q417. People who have been through difficult, painful and not very (a) position, embarking (b) deployment directing
__________ change efforts often end up __________ both (c) contingent, fabricating (d) commitment, sustaining
pessimistic and angry conclusions. (e) satisfaction moulding
(a) successful, drawing (b) meaningful, projecting
(c) reliable, evolving (d) strong, following Q425. All the performances of human art, at which we look with praise or
(e) challenging, lamenting. wonder, are __________ of the restless, __________ of
perseverance.
(a) manifestations, pronouncement

Facebook Page- https://www.facebook.com/vishalpariharpage Youtube- https://www.youtube.com/vishalparihar


Follow
72 Vishal Sir Telegram Channel- https://t.me/englishbyvishalsirchannel Instagram- https://www.instagram.com/vishalthetrainer
By Vishal sir

SBI PO, SBI CLERK, IBPS PO, IBPS CLERK, RRB PO, RRB CLERK, NIACL,
- LIC RBI grade B, RBI ASSISTANT, & Other competitive Exams

(b) projections component 379Ans.(d) state,


(c) instances, force 380Ans.(a)
(d) proofs, humanity keep (one) on (one's) toes means: To force someone to stay active,
(e) visions, future alert
381Ans.(d)
Q426. He objected to the proposal because it was founded on a Juvenile means behaving like somebody of a younger age
__________ principle and also was – at time. Indistinctive means clearly different from others and therefore easy
(a) faulty, desirable (b) Imperative, reasonable to recognize
(c) wrong, Inconvenient, (d) sound, acceptable 382Ans.(b)
(e) uncomforting, deplorable In store for someone/something means planned or likely to happen
Q427. The criterion for __________ a player should be his recent 383Ans.(a) cited, reason
performance but unfortunately, the journalists are __________ to 384Ans.(b) accept,
be carried away by earlier successes. 385Ans.(a)
(a) condemning, satisfying (b) judging, prone Allure means the quality of being attractive and exciting
(c) revealing reluctant (d) eager, acclaims Lucrative means allowing somebody to earn a lot of money
(e) criticising, clean Flak means criticism
Q428. For the last half century he __________ himself to public affairs 386Ans.(a)
__________ taking a holiday. Forbids means not giving permission or refusing to allow but birds
(a) by, committed (b) after, offered do not need permission.
(c) devoted, without (d) scarified, after 387Ans.(e) shared, practice
(e) prepared, before 388Ans.(e) indicate, taken
Ans.(c) devoted, without 389Ans.(a)
Salient means important. Though option D fits grammatically but
Q429. You will see signs of __________ everywhere, which speak well
contextually it is incorrect.
for the __________ of these people. 390Ans.(b) enquired,
(a) decoration senses (b) clear debris 391Ans.(e) reduces and cancelled is the correct choice
(c) beauty careful (d) industry prosperity 392Ans.(d)
(e) repairs extravaganza Picturesque: beautiful/ attractive
Devoid means lacking
Q430. The police arrested Ramesh on a __________ of theft but for lack Arid means dry.
of evidence __________ him. 393Ans.(e)
(a) crime, imprisoned (b) punished, complaint glistening means shining with a sparkling light.
(c) left, condition (d) tip, absconding 394Ans.(b)
(e) charge, released Ushered means to show or guide (someone) somewhere and packed
Answers And Solutions will come along as a suitable word.
371Ans.(b) not, too 395Ans.(b)
372Ans.(b) grind to a halt or come to a grinding halt means slow down
To bust means to arrest or detain. gradually and then stop completely.
373Ans.(d) 396Ans. a) divulgence means the action of revealing private or sensitive
Persuade means to make somebody believe something information.
Remand means the time before a prisoner’s trial takes place ‘Either’ is used with ‘or’, ‘Neither’ is use in combination with
374Ans.(a) in, during ‘nor’. By the context of the sentence, ‘difference’ is the correct
375Ans.(c) caught, puzzle alternative here.
376Ans.(a) emergence, afford
377Ans.(b) 397Ans.(c) exhuming means unearth
Out of bounds means banned or forbidden 398Ans.(a) manufacturing means to make into a product suitable for use
378Ans.(e) challenge, choose 399Ans.(c)

Facebook Page- https://www.facebook.com/vishalpariharpage Youtube- https://www.youtube.com/vishalparihar


Follow
73 Vishal Sir Telegram Channel- https://t.me/englishbyvishalsirchannel Instagram- https://www.instagram.com/vishalthetrainer
By Vishal sir

SBI PO, SBI CLERK, IBPS PO, IBPS CLERK, RRB PO, RRB CLERK, NIACL,
- LIC RBI grade B, RBI ASSISTANT, & Other competitive Exams

Substantial means large in amount Deployment means act of moving something or someone into a
Meagre means too small in amount strategic position or a position of readiness,
400Ans.(d) order, entire Contingent means a group of armed forces forming part of a larger
401Ans.(c) easily, remove force
425Ans.(d)
402Ans.(d) complete Pronouncement means a formal or authoritative announcement or
declaration.
403Ans.(a) 426Ans.(c)
404Ans.(d) limp means to hobble Imperative means very important or urgent
405Ans.(c) proposing to put forward Deplorable means morally bad and deserving disapproval
427Ans.(b) judging,
406Ans.(c) revealed and prevent is the correct answer Q428. Ans.(c) devoted, without
407Ans.(b) Respond, taken
429Ans.(d)
408Ans.(e) superb, result
Debris means the remains of something broken down
409Ans.(e)
Prosperity means the state of being successful, especially with
Optimistic means to have positive approach
money
410Ans.(c)
Extravaganza means a large, exciting and impressive entertainment
411Ans.(a) face, employs
or event
430Ans.(e)
412Ans.(c) extinct, use
Absconding means to run away from a place where you should stay,
413Ans.(d) often,
sometimes with something that you should not take
414Ans.(a) Unlike, aware
415Ans.(e) produces, only
Direction (431-432): The following sentences consist of three blanks.
416Ans.(d) exhibited
You are provided with three words that fits coherently in the
417Ans.(a) successful, drawing sentence. Identify and mark the alternative catering the correct
418Ans.(d) sequence of the words in which they must be filled to make the
Retrogrades means a degenerated person sentence contextually meaningful and grammatically correct.
419Ans.(e)
Calamity means a terrible event that causes a lot of damage or harm Q431. With the _____________ of the Reserve Bank of India’s new bad
Reverenced means regard or treat with deep respect. loan ___________ framework that seeks greater
Virtue means behaviour which shows high moral standards _______________ from banks, Kotak warned small and medium
Vice means a moral weakness or bad habit enterprise entities could also be hit and banks will have to improve
420Ans.(a) their underwriting standards.
Anticipation means excited feelings about something that is going [I] Implementation [II] Disclosures
to happen [III] Resolution
421Ans.(b) (a)(I), (II), (III) (b)(II), (I), (III)
Parody means a piece of writing, speech or music that copies the (c)(I), (III), (II) (d)(III), (II), (I)
style of somebody/something in a funny way (e)(III), (I), (II)
Assimilate means to learn and understand something
422Ans.(a) Q432. The department-related parliamentary _______________, after
Reinforce means to make something stronger reviewing the ________________,
423Ans.(a) encouraged, reinforce said the bridge course should not be made a mandatory
Helm means the part of a boat or ship that is used to guide it. _______________ in the present Bill.
Perpetuation means the continuation or preservation of a situation, [I] Proposal
idea, etc [II] Provision
424Ans.(d) [III] Committee
(a)(I), (II), (III) (b)(II), (I), (III)

Facebook Page- https://www.facebook.com/vishalpariharpage Youtube- https://www.youtube.com/vishalparihar


Follow
74 Vishal Sir Telegram Channel- https://t.me/englishbyvishalsirchannel Instagram- https://www.instagram.com/vishalthetrainer
By Vishal sir

SBI PO, SBI CLERK, IBPS PO, IBPS CLERK, RRB PO, RRB CLERK, NIACL,
- LIC RBI grade B, RBI ASSISTANT, & Other competitive Exams

(c)(I), (III), (II) (d)(III), (II), (I) Inculcate means instil (an idea, attitude, or habit) by persistent
(e)(III), (I), (II) instruction.
Answers Solutions 434Ans.(c)
Outweigh means be heavier, greater, or more significant than.
431Ans.(c) Percolate means (of a liquid or gas) filter gradually through a
The correct sequence of the words to be filled in the sentence is (I), porous surface or substance.
(III), (II) to gain a grammatically correct and contextually
meaningful sentence. Thus, the sentence formed is “With the Directions (435-437): In each of the questions given below, a sentence
implementation of the Reserve Bank of India’s new bad loan is given with one blank. Below each sentence, FOUR words are
resolution framework that seeks greater disclosures from banks, given out of which two can fit into the sentence. Five options
Kotak warned small and medium enterprise entities could also be are provided with various combinations of these words. You
hit and bank will have to improve their underwriting standards.” have to choose the combination with the correct set of words
Hence, option (c) is the most suitable choice. which can coherently fit into the given sentence.
432Ans.(e)
The correct sequence of the words to be filled in the sentence is Q435. The friar-birds are noisy and _____________________ species of
(III), (I), (II) to gain a grammatically correct and contextually the group of honeyeaters, and mob hawks and other birds of prey,
meaningful sentence. Thus, the sentence formed is “The which leave them unmolested.
department-related parliamentary committee, after reviewing the (I)Miraculous
proposal, said the bridge course should not be made a mandatory (II)Pugnacious
provision in the present Bill.” Therefore, option (e) becomes the (III)Antagonistic
most viable choice. (IV)Ardent
(a)I-II (b)II-III
Directions (433-434): Each question below has two blanks. There are (c)I-III (d)I-IV
five pairs of words below the sentence. Each pair is lettered. (e)III-IV
Choose the pair of words which can be filled up in the blanks
in the sentence in the same order so as to complete the sentences Q436. The teacher created detailed lesson plans with examples so that her
meaningfully. students wouldn't _____________________ her with questions
about the assignment.
Q433. ______________ in yourself! Have faith in your abilities! Without (I)Pester (II)Muster
a humble but reasonable ______________ in your own powers you (III)Badger (IV)Nurture
cannot be successful or happy. (a)I-II (b)II-III
(a) encourage, enforced (b) inculcate, ordered (c)I-III (d)I-IV
(c) cooperate, stipulated (d) Believe, confidence (e)III-IV
(e) declared, pressurized
Q437. Everyone was shocked when the minister told Abigail she was a
Q434. The budget for the National Rural Drinking Water Mission, a key _____________________ hypocrite who judged people without
______________ of Jal Jeevan Mission, has been doubled from last acknowledging her own sins.
year’s revised ______________ to more than ₹10,000 crore. (I) Calm
(a) realization, damage (b) appreciation, percolate (II)Faithful
(c) component, estimates (d) achievement, crumble (III)Sanctimonious
(e) destination, magnify (IV)Hypocritical
(a)I-II (b)II-III
Answers And Solutions (c)I-III (d)I-IV
433Ans.(d) (e)III-IV
Stipulated means demand or specify (a requirement), typically as Answers And Solutions
part of an agreement.
Persisted means continue in an opinion or course of action in spite 435Ans.(b)Pugnacious -eager to fight or argue, antagonistic is the
of difficulty or opposition. synonym of the pugnacious

Facebook Page- https://www.facebook.com/vishalpariharpage Youtube- https://www.youtube.com/vishalparihar


Follow
75 Vishal Sir Telegram Channel- https://t.me/englishbyvishalsirchannel Instagram- https://www.instagram.com/vishalthetrainer
By Vishal sir

SBI PO, SBI CLERK, IBPS PO, IBPS CLERK, RRB PO, RRB CLERK, NIACL,
- LIC RBI grade B, RBI ASSISTANT, & Other competitive Exams

Here the sentence coveys the argumentative tone so the correct (c) Both (1) and (3) (d) All of (1), (2), (3) and (4)
combination of words to be used in the blank is II-III (e) None of (1), (2), (3) and (4)
Ardent- very enthusiastically or passionately, so the best answer ‘
choice here is the option (b). Q440. I. Dunluce Castle, between Portrush and Bushmills, stands on a
436Ans.(c) rock separated from the mainland by a ____________which is
Badger, Pester -repeatedly and annoyingly ask (someone) to do spanned by a bridge
something. II. Jenn stared, the image of her lifeless daughter falling into
Muster- assemble (troops), especially for inspection or in the ___________ replaying over and over in her mind.
preparation for battle. (1) Dichotomy (2) Paean
The correct combination of words to be used in the sentence is I- (3) Testimonial (4) Chasm
III. (a) Both (1) and (3) (b) Both (1) and (4)
The best answer choice here is the option (c). (c) Only (2) (d) All of (1), (2), (3) and (4)
437Ans.(e) (e) None of the above
Sanctimonious- making a show of being pious(derogatory),
Hypocritical- behaving in a way that suggests one has higher
Answers And Solutions
standards or more noble beliefs than is the case. 438Ans.(c)
Sanctity- the state or quality of being holy, sacred, or saintly, sneer- Chasm [noun] means a deep fissure in the earth’s surface or a
a contemptuous or mocking smile, remark, or tone profound difference between people, viewpoints, feelings etc.
Here the sentence coveys the arrogant behavior of a woman so the Accolades, Encomiums and Eulogies are plural nouns and
correct combination of words to be used in the sentence is III-IV. synonyms which mean a speech or a piece of writing that praises
The best answer choice here is the option (e). someone or something highly.
The word which satisfy the contextual requirement, of the above
given sentences, is praises.
Directions (438-440): Below are two sentences with one blank each. So, (1), (3) and (4) are the correct answer and (c) is the correct
Choose the correct word/pair which would fill both blanks answer.
while keeping the sentences grammatically correct and
contextually meaningful.
439Ans.(d)
Q438. I. After all that Mother Teresa accomplished during her life, she Clandestine, Furtive, Surreptitious are synonyms of the word
certainly deserves every ________ that was ever written praising ‘secret’ which mean
her tireless work for God and humanity. keeping something secret; kept secret or done secretly, especially
II. Since the lone fireman rescued twenty children from a burning because illicit.
bus, he will more than likely receive an ____________ from the So, all of the given alternatives (1), (2), (3) and (4) could fill the
president. blank of above given sentences.
(1) Accolades (2) Chasm Hence, option (d) is the correct answer.
(3) Encomiums (4) Eulogies 440Ans.(b)
(a) (3) and (4) (b) Only (2) Dichotomy [noun] and chasm [noun] are synonyms which mean ‘a
(c) (1), (3) and (4) (d) (1) and (2) division or contrast between two things that are or are represented
(e) None of (1), (2), (3) and (4) as being opposed or entirely different’.
Paean [noun] and Testimonial [noun] are synonyms which mean ‘a
Q439. I. Other than their ______________glances at the audience, they song of praise or triumph’ or ‘a creative work expressing
were totally involved all the time. enthusiastic praise’.
II. At this time Protestant opinions were being disseminated in Among the alternatives, (1) and (4) have a meaning closer to the
England chiefly by the ______________circulation of the works of meaning of ‘difference’ and appropriately fit the blanks.
Wycliffe, and especially of his translations of the New Testament. Hence, option (b) is the correct answer.
(1) Secret (2) Clandestine
(3) Furtive (4) Surreptitious Direction (441-444): In following questions given below there are two
(a) Only (1) (b) Both (1) and (2) statements, each statement consists of two blanks. You have to

Facebook Page- https://www.facebook.com/vishalpariharpage Youtube- https://www.youtube.com/vishalparihar


Follow
76 Vishal Sir Telegram Channel- https://t.me/englishbyvishalsirchannel Instagram- https://www.instagram.com/vishalthetrainer
By Vishal sir

SBI PO, SBI CLERK, IBPS PO, IBPS CLERK, RRB PO, RRB CLERK, NIACL,
- LIC RBI grade B, RBI ASSISTANT, & Other competitive Exams

choose the option which provides the correct set of words that Dappled- marked with spots or rounded patches.
fits both the blanks in both the statements appropriately and in Daubed- carelessly coat or smear (a surface) with a thick or sticky
the same order making them meaningful and grammatically substance.
correct. Hence, the option (a) is the correct answer.

Q441. (1) In many situations, remaining silent is __________to admitting


Answers And Solutions
guilt, so speak to prove your________. 441Ans.(a)
(2) This is _____________ to an assumption that Raghu is doing Tantamount- equivalent in seriousness to; virtually the same as.
all this to let everyone aware about his ___________. Reckoning- the action or process of calculating or estimating
(a) Tantamount, innocence (b) Redundant, Duress something.
(c) Reckoning, bondage (d) Reign, autonomy Reign- hold royal office; rule as monarch.
(e) Dominion, volition Hence, the option (a) is the correct answer.
442Ans.(c)
Q442. (1) After her third pet dog died, Maria was simply___________: Indecency- indecent behavior.
this time even the possibility of buying a new dog no longer held Abate- Something unpleasant or severe) become less intense or
any_____. widespread.
(2) Early morning, ________ figures at bus stops might take away Forlorn- pitifully sad and abandoned or lonely.
your ______. Ecstasy- an overwhelming feeling of great happiness or joyful
(a) Tenant, indecency (b) Swagger, Abate excitement.
(c) Forlorn, joy (d) Ecstasy, Haste Hence, the option (c) is the correct answer
(e) Diluted, Vibrant 443Ans.(d)
. Eclectic- deriving ideas, style, or taste from a broad and diverse
Q443. (1) Shakespeare is an ___________author in the English language, range of sources.
but I find his writing uninteresting and_________. Eulogy- a speech or piece of writing that praises someone or
(2) Despite of being an_________ psychologist he used to show his something highly, especially a tribute to someone who has just
________________ behavior during the treatment with his died.
patients. Intrigue-arouse the curiosity or interest of; fascinate.
(a) Eclectic, ambiguous (b) Eulogy, Agitating Eminent- (of a person) famous and respected within a particular
(c) Intrigue, Fervor (d) Eminent, melodramatic sphere.
(e) Eccentric, Vicious Melodramatic- characteristic of melodrama, especially in being
exaggerated or overemotional.
Q444. (1) The rise of the computer was an ____________ shift in Vicious- deliberately cruel or violent.
technology and ______________. Hence, the option (d) is the correct answer.
(2) The dispute among different religions was
____________because of the increasing differences in________. 444Ans.(e)
(a) Intolerable, invention (b) Introvert, Fashion Inexorable- impossible to stop or prevent.
(c) Insurmountable, strategy (d) Inseparable, vulture Vulture- a large bird of prey with the head and neck more or less
(e) Inexorable, culture bare of feathers.
Insurmountable- too great to be overcome.
Q445. (1) Count Rumford _________ the new theory of heat, Fashion and Invention has nothing to do with the difference in
demonstrating that it was wholly ________________ to explain the religions or rise of the computer.
observations. Hence, the option (e) is the correct answer.
(2) He was __________ by the opposition though, the latter had 445Ans.(a)
__________ knowledge in the field. Denigrated- criticize unfairly; disparage.
(a) Denigrated, inadequate (b) Daunted, improper Inadequate- lacking the quality or quantity required; insufficient for
(c) Dappled, insufficient (d) Daubed, outwit a purpose.
(e) Danube, unable
Daunted- make (someone) feel intimidated or apprehensive.
Facebook Page- https://www.facebook.com/vishalpariharpage Youtube- https://www.youtube.com/vishalparihar
Follow
77 Vishal Sir Telegram Channel- https://t.me/englishbyvishalsirchannel Instagram- https://www.instagram.com/vishalthetrainer
By Vishal sir

SBI PO, SBI CLERK, IBPS PO, IBPS CLERK, RRB PO, RRB CLERK, NIACL,
- LIC RBI grade B, RBI ASSISTANT, & Other competitive Exams

Directions (446-450)The following question consists of a sentence(s)


with one blank only. You are given four words as answer
Answers And Solutions
choices and from the four choices you have to pick up two
correct answers/pairs, either of which will make the sentence 446Ans.(c)
meaningfully complete. Acumen means the ability to make a good judgement.
Q 446. A major disadvantage of online _____________ is that people can Discourse means written or spoken communication or debate.
often say whatever they like in complete anonymity. Curriculum means the subjects comprising a course of study in a
A. Acumen B. Discourse school or college.
C. Curriculum D. Debate Debate is a serious discussion of a subject in which many people
(a) AC (b) BC take part.
(c) BD (d) AB Thus, both B and D make the sentence meaningful.
(e) CD Thus option 3 is the correct answer.
447Ans.(c)
Drama an exciting, emotional, or unexpected event or
Q447. Because the cult leader’s brainwashing was so __________, it took circumstance.
months for the man to break free of the programming Monolithic means huge, massive and uniform, consisting of one
A. Drama B. Monolithic thing only.
C. Joke D. Uniform Joke is a thing that someone says to cause amusement or laughter.
(a) DC (b) CA Uniform remaining the same in all cases and at all times.
(c) BC (d) BD Thus, both B and D make the sentence meaningful.
(e) AB Thus option 3 is the correct answer.
448Ans.(c)
Q448. Cara foolishly believed defaulting on her student loans would Nullify means make legally null and void.
__________ her obligation to pay them and allow her to spend her Appease means to satisfy or please someone.
extra money on other things. Shun means to avoid or ignore.
A. Nullify B. Appease Negate means make ineffective; nullify.
C. Shun D. Negate Thus, both A and D make the sentence meaningful.
(a) AC (b) DC Thus option c is the correct answer.
(c) DA (D) CD 449Ans.(a)
(E) BC Animosity means hatred and bitter feeling.
Proletariat refers to working-class people.
Q449. The people in the ____________ are angry because as they fight Heterogeneous means consisting of parts or things that are very
for food, the wealthier classes are throwing away produce and different from each other.
meats. Commoners refers to ordinary citizens.
A. Animosity B. Proletariat Thus, both B and C make the sentence meaningful.
C. Commoners D. Heterogeneous Thus option 1 is the correct answer.
(a) BC (b) CA 450Ans.(b)
(c) AD (d) DB Invalidate means to render invalid, cancel.
(e) CD Discredited means cause (an idea or account) to seem false or make
ineffective.
Q450. The myth of philanthropy is quickly ___________ by a realistic Anguished means experiencing or expressing severe mental or
look at how older people lead their lives. physical pain.
A. Invalidated B. Discredited Concealed means hidden away from plain sight.
C. Anguished D. Concealed Thus, both B and A make the sentence meaningful.
(a) CB (b) BA (c) AC Thus option 2 is the correct answer.
(d) AD (e) CD

Facebook Page- https://www.facebook.com/vishalpariharpage Youtube- https://www.youtube.com/vishalparihar


Follow
78 Vishal Sir Telegram Channel- https://t.me/englishbyvishalsirchannel Instagram- https://www.instagram.com/vishalthetrainer
By Vishal sir

SBI PO, SBI CLERK, IBPS PO, IBPS CLERK, RRB PO, RRB CLERK, NIACL,
- LIC RBI grade B, RBI ASSISTANT, & Other competitive Exams

Directions (451- 455) Each of the following questions has two sentences 451Ans.(d)
with one blank each. Choose the option that best fits in both the Gravitating means an attractive force
blanks. Endeavors means ventures
Ancestry means generational lineage
Q451.1. A girl's place is in the home, not ___________________ around Gallivanting means wandering aimlessly for pleasure and dainty
and filling her head with nonsense. means small and delicate.
2. Well you won't be able to go off ___________ around like this The answer is gallivanting because it has the most appropriate fit.
when there's a baby to be taken care of. Thus, option 4 is the correct answer.
(a) Gravitating (b) Endeavors 452Ans.(a)
(c) Ancestry (d) Gallivanting Capricious means variable
(e) Dainty Vibrant means colorful or cheerful
Gaping means widening
Q452. 1. They had indulged them with a ______________ dominion, Mellow means pleasant or also matured
sometimes over the one, sometimes over the other object of desire. Ringing can refer to buzzing.
2. Employees need legal protection against _____________ and The answer is capricious as it fits the most appropriately in both the
unfair actions by their employers. blanks equally.
(a) Capricious (b) Vibrant Thus, option a is the correct answer.
(c) Gaping (d) Mellow 543Ans.(c)
(e) Ringing Furnished means well-provided
Hollowed means vacant
Q453. 1. ____________ out mid-way was a small chamber which Dusty is being covered with dust
contained seven standing stones, now clamped to the walls with Saddled means burden
metal hoops. Dicey means unpredictable.
2. Each of the _________ track got filled by debris after the storm, The answer is hollowed as it fits most appropriately in both the
doubling up their work henceforth. blanks.
(a) Furnished (b) Dusty Thus, option c is the correct answer.
(c) Hollowed (d) Saddled
(e) Dicey 454Ans.(e)
Unembellished means not having added features
Q454. Unencumbered means without restrictions
1. Based on blind number theory Undignified means lacking dignity
in _______________ mathematics, primary research on water Impaled means pierced by sharp object
environment risk is made with uncertained information. Unascertained means being unknown
2. The entire neighborhood went _________ when the census took The answer is unascertained because from the given words, it is the
place. most fitting.
(a) Unembellished (b) Unencumbered Thus, option 5 is the correct answer.
(c) Undignified (d) Impaled 455Ans.(d)
(e) Unascertained Dispensable means replaceable
. Resolute means determined,
Q455. 1. Even though his argument made it big with the general public, Gorging means eating greedily,
the higher-ups held it ________. Facile means superficial
2. In these circumstances ____________ and fallacious deductions Broken means not together.
about the consequences of having abolished the death penalty were The answer is facile as it the best fitting for both of the blanks.
bound to be rife. Thus, option 4 is the correct answer
(a) Resolute (b) Gorging
(c) Facile (d) Broken Directions (456-460)The following question consists of a sentence(s)
Answers And Solutions with one blank only. You are given six words as answer choices

Facebook Page- https://www.facebook.com/vishalpariharpage Youtube- https://www.youtube.com/vishalparihar


Follow
79 Vishal Sir Telegram Channel- https://t.me/englishbyvishalsirchannel Instagram- https://www.instagram.com/vishalthetrainer
By Vishal sir

SBI PO, SBI CLERK, IBPS PO, IBPS CLERK, RRB PO, RRB CLERK, NIACL,
- LIC RBI grade B, RBI ASSISTANT, & Other competitive Exams

and from the six choices you have to pick up appropriate (b) Both IV and V (c) II, IV and VI
answer/s, which will make the sentence meaningfully complete. (d) Both I and IV (e) I, II, IV and V

Q456. Our quartet was out ____________ and we knew we stood good to
Answers And Solutions
take in a lot of change before the night was over. 456Ans.(d)
I. Hustling II. Fluttering Let's find out the meaning of given words :-
III. Averring IV. Assuaging Hustling - force (someone) to move hurriedly or unceremoniously.
V. Heckling VI. Jostling Fluttering - (of a bird or other winged creature) flap (its wings)
(a) I, II and VI (b) Both III and V quickly and lightly.
(c) Only VI (d) Both I and VI Averring - state or assert to be the case.
(e) Both II and IV Assuaging - make (an unpleasant feeling) less intense; satisfy.
Heckling - interrupt (a public speaker) with derogatory or
Q457. Clearly, those publish ___________ calumniator, rumormonger on aggressive comments or abuse.
the net, count to China with 100 million plan the netizen, it is Jostling - push, elbow, or bump against (someone) roughly,
minority after all. typically in a crowd; hustle.
I. Baleful II. Pent-up Since, in crowd generally push, thrust, moving hurriedly etc, are
III. Salubrious IV. Litany normal. Hence, option 4 i.e. I & VI is the correct answer.
V. Foliage VI. Permeable 457Ans.(e)
(a) IV, V and II (b) Only III Let's find out the meaning of given words :-
(c) Both I and III (d) Only VI Baleful - threatening harm; menacing.
(e) I, II, III and VI Pent-up - (of emotions, energy, etc.) unable to be expressed or
Q458. The clusters respond well to gibberellic acid or released.
cane _____________ to improve cluster compactness and berry Salubrious - health-giving; healthy; beneficial; wholesome.
size. Litany - a tedious recital or repetitive series.
I. Bolstering II. Girdling Foliage - plant leaves collectively.
III. Hunching IV. Culling Permeable - (of a material or membrane) allowing liquids or gases
V. Girding VI. Presaging to pass through it.
(a) I and III (b) II, IV and VI Here, these products can be either harmful to air water; or
(c) Only III (d) I, II, III and V beneficial. Also, it may be their energy is not produced in the air or
(e) All of the given water; or they can be produced. All the conditions are possible.
. Hence, option 5 is the correct answer.
458Ans.(d)
Q459. Autumn wind _____________ up the leaves dropping in the street, Let's find out the meaning of given words :-
a solemn sight of solitude with autumn-wind and dancing leaves. Bolstering - support or strengthen; boost.
I. Rolled up II. Mustered Girdling - surround; encircle.
III. Pristine IV. Furled Hunching - sit or stand with one's shoulders raised and the top of
V. Usurped VI. Hunched one's body bent forward; to push; thrust upon.
(a) Only I (b) Both I and IV Culling - select from a large quantity; obtain from a variety of
(c) Both IV and V (d) II, III and V sources.
(e) III, V and VI Girding - surround; encircle; encircle (a person or part of the body)
with a belt or band.
Q460. So he peered, leaning forward to catch a ______________ of scent, Presaging - be a sign or warning of an imminent event, typically an
listening to the man's quiet muttering. unwelcome one.
I. Mural II. Whiff The context explains that the Hollywood was managing itself to
III. Morrow bring out of recession. Thus, it boosted itself; pushed itself. Or,
IV. Zephyr V. Ambit perhaps it was surrounded by influence of recession. Thus, option
VI. Waft 4 i.e. I, II, III & V is the correct answer
(a) Only III 459Ans.(b)

Facebook Page- https://www.facebook.com/vishalpariharpage Youtube- https://www.youtube.com/vishalparihar


Follow
80 Vishal Sir Telegram Channel- https://t.me/englishbyvishalsirchannel Instagram- https://www.instagram.com/vishalthetrainer
By Vishal sir

SBI PO, SBI CLERK, IBPS PO, IBPS CLERK, RRB PO, RRB CLERK, NIACL,
- LIC RBI grade B, RBI ASSISTANT, & Other competitive Exams

Let's find out the meaning of given words :-


Rolled up - move in a particular direction by turning over and over Q464. It didn't take long ______ Lisa to adjust ____ her new job
on an axis. (a) During, in (b) After, outside
Mustered - assemble (troops), especially for inspection or in (c) For, to (d) Before, toward
preparation for battle. (e) At, near
Pristine - in its original condition; unspoilt. Q465. I _________ taken my lunch and decided to ________ in a box of
Furled - roll or fold up (something) neatly and securely. old books and family photographs.
Usurped - take the place of (someone in a position of power) (a) Have, look (b) Had, rummage
illegally; supplant. (c) Has, skimmed (d) Want, search
Hunched - to push; thrust. (e) Already, saw
The chief of the clan means head. Everyone follows him. Hence,
the men assembled or moved in a particular direction before the
Answers And Solutions
chief on his orders. Thus, option b i.e. I & IV is the correct answer. 461Ans.(b)
460Ans.(c) The above sentence, carries a sense of nostalgia and reminiscence.
Let's find out the meaning of given words :- “Seldom” is an adverb of time which doesn’t fit the connotation of
Mural - a painting or other work of art executed directly on a wall. the sentence and is thereby ruled out.
Whiff - a smell that is only smelt briefly or faintly; waft. “Often” sits in with the meaning the sentence wishes to express
Morrow - the following day; morning. which is the frequency with which the subject thinks about her
Zephyr - a soft gentle breeze. friends back at school.Though” is the only conjunction that sits
Ambit - the scope, extent, or bounds of something; boundary. with the adverb “even”.
The context explains that there was silence except in the far away Let us know the meanings of some of the words given above:
sea. And, trees would probably produce a sweet smell or help in Seldom means something or an event that doesn't occur frequently.
producing gentle breeze. Hence, option 3 i.e. II, IV & VI is the Often means something or an event that occurs quiet frequently.
correct answer. Thus, option 2 is the correct answer.
462Ans.(c)
In the above sentence, the filler has to be an adverb of time, as the
Directions (461-465 )There is a sentence with blank/s space/s in it.
sentence describes the frequency with which people believe in
Choose the word/s from the given options which fits
appropriately in both the blanks adding a proper and logical superstitions.
meaning to the sentences. Moreover, the latter filler is to be an preposition since it will be
explaining the relationship between the noun and the pronoun or
Q461. I ________ miss my friends from school even ______ we weren’t clause.
that close. Let us know the meaning of some of the words:
(a) Seldom, because (b) Often, though Seldom means something or an event which doesn't occur
(c) Want to, when (d) Don’t, because
frequently.
(e) Wont, If
“ Often means something or an event which occurs frequently.
Q462. People are __________ superstitious ___________ numbers, Thus, option c is the correct answer.
days and colors. 463Ans.(a)
(a) Seldom, more (b) Enjoying, about In the above sentence, the subject seems to be asking for something
(c) Often, about (d) Jealous, in “Asked” is the most apt filler, as the other fillers do not sit in
(e) Stuck-up, between accordance with the connotation of the sentence. The preposition
will be at.
Q463. I _________him once if he was not sometimes tired ______ night, “Enquired” is used for asking and doesn’t match the meaning of the
after working all day clause, thus it is ruled out.
(a) Asked, at (b) Enquired, around “Ordered” would be apt if the subject was ordering for the first
(c) Ordered, about (d) Pleaded, on time, however the use of “another round of drinks” clearly indicates
(e) Refused, in that it was the second order.

Facebook Page- https://www.facebook.com/vishalpariharpage Youtube- https://www.youtube.com/vishalparihar


Follow
81 Vishal Sir Telegram Channel- https://t.me/englishbyvishalsirchannel Instagram- https://www.instagram.com/vishalthetrainer
By Vishal sir

SBI PO, SBI CLERK, IBPS PO, IBPS CLERK, RRB PO, RRB CLERK, NIACL,
- LIC RBI grade B, RBI ASSISTANT, & Other competitive Exams

“Pleaded” doesn’t sit in accordance with the connotation, as diners .


never generally plead their servers.
“Refused” doesn’t agree with the clause and is thus ruled out. Q468. 1. The class together successfully pulled off the performance by
Let us know the meanings of some of the words given above: pooling their ________.
Asked means to say something in order to obtain an answer or some 2. It is given that to have an underlying ________ for a field, means
information. to be good at it if pursued.
Enquired means to ask for information from someone. (a) Savings (b) Regards
Ordered means to give an authoritative instruction to do something. (c) Ken (d) Resources
Pleaded means to make an emotional appeal. (e) Time
Refused means to indicate or show that one is not willing to do Q469. 1. The topic at hand seemed ________ for the present company
something. judging by their slackness.
Thus, option 1 is the correct answer. 2. The ___________ caption is cited just to emphasize the
464Ans.(c)
importance of a foreign correspondent's knowledge of the country
In the above sentence,
The first filler, is to be a preposition as it would show the he covers to the writing of good stories.
relationship between the nouns and the clause. (a) Prodigious (b) Solitary
The latter filler is to be a preposition of place. (c) Divergent (d) Doddery
Thus, option 3 is the correct answer. (e) Jejune
Q470. 1. Even though the store guard was dressed in the ____________
465Ans.(b) of a normal shopper, it was obvious he was a part of the store’s
In the above sentence, the first filler will be “have” in accordance
security team.
with the rules of present perfect.
The second filler would be a verb which would mean to look 2. The blood check revealed their _________ leading to the
through something without the intention of finding something . unexpected arrest.
Let us know the meanings of some of the words given above: (a) Apparatus
Rummage means to search unsystematically and untidily through (b) Build
something. (c) Guise
Skimmed means to remove a substance from the surface of a liquid. (d) Eminence
Thus, option 2 is the correct answer.
(e) Boost
Direction (466-470 ): Each of the following questions has two sentences Answers And Solutions
with one blank each. Choose the option that best fits in both the
blanks. 466Ans.(d)
Q466. 1. There was no need to tell her to channel her _________ side upon Mocking means teasing, amicable means polite, regal means
hearing the announcement. majestic, tenacious means stubborn and religious means devoted.
2. Even though Jackson was smaller than his other teammates, his The answer is tenacious as it makes the most appropriate sense.
__________ attitude allowed him to accomplish as much as they Thus, option 4 is the correct answer.
did. 467Ans.(a)
(a) Mocking (b) Amicable Lucid means easily understandable and clear, intimate means
(c) Regal (d) Tenacious something close and personal. The answer is lucid because water is
(e) Religious described as being clear and dreams can be easy to understand,
whereas the other words do not fit in both blanks.
Q467. 1. He wasn't very _________, he didn't quite know where he was.
Thus, option a is the correct answer
2. Although the water was ________, they all reluctantly took a 468Ans.(c)
step back. The answer is ken, which means knowledge and comprehension. It
(a) Lucid (b) Calm fits in well in both of the blanks. The other words do not go together
(c) Intimate (d) Clean in both of the blanks.
(e) Exciting
Facebook Page- https://www.facebook.com/vishalpariharpage Youtube- https://www.youtube.com/vishalparihar
Follow
82 Vishal Sir Telegram Channel- https://t.me/englishbyvishalsirchannel Instagram- https://www.instagram.com/vishalthetrainer
By Vishal sir

SBI PO, SBI CLERK, IBPS PO, IBPS CLERK, RRB PO, RRB CLERK, NIACL,
- LIC RBI grade B, RBI ASSISTANT, & Other competitive Exams

Thus, option c is the correct answer. (a) C (b) D


469Ans.(e) (c) E (d) F
Prodigious means remarkable, solitary means alone, divergent (e) E
means different, doddery means weakness due to old age and jejune
means lacking interest or significance. The answer is jejune as it Q.474. _______ is something which is difficult to describe in words.
fits the most appropriately. A. Rapture B. Desolate
Thus, option 5 is the correct answer. C. Deplore D. Malaise
47/Ans.(c) E. Aptness F. Blissfulness
Apparatus means tools or machinery, build can refer to physical (a) F (b) C
build or some kind of raise or coming together, guise means an (c) F (d) D
external appearance, eminence means acknowledged superiority (e) F
and boost means encouragement. The answer is guise as it fits most :
appropriately.
Thus, option c is the correct answer. Q.475. Organ trafficking is the issue that needs _______ attention.
Directions (471-475)The following question consists of a sentence(s) A. Solemn B. Jocular
with one blank only. You are given six words as answer choices C. Uproarious D. Humorless
and from the six choices you have to pick up two correct E. Screaming F. Slight
answers, either of which will make the sentence meaningfully (a) A-D (b) B-D
complete. (c) A-E (d) E-F
(e) C-E
Q.471. Black money is basically the accumulation of income obtained Answers And Solutions
_______.
471Ans.(c)
A. Illicitly B. Spuriously
Let’s find out meaning of above sentences:
C. Legitimately D. Justifiably
Illicitly: To not allowed by law; unlawful or illegal;
E. Statutory F. Feloniously
Spuriously: Based on false ideas or bad reasoning.
(a) B (b) C
Legitimately: Allowed according to rules and laws.
(c) F (d) E
Justifiably: To prove or show something to be just right or
(e) F
reasonable.
Q.472. Your thoughts build your _______. Feloniously: To obtain or gain in villainous and clever way.
Statutory: Of relating to formal laws or statutes.
A. Illusion B. Fantasy
Since accumulation of black money is against the law that is
C. Idealism D. Authenticity
obtained by clever or false ways. Hence, the correct option is option
E. Verity F. Betoken
3 i.e. A and F.
(a) B (b) F
Completed sentence:
(c) F (d) C
Black money is basically the accumulation of income obtained
(e) E
illicitly/ feloniously.
472Ans.(d )
Q473. Domestic violence is among the serious problems that should not
Let’s find out the meaning of above options:
be _______.
Illusion: Something that is false or not real but seems to be real.
A. Heed B. Pooh-pooh
Fantasy: The act of imagining something.
C. Disdained D. Slighted
E. Transgressed F. Cognizance
Facebook Page- https://www.facebook.com/vishalpariharpage Youtube- https://www.youtube.com/vishalparihar
Follow
83 Vishal Sir Telegram Channel- https://t.me/englishbyvishalsirchannel Instagram- https://www.instagram.com/vishalthetrainer
By Vishal sir

SBI PO, SBI CLERK, IBPS PO, IBPS CLERK, RRB PO, RRB CLERK, NIACL,
- LIC RBI grade B, RBI ASSISTANT, & Other competitive Exams

Idealism: The attitude of a person who believes that it is possible to Now, looking at the context of the sentence, happiness cannot be
live according to very high standards of behavior and honesty. described in words, it comes naturally from our inner self, hence
Authenticity: Real or genuine; not copied or false. the correct option is option 1 i.e. A-F. B, C, D and E do not match
Verity: Something that is regarded as true. the context of the sentence.
Betoken: To be a sign of something. Completed sentence -
Firstly, the blank cannot be filled with a verb i.e. 'betoken'. Since, Rapture/ blissfulness is something which is difficult to describe in
a possessive pronoun your is given. Hence, options 2 & 3 are words.
eliminated. Looking at the context of the sentence, B and C match 475Ans.(c)
with it, as your thoughts build your ideas and imagination. And, A, Let’s find out meaning of the above sentences:
D and E are incorrect. So, option 4 i.e. B-C is the correct answer. Solemn: Very serious ; done or made sincerely.
Completed sentence : Jocular: Said or done as a joke.
Your thoughts build your idealism/fantasy. Uproarious: Very noisy or extremely funny.
473Ans.(e) Humorless: Lacking a sense of humor i.e.funny or amusing quality.
Let’s find out meaning of above sentences: Screaming : Very noticeable and difficult to ignore.
Heed: To pay attention to [advice, a warning etc] Slight: To treat with disrespect.
Pooh-Pooh: To think or say that something is not very good or not Since organ trafficking is the trade of human organs but doesn’t
true. seem as a crime because poor is suppressed by giving financial
Disdained: To have feeling of strong dislike or disapproval of help, that
something. is why it needs a serious attention. Hence the correct option is
Slighted: To neglect, take no notice of. option 3 i.e. A and E.
Transgressed: To do something that is not allowed. Completed sentence:
Cognizance: Knowledge or awareness of something. Organ trafficking is the issue that needs solemn screaming
As we know domestic violence is among serious problems and it attention.
should be discouraged/not allowed in the society. Hence D and E
i.e. option 5 is possible as this problem should not be ignored. A,B,
C and F do not match the context. So, option 5 i.e. D-E is the correct Directions (476-480)There is a sentence with blank/s space/s in it.
answer. Choose the word/s from the given options which fits
Completed sentence: appropriately in both the blanks adding a proper and logical
Domestic violence is among the serious problems that should not meaning to the sentences.
be slighted/transgressed. Q476. Rudyard Kipling was not only an _________ writer, but he also
474Ans.(a) won the Nobel Prize for Literature, becoming the youngest
Let’s find out meaning of above options _______________ till date.
Rapture: A state feeling of great happiness, pleasure and love. (a) Acknowledged, writer (b) Famous, laureate
Desolate: (of a place) uninhabited and giving an impression of (c) Acclaimed, recipient (d) Best-selling, Indian
bleak emptiness. Option 3 is the apt filler as, the other options do not put forward the
Blissfulness: Extremely or completely happy. connotation of him being celebrated due to his work.
Malaise: A slight or general feeling of not being healthy or happy. Thus, option c is the correct answer.
Q477. He __________ to be in a bad mood today, maybe he won’t
Aptness: Likely to do something; having a tendency to do
be_____________ us for drinks after work.
something. (a) Looks, catching (b) Seems, joining
Deplore: To hate or dislike something very much; to strongly (c) Wants, coming (d) Is, meeting
disapprove of. (e) Felt, driving

Facebook Page- https://www.facebook.com/vishalpariharpage Youtube- https://www.youtube.com/vishalparihar


Follow
84 Vishal Sir Telegram Channel- https://t.me/englishbyvishalsirchannel Instagram- https://www.instagram.com/vishalthetrainer
By Vishal sir

SBI PO, SBI CLERK, IBPS PO, IBPS CLERK, RRB PO, RRB CLERK, NIACL,
- LIC RBI grade B, RBI ASSISTANT, & Other competitive Exams

Q478. I heard a playful _______ from my dog, as he looked at me with is the correct option, as it delivers the sense of uncertainty which is
____________ expecting me to play fetch. further supported by the use of “maybe” in the clause. While, the
(a) Yelp, anger other options do not match with the syntax as they do not sit in with
(b) Bark, excitement the preposition “to”.
(c) Meow, sadness "Joining" is the correct option, as it tells us that they are going out
(d) Squeal, enthusiasm as a group after work to drink.
(e) Snort, hope Hence, the other options do not sit in with the connotation of the
sentence.
Q479. Rohan was _____________ scolded by his mother,___________ Thus, option b is the correct answer.
misplacing the house keys. 478Ans.(b)
(a) Harshly, by (b) Softly, with In the above context, the subject is talking about her dog. Thus,
(c) Reprimanded, for (d) Severely, for options 3,4 and 5 are rules out since, they are not sounds emitted
(e) Hardly, to by dogs.
Dogs bark when they are generally angry or irritated or are trying
Q480. Meredith has achieved to ____________ her dream of becoming to catch our attention, whereas an yelp is a sound emitted by dogs
a doctor. Her friends and family are very __________ of her. when they are in pain. Also, anger is not the emotion of the dog as
Accomplish, proud Become, proud it is given in the sentence, that it has a look of expectation and was
Quit, proud Passion, embarrassed in a playful mood.
Plan, worried Let us see the meaning of some of the words given above.
(f) Known, European Yelp means a short, sharp cry especially of pain or alarm.
Squeal means a long high-pitched cry or noise.
Answers And Solutions Snort means an explosive sound made by the sudden forcing of
breath through one's nose, used to express indignation, derision or
incredulity.
476Ans.(c) Thus, option b is the correct answer.
Let us examine the meaning of the above given alternatives: 479Ans.(d)
Acknowledged means to recognize someone’s work to be good, In the above sentence, the situation tells us that the subject was
important, or legitimate; writer means a person who writes for a scolded by his mother as he had misplaced the keys.
living or for recognition. Harshly is an apt adverb, however the preposition “by” isn’t in
accordance with the syntax of the clause, thus option 1 is ruled out.
Famous means to be known by many people.
Softly, isn’t an adverb that best describes the verb in this context,
Laureate means a person who is honored with an award for hence, it is ruled out.
outstanding creative or intellectual achievement . Reprimanded is a verb and hence, is incorrect.
Acclaimed means to be publically praised or celebrated. Severely, is the apt adverb in the above context and the preposition
Recipient means a person or thing that receives or is awarded “for” sits in accordance with the syntax of the clause.
something. Hardly, again doesn’t describe the verb in the context in an apt
manner.
Best-selling means to be selling in greater quantities than others of
Let us now know the meanings of some of the words given above:
the same kind.
Harshly means an action performed in a manner that is unpleasantly
Indian means a person born in India.
rough or jarring to the senses.
Known means to be recognized, familiar or within scope of
Reprimanded is the past participle of the verb Reprimand. It means
knowledge.
a formal expression of disapproval.
European means a person born in Europe.
Thus, option d is the correct answer.
Rudyard Kipling is a very famous poet, known for his works world
480Ans.(a)
wide. In the above context, we are talking about his skill as a writer.
477Ans.(b) In the above context, it is quite understood that there is a sense of
achieving one’s goals, thus “accomplish” is the most apt verb.
In the above sentence, it is very important to pay attention to the
The other options are not viable as: “become” doesn’t sit in with
preposition in the first part and the clause in the latter part. “Seems”
the syntax of the sentence; “quit” doesn’t put forward the sense of
Facebook Page- https://www.facebook.com/vishalpariharpage Youtube- https://www.youtube.com/vishalparihar
Follow
85 Vishal Sir Telegram Channel- https://t.me/englishbyvishalsirchannel Instagram- https://www.instagram.com/vishalthetrainer
By Vishal sir

SBI PO, SBI CLERK, IBPS PO, IBPS CLERK, RRB PO, RRB CLERK, NIACL,
- LIC RBI grade B, RBI ASSISTANT, & Other competitive Exams

joy and achievement; “passion” is a noun; “plan” doesn’t sit in (e) BC


accordance with the syntax. Q485. Swamps are _________ and productive because of, not in spite of,
Let us know the meanings of some of the words given above: their diversity.
Accomplish means to achieve or complete something successfully. A. Waste
Passion means something that brings about strong emotions within B. Fecund
the subject.
C. Fertile
Thus, option a is the correct answer.
D. Useless
Directions (481-485)The following question consists of a sentence(s) (a) CD (b) BC
with one blank only. You are given four words as answer (c) AD (d) AB
choices and from the four choices you have to pick up two (e) BD
correct answers, either of which will make the sentence
meaningfully complete. Answers And Solutions
Q481. My mum died too young to see me turn from an ungrateful,
_________ teenager into a dutiful son and father. 481Ans.(c)
A. Belligerent B. Truculent Belligerent is hostile and aggressive.
C. Introvert D. Stupid Truculent means eager or quick to argue or fight, aggressive.
(a) CD (b) AC Introvert is someone who is shy, quiet, and unable to make friends
(c) AB (d) DB easily.
(e) AD Stupid is someone with a lack of intelligence or common sense.
Thus, both A and B make the sentence meaningful.
Q482. Mr. Comey recorded that Mr. Trump raised the story in multiple Thus option c is the correct answer.
meetings, asking the FBI director to investigate and rebut the 482Ans.(a)
allegation, a mission from which Mr. Comey _________. Demur means to raise objections or show reluctance.
A. Demurred B. Obscene Obscene means offensive or disgusting by accepted standards of
C. Objected D. Heralded morality and decency.
(a) AC (b) CD Object means to feel or express opposition to something or
(c) BA (d) AD someone.
(e) BD Herald means to be a sign that something important, and often
good, is starting to happen.
Q483. Millions of people experience severe migraines so often that they Thus, both A and C make the sentence meaningful.
are disabled and in ____. Thus option a is the correct answer.
A. Despair B. Coddle 483Ans.(b)
C. Desperation D. Impost Despair is the complete loss or absence of hope.
(a) BD (b) AC Coddle means to treat someone in an overprotective and kind way
(c) AD (d) BC Desperation also means loss of hope and feeling of despair.
(e) CD Impost is a tax or similar compulsory payment.
Thus, both A and C make the sentence meaningful.
Q484. And they offered trenchant observations and analyses that remain Thus option b is the correct answer.
_________ and instructive. 484Ans.(e)
A. Heedful Heedful means aware and careful.
B. Relevant Relevant means closely connected or appropriate to what is being
C. Apposite done or considered.
D. Absorb Apposite means suitable and right for the occasion.
(a) BA (b) BD Absorb means take in or soak up.
(c) AD (d) AC Thus, both B and C make the sentence meaningful.

Facebook Page- https://www.facebook.com/vishalpariharpage Youtube- https://www.youtube.com/vishalparihar


Follow
86 Vishal Sir Telegram Channel- https://t.me/englishbyvishalsirchannel Instagram- https://www.instagram.com/vishalthetrainer
By Vishal sir

SBI PO, SBI CLERK, IBPS PO, IBPS CLERK, RRB PO, RRB CLERK, NIACL,
- LIC RBI grade B, RBI ASSISTANT, & Other competitive Exams

Thus option 5 is the correct answer. (c)Considerate (d)Attribute


485Ans.(b) (e)Appreciable
Waste means something no longer of use. Answers And Solutions
Fecund means able to produce a lot of crops, fruit, babies, young
486Ans. (b)
animals, etc.
Sol. The most appropriate option to be filled in the blank is option (b).
Fertile means producing or capable of producing abundant
Impermeable- not allowing fluid to pass through
vegetation or crops.
Impertinent - not showing proper respect; rude
Useless means having no ability or skill.
Treacherous- guilty of or involving betrayal or deception
Thus, both B and C make the sentence meaningful.
487Ans. (a)
Thus option b is the correct answer.
Sol. The most appropriate option to be filled in the blank is option (a).
Retreat- move back or withdraw
Directions (486-490): Each question below has one blank, which is
Resolved- firmly determined to do something
indicating that something has been omitted. Find out which
Resurrected- restore (a dead person) to life
option can be used to fill up the blank in the sentence in the
488Ans. (c)
same sequence to make it meaningfully complete.
Sol. The most appropriate option to be filled in the blank is option (c).
Discriminate- recognize or perceive the difference
Q486. The sand bags placed on the river formed an __________barrier,
Detrimental- tending to cause harm
protecting the town from flooding.
489Ans. (d)
(a)Treacherous (b)Impermeable
Sol. The most appropriate option to be filled in the blank is option (d).
(c)Impertinent (d) Improper
Robust- strong and healthy; vigorous
(e)Exhaustive
Converse- engage in conversation
Q487. After much thought, Ted _________not to travel abroad this
490Ans. (e)
summer because he didn’t have much money in his account.
Sol. The most appropriate option to be filled in the blank is option (e).
(a)Resolved (b)Resurrected
Appreciable- large enough to be noticed
(c)Remembered (d)Retreat
Apprehend- arrest (someone) for a crime
(e)Revolved
Considerate- careful not to inconvenience or harm others
Attribute- regard something as being caused by
Q488. Priya couldn’t ___________between a good wine and a bad wine,
so she avoided wine tastings.
(a)Detrimental (b)Determination
(c)Discriminate (d)Realized Directions (491-495): In each of the questions given below a sentence
(e)Fight is given with one blank. Below each sentence FOUR words are
given out of which two can fit the sentence. Five options are
Q489. Himanshu preferred flavorless and mild juice, but Bhavesh given with various combinations of these words. You have to
preferred a juice with more _________flavor. choose the combination with the correct set of words which can
(a)Proposal (b)Productive fit in the given sentence.
(c)Ceased (d)Robust
(e)Converse Q491. An amazing transformation has _________ in the course of a
generation.
Q490. There is a/an ___________difference between those who say they (A) occurred (B) emerged
can get the job done and those who actually get the job done. (C) diluted (D) changed
(a)Apprehend (b)Alter (a) A-B (b) B-D
Facebook Page- https://www.facebook.com/vishalpariharpage Youtube- https://www.youtube.com/vishalparihar
Follow
87 Vishal Sir Telegram Channel- https://t.me/englishbyvishalsirchannel Instagram- https://www.instagram.com/vishalthetrainer
By Vishal sir

SBI PO, SBI CLERK, IBPS PO, IBPS CLERK, RRB PO, RRB CLERK, NIACL,
- LIC RBI grade B, RBI ASSISTANT, & Other competitive Exams

(c) C-D (d) B-C Sol. ‘Bankruptcy’ and ‘indebtedness’ is the correct choice of the words.
(e) A-C The conjunction ‘and’ is used to connect the word ‘insolvency’
with the other word which should be similar in meaning with the
Q492. The Insolvency and _________ Code has now been in force for previous word. Thus, option(b) is correct.
more than a year and given its ambitious objectives and impact. 493Ans. (c)
(A) economical (B) bankruptcy Sol. Option (c) is the correct choice. ‘component’ means a part or
(C) indebtedness (D) inflation element of a larger whole, especially a part of a machine or vehicle
(a) A-C (b) B-C and ‘ingredient’ means a component part or element of something.
(c) C-D (d) D-A Hence, they can be substituted for each other
(e) A-B 494Ans. (c)
Sol. Option (c) is the right choice. ‘contemporaneous’ means existing at
Q493. Investments in public infrastructure is a key _________ of or occurring in the same period of time. Hence, contemporaneous
economic growth strategy among emerging economies. and coexistent can be used interchangeably.
(A) division (B) structure 49Ans. (a)
(C) component (D) ingredient Sol. Option (a) is the correct choice. The preposition ‘behind’ and ‘for’
(a) A-B (b) B-C can be used to substitute each other.
(c) C-D (d) A-D
(e) B-D Directions (496–500): In each of the following questions given below
there are three blanks, each blank indicates that something has
Q494. Improvements can also come from changes in other
been omitted. Choose the set of words for each blank which best
______________ environmental or policy factors.
(A) coexistent (B) corelated fits to the meaning of the sentence as a whole.
(C) auxiliary (D) contemporaneous
(a) B-C (b) C-D
Q496. Not that Ms. Yellen sees the healthy expansion _________ by
(c) A-D (d) B-D
(e) A-B household spending weakening, the _________ from the recent
hurricanes that have _________ coastal regions in the U.S. south
Q495. The reason __________ the lack of research is that infrastructure is
notwithstanding.
not placed randomly across regions.
(A) towards (B) behind (a) weakened, desecrate, serrated
(C) into (D) for (b) strutted, molest, assisted
(a) B-D (b) D-C (c) contradicted, violate, frayed
(c) C-A (d) A-B
(e) A-D (d) undermined, taint, fragmented

Answers And Solutions (e) undergirded, damage, battered


491Ans. (a) Q497. While the ongoing moderate expansion in the U.S. bodes well for
Sol. ‘Occurred’ means happened or taken place which is similar in the country’s _______ exporters, the end of easy money conditions
meaning to ‘emerged’ in the context of the sentence. Thus, option
could ______ a slowdown in investment inflows from abroad and
(a) is correct.
492Ans. (b) resultant pressure on the current account __________.
(a) vehemently, predict, adequacy

Facebook Page- https://www.facebook.com/vishalpariharpage Youtube- https://www.youtube.com/vishalparihar


Follow
88 Vishal Sir Telegram Channel- https://t.me/englishbyvishalsirchannel Instagram- https://www.instagram.com/vishalthetrainer
By Vishal sir

SBI PO, SBI CLERK, IBPS PO, IBPS CLERK, RRB PO, RRB CLERK, NIACL,
- LIC RBI grade B, RBI ASSISTANT, & Other competitive Exams

(b) neglecting, suggest, failure Answers And Solutions


(c) struggling, augur, deficit 496Ans. (e)
(d) anguishing, reckon, flaw Sol. ‘undergirded, damage, battered’ is a correct set of words which are
making the sentence meaningful.
(e) fiercing, signify, fault
Undergirded means provide support or a firm basis for.
Q498. Given that monetary authorities in the U.S. are _______ on Taint means contaminate or pollute.
reflating the economy by ________ “further strengthening” in the Battered means injured by repeated blows or punishment.
labour market through an accommodative stance, the central bank Serrated means having or denoting a jagged edge; sawlike.
497Ans. (c)
has to remain _______ in warding off any let-up in expansionary Sol. ‘struggling, augur, deficit’ is a correct set of words which are
momentum. making the sentence meaningful.
(a) intensified, complying, studious Vehemently means in a forceful, passionate, or intense manner;
with great feeling.
(b) focused, supporting, vigilant
Augur means portend a good or bad outcome.
(c) unified, contributing, enthralled 498Ans. (b)
(d) gathered, ratifying, fascinated Sol. ‘focused, supporting, vigilant’ is a correct set of words which are
making the sentence meaningful.
(e) consolidated, consenting, careless
Enthralled means capture the fascinated attention of.
Q499. The U.S. Defence Secretary’s trip will happen _________ a month Studious means spending a lot of time studying or reading.
after President Donald Trump ________ the latest U.S. policy on 499Ans. (a)
Sol. ‘barely, announced, welcomed’ is a correct set of words which are
Afghanistan on August 21, a blueprint that has been _______ in
making the sentence meaningful.
Kabul and criticised in Islamabad. Passably means in a way that is just good enough, reasonably.
(a) barely, announced, welcomed Anointed means smear or rub with oil, typically as part of a
(b) fairly, suppressed, sanctioned religious ceremony.
Touted means attempt to sell (something).
(c) gradually, drafted, touted 500Ans. (c)
(d) passably, anointed, secured Sol. ‘peace, embedded, renunciation’ is a correct set of words which are
(e) presently, delivered, established making the sentence meaningful.
Enchased means decorate (a piece of jewellery or work of art) by
Q500. Indeed ______ and reconciliation were ________ in the very first
inlaying, engraving, or carving.
international compact on Afghanistan, in the text of the Bonn Equanimity means calmness and composure.
Agreement of December 2001. ___________ of violence will help Entrenched means firmly established and difficult or unlikely to
change, ingrained.
this process.
Screwed means in a difficult or hopeless situation, ruined or
(a) equanimity, entrenched, Abstaining broken.
(b) ecstasy, imprisoned, sobriety Conformity means compliance with standards, rules, or laws.
(c) peace, embedded, renunciation
(d) serenity, enchased, withdrawing
.
(e) conformity, screwed, evasion

Facebook Page- https://www.facebook.com/vishalpariharpage Youtube- https://www.youtube.com/vishalparihar


Follow
89 Vishal Sir Telegram Channel- https://t.me/englishbyvishalsirchannel Instagram- https://www.instagram.com/vishalthetrainer
By Vishal sir

SBI PO, SBI CLERK, IBPS PO, IBPS CLERK, RRB PO, RRB CLERK, NIACL,
- LIC RBI grade B, RBI ASSISTANT, & Other competitive Exams

Facebook Page- https://www.facebook.com/vishalpariharpage Youtube- https://www.youtube.com/vishalparihar


Follow
90 Vishal Sir Telegram Channel- https://t.me/englishbyvishalsirchannel Instagram- https://www.instagram.com/vishalthetrainer

You might also like